Re: [Wien] confusion regarding band structure of Graphene

2022-12-30 Thread xavier rocquefelte

OK :) Using HSE06 allows to do not have to do correction.

But you must be aware that you are in the error bar of the method. It is 
why I was surprised by this calculation (between materials used as 
negative electrodes).


The error bar in HSE is about 0.1, 0.2 and can reach 0.4 V in some 
cases, as shown in this paper: 
https://www.nature.com/articles/npjcompumats20162.


Best regards

Xavier




On 30/12/2022 17:44, shamik chakrabarti wrote:

Dear Prof. Xavier,

                   I have used HSE06 to simulate both C6 (graphene) & 
LiC6 with monoclinic structure & obtained 0.12 V (with respect to Li 
cathode) which matches perfectly with the experimental report.


Looking forward to your further advice Sir.

with regards,





On Fri, 30 Dec 2022 at 21:01, xavier rocquefelte 
 wrote:


One more point the negative electrode is usually graphite and not
graphene.

To estimate a voltage you need a positive electrode.

Estimating the voltage implies to properly treat not only the
negative but the positive electrode materials and with the same
functional.

Best regards

Xavier

On 30/12/2022 16:26, xavier rocquefelte wrote:


Dear Shamik

I certainly did not follow all the discussion but it seems to me
that your objective is reachable if you properly define your
parameters.

You mention that the band structure is finally not the essential
point and you want to have a proper estimation of the voltage.

Here are two well-known problems:

- Li batteries voltage estimation is usually wrong and
corrections must be applied (depending on the functional). I
recommend the publications of Ceder's group.

- The choice of the functional may be crucial and for carbon GGA
is sometimes not the best.

I will thus recommend first to choose your functional by
estimating properly the band structure of the hexagonal cell of
graphene and only after switch to the monoclinic cell to estimate
the voltage.

Best regards

Xavier

On 30/12/2022 15:51, shamik chakrabarti wrote:

Dear Prof. Blaha,

                    I think the structure as used by me & as can
be used by downloading Graphene CIF file from material project
database are same with different representation. I have checked
the XRD of both the structures in VESTA & they are providing the
same XRD pattern for both the structures. In this regard
lithiation voltage for both the cases should be same. However
the bandstructures need special k points which is available for
hexagonal lattice & not for this monoclinic lattice. However, if
the voltage is accurate I will not bother for the band structure.

with kind regards,

On Fri, 30 Dec 2022 at 19:08, shamik chakrabarti
 wrote:

Specially if I want LiC6 unit cell?...

On Fri, Dec 30, 2022, 18:47 shamik chakrabarti
 wrote:

Dear Prof. Blaha,
 .   .   .  .  .  .  Thank you for your reply Sir. I
have a very dumb question. Whether the voltage simulated
by this attached structure will provide the same as will
be provided by the single unit cell?

With kind regards,

On Fri, Dec 30, 2022, 18:03 Peter Blaha
 wrote:

This is not the smallest unit cell of graphene but a
3 times larger
monoclinic cell.

Start with graphite, remove the atoms at the second
z=0.75 layer and
eventually add some vacuum along z.


Am 29.12.2022 um 18:34 schrieb shamik chakrabarti:
> The structure of monolayer Graphenbe as I have
used is attached for your
> reference.
>
> On Thu, 29 Dec 2022 at 18:32, shamik chakrabarti
> mailto:shamik15041...@gmail.com>> wrote:
>
>     Dear Wien2k users,
>
>                              I have simulated a
structure of Graphene
>     monolayer with primitive cell. I am getting
DOS & voltage
>     accurately. However, while plotting band
structure I am facing a
>     problem. In hexagonal Graphene the band
structure can be obtained
>     with Gamma-M-K-Gamma, where K is the dirac
point. However, in my
>     primitive cell I am getting a Brillouin Zone
of reciprocal lattice &
>     not able to detect the Dirac point.
>
>     The purpose of taking a primitive cell of
monolayer Graphene lies on
>     the fact that I need control over all 

Re: [Wien] confusion regarding band structure of Graphene

2022-12-30 Thread xavier rocquefelte

One more point the negative electrode is usually graphite and not graphene.

To estimate a voltage you need a positive electrode.

Estimating the voltage implies to properly treat not only the negative 
but the positive electrode materials and with the same functional.


Best regards

Xavier

On 30/12/2022 16:26, xavier rocquefelte wrote:


Dear Shamik

I certainly did not follow all the discussion but it seems to me that 
your objective is reachable if you properly define your parameters.


You mention that the band structure is finally not the essential point 
and you want to have a proper estimation of the voltage.


Here are two well-known problems:

- Li batteries voltage estimation is usually wrong and corrections 
must be applied (depending on the functional). I recommend the 
publications of Ceder's group.


- The choice of the functional may be crucial and for carbon GGA is 
sometimes not the best.


I will thus recommend first to choose your functional by estimating 
properly the band structure of the hexagonal cell of graphene and only 
after switch to the monoclinic cell to estimate the voltage.


Best regards

Xavier

On 30/12/2022 15:51, shamik chakrabarti wrote:

Dear Prof. Blaha,

                    I think the structure as used by me & as can be 
used by downloading Graphene CIF file from material project database 
are same with different representation. I have checked the XRD of 
both the structures in VESTA & they are providing the same XRD 
pattern for both the structures. In this regard lithiation voltage 
for both the cases should be same. However the bandstructures need 
special k points which is available for hexagonal lattice & not for 
this monoclinic lattice. However, if the voltage is accurate I will 
not bother for the band structure.


with kind regards,

On Fri, 30 Dec 2022 at 19:08, shamik chakrabarti 
 wrote:


Specially if I want LiC6 unit cell?...

On Fri, Dec 30, 2022, 18:47 shamik chakrabarti
 wrote:

Dear Prof. Blaha,
 .   .   .  .  .  .  Thank you for your reply Sir. I have a
very dumb question. Whether the voltage simulated by this
attached structure will provide the same as will be provided
by the single unit cell?

With kind regards,

On Fri, Dec 30, 2022, 18:03 Peter Blaha
 wrote:

This is not the smallest unit cell of graphene but a 3
times larger
monoclinic cell.

Start with graphite, remove the atoms at the second
z=0.75 layer and
eventually add some vacuum along z.


Am 29.12.2022 um 18:34 schrieb shamik chakrabarti:
> The structure of monolayer Graphenbe as I have used is
attached for your
> reference.
>
> On Thu, 29 Dec 2022 at 18:32, shamik chakrabarti
> mailto:shamik15041...@gmail.com>> wrote:
>
>     Dear Wien2k users,
>
>                              I have simulated a
structure of Graphene
>     monolayer with primitive cell. I am getting DOS &
voltage
>     accurately. However, while plotting band structure
I am facing a
>     problem. In hexagonal Graphene the band structure
can be obtained
>     with Gamma-M-K-Gamma, where K is the dirac point.
However, in my
>     primitive cell I am getting a Brillouin Zone of
reciprocal lattice &
>     not able to detect the Dirac point.
>
>     The purpose of taking a primitive cell of monolayer
Graphene lies on
>     the fact that I need control over all the C atoms.
>
>     Looking forward to hearing from you.
>
>     --
>     Dr. Shamik Chakrabarti
>     Research Fellow
>     Department of Physics
>     Indian Institute of Technology Patna
>     Bihta-801103
>     Patna
>     Bihar, India
>
>
>
> --
> Dr. Shamik Chakrabarti
> Research Fellow
> Department of Physics
> Indian Institute of Technology Patna
> Bihta-801103
> Patna
> Bihar, India
>
> ___
> Wien mailing list
> Wien@zeus.theochem.tuwien.ac.at
> http://zeus.theochem.tuwien.ac.at/mailman/listinfo/wien
> SEARCH the MAILING-LIST at:

http://www.mail-archive.com/wien@zeus.theochem.tuwien.ac.at/index.html

-- 
--

Re: [Wien] confusion regarding band structure of Graphene

2022-12-30 Thread xavier rocquefelte
-
___
Wien mailing list
Wien@zeus.theochem.tuwien.ac.at
http://zeus.theochem.tuwien.ac.at/mailman/listinfo/wien
SEARCH the MAILING-LIST at:

http://www.mail-archive.com/wien@zeus.theochem.tuwien.ac.at/index.html



--
Dr. Shamik Chakrabarti
Research Fellow
Department of Physics
Indian Institute of Technology Patna
Bihta-801103
Patna
Bihar, India

___
Wien mailing list
Wien@zeus.theochem.tuwien.ac.at
http://zeus.theochem.tuwien.ac.at/mailman/listinfo/wien
SEARCH the MAILING-LIST 
at:http://www.mail-archive.com/wien@zeus.theochem.tuwien.ac.at/index.html


--
----
Professeur des Universités de Rennes 1
Institut des Sciences Chimiques de Rennes (ISCR)
Univ Rennes - CNRS - UMR6226, France
https://iscr.univ-rennes1.fr/xavier-rocquefelte

___
Wien mailing list
Wien@zeus.theochem.tuwien.ac.at
http://zeus.theochem.tuwien.ac.at/mailman/listinfo/wien
SEARCH the MAILING-LIST at:  
http://www.mail-archive.com/wien@zeus.theochem.tuwien.ac.at/index.html


Re: [Wien] Spin-polarized state not really spin-polarized

2022-10-30 Thread xavier rocquefelte
hem.tuwien.ac.at
http://zeus.theochem.tuwien.ac.at/mailman/listinfo/wien
SEARCH the MAILING-LIST at: 
http://www.mail-archive.com/wien@zeus.theochem.tuwien.ac.at/index.html


Pascal Boulet
—
/Professor in computational materials chemistry - DEPARTMENT OF CHEMISTRY/
University of Aix-Marseille - Avenue Escadrille Normandie Niemen - 
F-13013 Marseille - FRANCE

Tél: +33(0)4 13 55 18 10 - Fax : +33(0)4 13 55 18 50
Email : pascal.bou...@univ-amu.fr





___
Wien mailing list
Wien@zeus.theochem.tuwien.ac.at
http://zeus.theochem.tuwien.ac.at/mailman/listinfo/wien
SEARCH the MAILING-LIST 
at:http://www.mail-archive.com/wien@zeus.theochem.tuwien.ac.at/index.html


--

Professeur des Universités de Rennes 1
Institut des Sciences Chimiques de Rennes (ISCR)
Univ Rennes - CNRS - UMR6226, France
https://iscr.univ-rennes1.fr/xavier-rocquefelte

___
Wien mailing list
Wien@zeus.theochem.tuwien.ac.at
http://zeus.theochem.tuwien.ac.at/mailman/listinfo/wien
SEARCH the MAILING-LIST at:  
http://www.mail-archive.com/wien@zeus.theochem.tuwien.ac.at/index.html


Re: [Wien] Question about WIEN2k and paramagnetic system

2022-05-19 Thread xavier rocquefelte
In addition to the comments of Lyudmila and Peter I would add few 
questions:


- is your system intermetallic? (is your system both involving localized 
and delocalized states?)


- is the magnetic element based on 4f or 5f states? (require more than a 
single-determinant treatment but in some cases can be treated 
approximately using DFT+SO+U)


- which states are near the Fermi level?  (states of NM elements?)

- which simulated properties do not agree with experiments? (optical?)

Best regards
Xavier


On 19/05/2022 18:44, Lyudmila Dobysheva wrote:

19.05.2022 19:22, Tenzin, K. wrote:
I have a crystal system composed of three heavy atoms (minimum atomic 
number of 60), one of which has quite a large magnetic moment (about 
10) while the other two are non-magnetic.  Experimentally, the 
overall crystal is found to be paramagnetic but with quite large 
magnetic moments (about 10 as well) and it is speculated to be a weyl 
semimetal.
I performed DFT calculation for the crystal using wien2k (non 
spin-polarized with MBJLDA) and some calculated properties (such as 
quantum oscillation) do agree with experiments. But there are also 
some properties that do not agree with experiments.
So my question is how do I correctly model such a crystal system, 
paramagnetic with relatively large magnetic moment? Should I perform 
non spin-polarized calculation and later add external magnetic field, 
to mimic intrinsic magnetic moment, via -orb scheme?  Is it even 
possible to run_lapw -orb for non spin polarization calculation?


I'd say that making the common spin-polarized calculation should give 
you the magnetic moments, in ferromagnetic structure. To simulate 
paramagnetic, I'd construct antiferromagnetic arrangement and compare 
three energies: FM, AFM, and nonmagnetic NM which you have obtained. 
If the FM and AFM are closer to each other, as compared to the NM, you 
can think about this. For some systems the AFM is a good simulation of 
paramagnetism.


Best wishes
Lyudmila Dobysheva
--
http://ftiudm.ru/content/view/25/103/lang,english/
Institute of Physics and Technology,
Udmurt Federal Research Center, Ural Br. of Rus.Ac.Sci.
426000 Izhevsk Kirov str. 132
Russia
---
Tel. +7 (34I2)43-24-59 (office), +7 (9I2)OI9-795O (home)
Skype: lyuka18 (office), lyuka17 (home)
E-mail: lyuk...@mail.ru (office), lyuk...@gmail.com (home)
___
Wien mailing list
Wien@zeus.theochem.tuwien.ac.at
http://zeus.theochem.tuwien.ac.at/mailman/listinfo/wien
SEARCH the MAILING-LIST at: 
http://www.mail-archive.com/wien@zeus.theochem.tuwien.ac.at/index.html


--

Professeur des Universités de Rennes 1
Institut des Sciences Chimiques de Rennes (ISCR)
Univ Rennes - CNRS - UMR6226, France
https://iscr.univ-rennes1.fr/xavier-rocquefelte


___
Wien mailing list
Wien@zeus.theochem.tuwien.ac.at
http://zeus.theochem.tuwien.ac.at/mailman/listinfo/wien
SEARCH the MAILING-LIST at:  
http://www.mail-archive.com/wien@zeus.theochem.tuwien.ac.at/index.html


Re: [Wien] A basic question regarding using GGA+U approach

2022-02-11 Thread xavier rocquefelte

You are right Peter.

Here is a paper of Ceder: https://www.nature.com/articles/npjcompumats20162

It is quite tricky.

Cheers

Xavier

On 11/02/2022 08:50, Peter Blaha wrote:

Hmm.

Depending on the metal, a hybrid DFT calculation for the metal is as 
problematic (or even more) that a LDA+U calculation.
It overestimates itinerant magnetic moments and in addition also 
affects the "free-electron" like 4s electrons ...


Formation energies (voltages) with correlated electrons are always 
"tricky". I think Ceder has published a lot of such calculations on 
voltages including various more or less empirical "tricks".


Eventually, you may try a meta-GGA ., but this is probably also 
not very good.


Best regards
Peter Blaha

Am 2/11/22 um 08:19 schrieb xavier rocquefelte:

Dear Shamik,

To my point of view using the strategy (1) is not correct. I 
understand that B will require a different treatment in ABS2 and pure 
B phases.


You certainly has no other choice than using hybrid functional for 
all calculations ... and then you will be able to compare to the 
results of strategy (2).


Best Regards

Xavier



On 11/02/2022 06:40, shamik chakrabarti wrote:

Dear Wien2k users,

                           I have studied the intercalation of A in 
BS2 to form ABS2. In this calculation, I have used Hubbard U for B 
in BS2, and in ABS2 & I got reasonable voltage.
However, now I want to study the voltage corresponding to the 
conversion reaction; ABS2 + A =2A2S +B. In this case, B is a metal & 
hence to simulate the voltage
(1) Should I need to consider the energy value corresponding to 
GGA+U approach applied to ABS2 or,
(2) Should I need to consider the energy value corresponding to GGA 
approach applied to ABS2


As A & A2S & B have been simulated using GGA.

Looking forward to your reply in this regard.

with regards,

--
Dr. Shamik Chakrabarti
Research Fellow
Department of Physics
Indian Institute of Technology Patna
Bihta-801103
Patna
Bihar, India

___
Wien mailing list
Wien@zeus.theochem.tuwien.ac.at
http://zeus.theochem.tuwien.ac.at/mailman/listinfo/wien
SEARCH the MAILING-LIST 
at:http://www.mail-archive.com/wien@zeus.theochem.tuwien.ac.at/index.html


--

Professeur des Universités de Rennes 1
Institut des Sciences Chimiques de Rennes (ISCR)
Univ Rennes - CNRS - UMR6226, France
https://iscr.univ-rennes1.fr/xavier-rocquefelte



___
Wien mailing list
Wien@zeus.theochem.tuwien.ac.at
http://zeus.theochem.tuwien.ac.at/mailman/listinfo/wien
SEARCH the MAILING-LIST at: 
http://www.mail-archive.com/wien@zeus.theochem.tuwien.ac.at/index.html



--

Professeur des Universités de Rennes 1
Institut des Sciences Chimiques de Rennes (ISCR)
Univ Rennes - CNRS - UMR6226, France
https://iscr.univ-rennes1.fr/xavier-rocquefelte


___
Wien mailing list
Wien@zeus.theochem.tuwien.ac.at
http://zeus.theochem.tuwien.ac.at/mailman/listinfo/wien
SEARCH the MAILING-LIST at:  
http://www.mail-archive.com/wien@zeus.theochem.tuwien.ac.at/index.html


Re: [Wien] How to find the exact value of infinte epsilon?

2021-11-29 Thread xavier rocquefelte

Dear Atefe Marasi,

Infinite epsilon means that you extrapolate the epsilon value for the 
zero of energy.


You must plot the real part of the dielectric function to properly 
estimate this value.


You must be careful because if you have a band gap and a bad description 
of the gap value ... the estimation will be not good at all.


Regards

Xavier


On 29/11/2021 11:38, Atefe Marasi wrote:

Hello dear WIEN2k users,
I am interested in finding the exact value of infinite epsilon, i.e., 
dielectric constant at high frequency. As you know, as frequency goes 
to infinity, the value of epsilon asymptotically approaches to unity. 
But data reported in literatures for the infinite epsilon are not 
equal to one! How can I choose a correct energy range or frequency to 
obtain the appropriate infinite epsilon?

Is there any way to find the infinite epsilon by WIEN2k?

Thank you,
Best Regards,
Atefe Marasi



___
Wien mailing list
Wien@zeus.theochem.tuwien.ac.at
http://zeus.theochem.tuwien.ac.at/mailman/listinfo/wien
SEARCH the MAILING-LIST 
at:http://www.mail-archive.com/wien@zeus.theochem.tuwien.ac.at/index.html___
Wien mailing list
Wien@zeus.theochem.tuwien.ac.at
http://zeus.theochem.tuwien.ac.at/mailman/listinfo/wien
SEARCH the MAILING-LIST at:  
http://www.mail-archive.com/wien@zeus.theochem.tuwien.ac.at/index.html


Re: [Wien] EECE with orbc

2021-02-15 Thread Xavier Rocquefelte

Thank you for your fast reply.

Indeed, I wanted to do the same, i.e. DFT+U and then EECE.

I agree that getting "the right spin & parity/eigenvectors for 4f is a 
pain" ;)


Best Regards

Xavier

Le 16/02/2021 à 08:20, Laurence Marks a écrit :
Unfortunately there is no way to do this currently with -eece. The 
.forceXxx force the unmixed dmat/vorb to be used in the next cycle 
which is sometimes useful.


It might work to do:
a) Run with U to get the state you want
b) save_lapw (or rm *bro*)
c) With the relevant case.ineece, do runeece_lapw (-p)
d) Now do runsp_lapw -eece ...
N.B, add -so if relevant.

Getting the right spin & parity/eigenvectors for 4f is a pain. There 
are ways to manipulate the spins; doing this for the 
parity/eigenvectors is harder.


_
Professor Laurence Marks
"Research is to see what everybody else has seen, and to think what 
nobody else has thought", Albert Szent-Gyorgi

www.numis.northwestern.edu <http://www.numis.northwestern.edu>

On Tue, Feb 16, 2021, 01:03 Xavier Rocquefelte 
<mailto:xavier.rocquefe...@univ-rennes1.fr>> wrote:


Dear Colleagues,

We are doing EECE calculations and we want to constrain the
electronic
configuration of Eu.

It is possible to do it using DFT+U and -orbc option.

Do you know if we can do similar calculations using EECE? We look at
runsp and we found the use of .forcedmat and .forceorb.

Could it be used for this purpose?

The other option will be to adapt runsp for such EECE constrained
calculations.

Best Regards

Xavier


-- 


Institut des Sciences Chimiques de Rennes (ISCR)
Univ Rennes - CNRS - UMR6226, France

https://urldefense.com/v3/__https://iscr.univ-rennes1.fr/cti/people/permanent-staff/rocquefelte-xavier__;!!Dq0X2DkFhyF93HkjWTBQKhk!G38zbfeI5iZ4j5VIfpEJyYWf7Iy3rWWEhMY_WiQRGGwoA7vLl3MMXx5DuVfaiO6WvFregg$

<https://urldefense.com/v3/__https://iscr.univ-rennes1.fr/cti/people/permanent-staff/rocquefelte-xavier__;!!Dq0X2DkFhyF93HkjWTBQKhk!G38zbfeI5iZ4j5VIfpEJyYWf7Iy3rWWEhMY_WiQRGGwoA7vLl3MMXx5DuVfaiO6WvFregg$>


ICAMM2019 : VASP Workshop and International Materials Modelling Conf
June 26-July 3 2019, Rennes France
icamm2019.sciencesconf.org <http://icamm2019.sciencesconf.org> :
Registration opening soon, see you there!

___
Wien mailing list
Wien@zeus.theochem.tuwien.ac.at
<mailto:Wien@zeus.theochem.tuwien.ac.at>

https://urldefense.com/v3/__http://zeus.theochem.tuwien.ac.at/mailman/listinfo/wien__;!!Dq0X2DkFhyF93HkjWTBQKhk!G38zbfeI5iZ4j5VIfpEJyYWf7Iy3rWWEhMY_WiQRGGwoA7vLl3MMXx5DuVfaiO7gDU64yw$

<https://urldefense.com/v3/__http://zeus.theochem.tuwien.ac.at/mailman/listinfo/wien__;!!Dq0X2DkFhyF93HkjWTBQKhk!G38zbfeI5iZ4j5VIfpEJyYWf7Iy3rWWEhMY_WiQRGGwoA7vLl3MMXx5DuVfaiO7gDU64yw$>

SEARCH the MAILING-LIST at:

https://urldefense.com/v3/__http://www.mail-archive.com/wien@zeus.theochem.tuwien.ac.at/index.html__;!!Dq0X2DkFhyF93HkjWTBQKhk!G38zbfeI5iZ4j5VIfpEJyYWf7Iy3rWWEhMY_WiQRGGwoA7vLl3MMXx5DuVfaiO4hTspOgw$

<https://urldefense.com/v3/__http://www.mail-archive.com/wien@zeus.theochem.tuwien.ac.at/index.html__;!!Dq0X2DkFhyF93HkjWTBQKhk!G38zbfeI5iZ4j5VIfpEJyYWf7Iy3rWWEhMY_WiQRGGwoA7vLl3MMXx5DuVfaiO4hTspOgw$>



___
Wien mailing list
Wien@zeus.theochem.tuwien.ac.at
http://zeus.theochem.tuwien.ac.at/mailman/listinfo/wien
SEARCH the MAILING-LIST at:  
http://www.mail-archive.com/wien@zeus.theochem.tuwien.ac.at/index.html


--

Institut des Sciences Chimiques de Rennes (ISCR)
Univ Rennes - CNRS - UMR6226, France
https://iscr.univ-rennes1.fr/cti/people/permanent-staff/rocquefelte-xavier

ICAMM2019 : VASP Workshop and International Materials Modelling Conf
June 26-July 3 2019, Rennes France
icamm2019.sciencesconf.org  : Registration opening soon, see you there!

___
Wien mailing list
Wien@zeus.theochem.tuwien.ac.at
http://zeus.theochem.tuwien.ac.at/mailman/listinfo/wien
SEARCH the MAILING-LIST at:  
http://www.mail-archive.com/wien@zeus.theochem.tuwien.ac.at/index.html


[Wien] EECE with orbc

2021-02-15 Thread Xavier Rocquefelte

Dear Colleagues,

We are doing EECE calculations and we want to constrain the electronic 
configuration of Eu.


It is possible to do it using DFT+U and -orbc option.

Do you know if we can do similar calculations using EECE? We look at 
runsp and we found the use of .forcedmat and .forceorb.


Could it be used for this purpose?

The other option will be to adapt runsp for such EECE constrained 
calculations.


Best Regards

Xavier


--

Institut des Sciences Chimiques de Rennes (ISCR)
Univ Rennes - CNRS - UMR6226, France
https://iscr.univ-rennes1.fr/cti/people/permanent-staff/rocquefelte-xavier

ICAMM2019 : VASP Workshop and International Materials Modelling Conf
June 26-July 3 2019, Rennes France
icamm2019.sciencesconf.org  : Registration opening soon, see you there!

___
Wien mailing list
Wien@zeus.theochem.tuwien.ac.at
http://zeus.theochem.tuwien.ac.at/mailman/listinfo/wien
SEARCH the MAILING-LIST at:  
http://www.mail-archive.com/wien@zeus.theochem.tuwien.ac.at/index.html


Re: [Wien] How to start ferromagnetic calculation

2020-07-13 Thread Xavier Rocquefelte
1. You cannot simulate paramagnetism using DFT. Indeed, you need to go 
beyond in order to take into account the effect of temperature on the 
dynamical disorder of the magnetic moments. Using DFT you will have 
access to a static picture, which is OK if you want to simulate:


- a non-magnetic state (approx. diamagnetic state)

- a long-range order (antiferromagnetic, ferromagnetic, ferrimagnetic...)

When you run a spin-polarized calculation using WIEN2k (runsp_lapw ...), 
you can check your magnetic order by doing:


grep :MM *.scf

You will have the total magnetic moment in the cell (non-zero if 
ferromagnetism) and the magnetic moments projected on each atomic sphere.


2. I don't think so. The AFM utility is here to force a specific 
antiferromagnetic order. In addition, it is not recommended to use it 
first. The best is to generate you magnetic order during the 
initialization step (lstart) and do a regular spin polarized calculation 
(runsp_lapw -ec  -cc ...). Then check the magnetic moment in the 
case.scf file.


Best regards

Xavier

Le 13/07/2020 à 09:52, Peeyush kumar kamlesh a écrit :

Hello wien2k users,
I am working on half-Heusler compounds. I have a few questions:
1. Which command should I use to know that my material is para, dia or 
ferromagnetic material?

2. Can I run ferromagnetic properties using the AFM program?
Thanks and Regards

___
Wien mailing list
Wien@zeus.theochem.tuwien.ac.at
http://zeus.theochem.tuwien.ac.at/mailman/listinfo/wien
SEARCH the MAILING-LIST at:  
http://www.mail-archive.com/wien@zeus.theochem.tuwien.ac.at/index.html


--

Institut des Sciences Chimiques de Rennes (ISCR)
Univ Rennes - CNRS - UMR6226, France
https://iscr.univ-rennes1.fr/cti/people/permanent-staff/rocquefelte-xavier

ICAMM2019 : VASP Workshop and International Materials Modelling Conf
June 26-July 3 2019, Rennes France
icamm2019.sciencesconf.org  : Registration opening soon, see you there!

___
Wien mailing list
Wien@zeus.theochem.tuwien.ac.at
http://zeus.theochem.tuwien.ac.at/mailman/listinfo/wien
SEARCH the MAILING-LIST at:  
http://www.mail-archive.com/wien@zeus.theochem.tuwien.ac.at/index.html


Re: [Wien] Structure optimization of Beta -Sn

2020-06-04 Thread Xavier Rocquefelte

This last input file leads to a correct structure.

Best regards

Xavier

Le 04/06/2020 à 11:11, shamik chakrabarti a écrit :

Dear Xavier Sir,

                  With the modified cif file & origin choice one, I 
get the structure as attached herewith this mail. Kindly let me know 
if it is the structure you are asking for. I am using wien2k 19.1.


with regards

On Thu, 4 Jun 2020 at 13:30, Tran, Fabien <mailto:fabien.t...@tuwien.ac.at>> wrote:


With your modified cif file, I obtain at the end of init_lapw a
struct file which has two atoms per cell. Since there is no atoms
at the corners, it is visually not 100% clear that it is the
correct beta-Sn, but I guess it is (?).

I am wondering where the problem with original cif file comes
from. From cif2struct or a non-conventional format of the cif
file? Maybe Peter knows.


From: Wien mailto:wien-boun...@zeus.theochem.tuwien.ac.at>> on behalf of
    Xavier Rocquefelte mailto:xavier.rocquefe...@univ-rennes1.fr>>
Sent: Thursday, June 4, 2020 9:28 AM
To: wien@zeus.theochem.tuwien.ac.at
<mailto:wien@zeus.theochem.tuwien.ac.at>
Subject: Re: [Wien] Structure optimization of Beta -Sn

Indeed, I used the one from VESTA! Yesteray night I was working in
parallel ;)

Here is the file I previously sent with modification by hand ...
the cif
format was not totally supported by cif2struct.

Regards

Xavier
___
Wien mailing list
Wien@zeus.theochem.tuwien.ac.at
<mailto:Wien@zeus.theochem.tuwien.ac.at>
http://zeus.theochem.tuwien.ac.at/mailman/listinfo/wien
SEARCH the MAILING-LIST at:
http://www.mail-archive.com/wien@zeus.theochem.tuwien.ac.at/index.html



--
Dr. Shamik Chakrabarti
Research Fellow
Department of Physics
Indian Institute of Technology Patna
Bihta-801103
Patna
Bihar, India

___
Wien mailing list
Wien@zeus.theochem.tuwien.ac.at
http://zeus.theochem.tuwien.ac.at/mailman/listinfo/wien
SEARCH the MAILING-LIST at:  
http://www.mail-archive.com/wien@zeus.theochem.tuwien.ac.at/index.html


--

Institut des Sciences Chimiques de Rennes (ISCR)
Univ Rennes - CNRS - UMR6226, France
https://iscr.univ-rennes1.fr/cti/people/permanent-staff/rocquefelte-xavier

ICAMM2019 : VASP Workshop and International Materials Modelling Conf
June 26-July 3 2019, Rennes France
icamm2019.sciencesconf.org  : Registration opening soon, see you there!

___
Wien mailing list
Wien@zeus.theochem.tuwien.ac.at
http://zeus.theochem.tuwien.ac.at/mailman/listinfo/wien
SEARCH the MAILING-LIST at:  
http://www.mail-archive.com/wien@zeus.theochem.tuwien.ac.at/index.html


Re: [Wien] Structure optimization of Beta -Sn

2020-06-04 Thread Xavier Rocquefelte
I have to remove "s" as you did and rewrite the loop with the symmetry 
operations.


It is a problem with the origin choice!

Regards

Xavier





Le 04/06/2020 à 09:33, Tran, Fabien a écrit :


​Dear Xavier,

Your cif file can be converted to struct, but only if I remove ​"s" as 
suggested by Shamik. I tried with cif2struct from old and new versions 
of cWIEN2k. But, I still get the wrong structure at the end of 
init_lapw (4 equivalent atoms). If I understood correctly, you did not 
have these two problems with your cif file?




*From:* Wien  on behalf of 
Xavier Rocquefelte 

*Sent:* Thursday, June 4, 2020 9:13 AM
*To:* wien@zeus.theochem.tuwien.ac.at
*Subject:* Re: [Wien] Structure optimization of Beta -Sn

Dear Fabien

This is strange... Perhaps a conversion problem. Yes I have used the 
following cif file and cif2struct works for me!


Regards

Xavier



Le 04/06/2020 à 09:10, Tran, Fabien a écrit :


Dear Xavier,

I read your email from yesterday, but I thought that his struct 
filewas correct, because I got the same struct file from this cif file:


http://rruff.geo.arizona.edu/AMS/download.php?id=13353.cif=cif

But you are right; if I visualize it, I can see that it is not 
beta-Sn. Besides, I can not convert cif to struct other cif files 
like the one that you sent. With cif2struct, I get "unknown space 
group name: I41/amds" and no struct file is generated.


Did cif2struct​ work for you?




*From:* Wien  on behalf of 
Xavier Rocquefelte 

*Sent:* Wednesday, June 3, 2020 11:54 PM
*To:* wien@zeus.theochem.tuwien.ac.at
*Subject:* Re: [Wien] Structure optimization of Beta -Sn

Dear Fabien,

I explained to Shamik that the structure he was using was not correct.

I also sent cif and struct file.

See below a proper case.struct file.

Shamik could you please send email only to the wienlist to avoid 
multiple answers from the list and many people trying to help you 
without having all the details?



blebleble
B   LATTICE,NONEQUIV.ATOMS:  1 141 I41/amd
MODE OF CALC=RELA unit=bohr
 11.019938 11.019938  6.011975 90.00 90.00 90.00
ATOM   1: X=0.5000 Y=0.2500 Z=0.1250
  MULT= 2  ISPLIT=-2
   1: X=0. Y=0.2500 Z=0.3750
Sn1    NPT=  781  R0=.1 RMT= 2.5 Z:  50.
LOCAL ROT MATRIX:    1.000 0.000 0.000
 0.000 1.000 0.000
 0.000 0.000 1.000
  16  NUMBER OF SYMMETRY OPERATIONS
 1 0 0 0.
 0 1 0 0.
 0 0 1 0.
   1
-1 0 0 0.5000
 0-1 0 0.
 0 0 1 0.5000
   2
 0-1 0 0.2500
 1 0 0 0.7500
 0 0 1 0.2500
   3
 0 1 0 0.2500
-1 0 0 0.2500
 0 0 1 0.7500
   4
-1 0 0 0.5000
 0 1 0 0.
 0 0-1 0.5000
   5
 1 0 0 0.
 0-1 0 0.
 0 0-1 0.
   6
 0 1 0 0.2500
 1 0 0 0.7500
 0 0-1 0.2500
   7
 0-1 0 0.2500
-1 0 0 0.2500
 0 0-1 0.7500
   8
-1 0 0 0.
 0-1 0 0.
 0 0-1 0.
   9
 1 0 0 0.5000
 0 1 0 0.
 0 0-1 0.5000
  10
 0 1 0 0.7500
-1 0 0 0.2500
 0 0-1 0.7500
  11
 0-1 0 0.7500
 1 0 0 0.7500
 0 0-1 0.2500
  12
 1 0 0 0.5000
 0-1 0 0.
 0 0 1 0.5000
  13
-1 0 0 0.
 0 1 0 0.
 0 0 1 0.
  14
 0-1 0 0.7500
-1 0 0 0.2500
 0 0 1 0.7500
  15
 0 1 0 0.7500
 1 0 0 0.7500
 0 0 1 0.2500
  16





Le 03/06/2020 à 21:38, Tran, Fabien a écrit :


Using a RKmax above 7 should not lead to completely wrong results. 
One important point is how the lattice constants a and c 
were varied. In a meaningful way?




*From:* Wien  on behalf of 
shamik chakrabarti 

*Sent:* Wednesday, June 3, 2020 8:40 PM
*To:* A Mailing list for WIEN2k users
*Subject:* Re: [Wien] Structure optimization of Beta -Sn
Dear Dr. Tran,

                  I have used both plain GGA & nlvdw independently & 
in both cases the results are same. I have used Rmr*Kmax=9 for nlvdw 
& 7 for GGA. Do you think moving to larger Rmt*Kmax may solve the 
problem? I am currently going through the literature you have sent..


with regards,

On Wed, 3 Jun 2020 at 23:53, Tran, Fabien <mailto:fabien.t...@tuwien.ac.at>> wrote:


At first sight you struct file seems ok, but this is difficult
to help you without more details. For instance: Which functional
have you used? Are you keeping the c/a ratio fixed? Have you
looked into the literature:

https://aip.scitation.org/doi/abs/10.1063/1.4948434



*From:* Wien mailto:wien-boun...@zeus.theochem.tuwien.ac.at>> on behalf 

Re: [Wien] Structure optimization of Beta -Sn

2020-06-04 Thread Xavier Rocquefelte
Indeed, I used the one from VESTA! Yesteray night I was working in 
parallel ;)


Here is the file I previously sent with modification by hand ... the cif 
format was not totally supported by cif2struct.


Regards

Xavier
) 2020 by FIZ Karlsruhe - Leibniz Institute for Information Infrastructure.  
All rights reserved.
data_106072-ICSD
_database_code_ICSD 106072
_audit_creation_date 2004-10-01
_audit_update_record 2017-08-01
_chemical_name_common Tin
_chemical_formula_structural Sn
_chemical_formula_sum Sn1
_chemical_name_structure_type Sn(tI4)
_exptl_crystal_density_diffrn 7.29
_diffrn_ambient_temperature 296.
_citation_title

;
X-ray investigation of thermal expansion and atomic thermal vibrations of tin,
indium, and their alloys
;
loop_
_citation_id
_citation_journal_full
_citation_year
_citation_journal_volume
_citation_page_first
_citation_page_last
_citation_journal_id_ASTM
primary 'Physica Status Solidi B: Basic Solid State Physics' 1981 107 245 253
PSSBBD
loop_
_citation_author_citation_id
_citation_author_name
primary 'Wolcyrz, M.'
primary 'Kubiak, R.'
primary 'Maciejewski, S.'
_cell_length_a 5.8308(2)
_cell_length_b 5.8308(2)
_cell_length_c 3.1810(2)
_cell_angle_alpha 90.
_cell_angle_beta 90.
_cell_angle_gamma 90.
_cell_volume 108.15
_cell_formula_units_Z 4
_symmetry_space_group_name_H-M 'I 41/a m d'
_symmetry_Int_Tables_number 141
loop_

loop_
_symmetry_equiv_pos_as_xyz
   'x, y, z'
   '-x+1/2, -y+1/2, z+1/2'
   '-y, x+1/2, z+1/4'
   'y+1/2, -x, z+3/4'
   '-x+1/2, y, -z+3/4'
   'x, -y+1/2, -z+1/4'
   'y+1/2, x+1/2, -z+1/2'
   '-y, -x, -z'
   '-x, -y+1/2, -z+1/4'
   'x+1/2, y, -z+3/4'
   'y, -x, -z'
   '-y+1/2, x+1/2, -z+1/2'
   'x+1/2, -y+1/2, z+1/2'
   '-x, y, z'
   '-y+1/2, -x, z+3/4'
   'y, x+1/2, z+1/4'
   'x+1/2, y+1/2, z+1/2'
   '-x, -y, z'
   '-y+1/2, x, z+3/4'
   'y, -x+1/2, z+1/4'
   '-x, y+1/2, -z+1/4'
   'x+1/2, -y, -z+3/4'
   'y, x, -z'
   '-y+1/2, -x+1/2, -z+1/2'
   '-x+1/2, -y, -z+3/4'
   'x, y+1/2, -z+1/4'
   'y+1/2, -x+1/2, -z+1/2'
   '-y, x, -z'
   'x, -y, z'
   '-x+1/2, y+1/2, z+1/2'
   '-y, -x+1/2, z+1/4'
   'y+1/2, x, z+3/4'

loop_
_atom_type_symbol
_atom_type_oxidation_number
Sn0+ 0
loop_
_atom_site_label
_atom_site_type_symbol
_atom_site_symmetry_multiplicity
_atom_site_Wyckoff_symbol
_atom_site_fract_x
_atom_site_fract_y
_atom_site_fract_z
_atom_site_B_iso_or_equiv
_atom_site_occupancy
Sn1 Sn0+ 4 a 0 0 0 . 1.
loop_
_atom_site_aniso_label
_atom_site_aniso_type_symbol
_atom_site_aniso_U_11
_atom_site_aniso_U_22
_atom_site_aniso_U_33
_atom_site_aniso_U_12
_atom_site_aniso_U_13
_atom_site_aniso_U_23
Sn1 Sn0+ 0.0200(2) 0.0200(2) 0.0241(2) 0. 0. 0.
___
Wien mailing list
Wien@zeus.theochem.tuwien.ac.at
http://zeus.theochem.tuwien.ac.at/mailman/listinfo/wien
SEARCH the MAILING-LIST at:  
http://www.mail-archive.com/wien@zeus.theochem.tuwien.ac.at/index.html


Re: [Wien] Structure optimization of Beta -Sn

2020-06-04 Thread Xavier Rocquefelte

Dear Fabien

This is strange... Perhaps a conversion problem. Yes I have used the 
following cif file and cif2struct works for me!


Regards

Xavier



Le 04/06/2020 à 09:10, Tran, Fabien a écrit :


Dear Xavier,

I read your email from yesterday, but I thought that his struct 
filewas correct, because I got the same struct file from this cif file:


http://rruff.geo.arizona.edu/AMS/download.php?id=13353.cif=cif

But you are right; if I visualize it, I can see that it is not 
beta-Sn.  Besides, I can not convert cif to struct other cif files 
like the one that you sent. With cif2struct, I get "unknown space 
group name: I41/amds" and no struct file is generated.


Did cif2struct​ work for you?




*From:* Wien  on behalf of 
Xavier Rocquefelte 

*Sent:* Wednesday, June 3, 2020 11:54 PM
*To:* wien@zeus.theochem.tuwien.ac.at
*Subject:* Re: [Wien] Structure optimization of Beta -Sn

Dear Fabien,

I explained to Shamik that the structure he was using was not correct.

I also sent cif and struct file.

See below a proper case.struct file.

Shamik could you please send email only to the wienlist to avoid 
multiple answers from the list and many people trying to help you 
without having all the details?



blebleble
B   LATTICE,NONEQUIV.ATOMS:  1 141 I41/amd
MODE OF CALC=RELA unit=bohr
 11.019938 11.019938  6.011975 90.00 90.00 90.00
ATOM   1: X=0.5000 Y=0.2500 Z=0.1250
  MULT= 2  ISPLIT=-2
   1: X=0. Y=0.2500 Z=0.3750
Sn1    NPT=  781  R0=.1 RMT= 2.5 Z:  50.
LOCAL ROT MATRIX:    1.000 0.000 0.000
 0.000 1.000 0.000
 0.000 0.000 1.000
  16  NUMBER OF SYMMETRY OPERATIONS
 1 0 0 0.
 0 1 0 0.
 0 0 1 0.
   1
-1 0 0 0.5000
 0-1 0 0.
 0 0 1 0.5000
   2
 0-1 0 0.2500
 1 0 0 0.7500
 0 0 1 0.2500
   3
 0 1 0 0.2500
-1 0 0 0.2500
 0 0 1 0.7500
   4
-1 0 0 0.5000
 0 1 0 0.
 0 0-1 0.5000
   5
 1 0 0 0.
 0-1 0 0.
 0 0-1 0.
   6
 0 1 0 0.2500
 1 0 0 0.7500
 0 0-1 0.2500
   7
 0-1 0 0.2500
-1 0 0 0.2500
 0 0-1 0.7500
   8
-1 0 0 0.
 0-1 0 0.
 0 0-1 0.
   9
 1 0 0 0.5000
 0 1 0 0.
 0 0-1 0.5000
  10
 0 1 0 0.7500
-1 0 0 0.2500
 0 0-1 0.7500
  11
 0-1 0 0.7500
 1 0 0 0.7500
 0 0-1 0.2500
  12
 1 0 0 0.5000
 0-1 0 0.
 0 0 1 0.5000
  13
-1 0 0 0.
 0 1 0 0.
 0 0 1 0.
  14
 0-1 0 0.7500
-1 0 0 0.2500
 0 0 1 0.7500
  15
 0 1 0 0.7500
 1 0 0 0.7500
 0 0 1 0.2500
  16





Le 03/06/2020 à 21:38, Tran, Fabien a écrit :


Using a RKmax above 7 should not lead to completely wrong results. 
One important point is how the lattice constants a and c were varied. 
In a meaningful way?




*From:* Wien  on behalf of 
shamik chakrabarti 

*Sent:* Wednesday, June 3, 2020 8:40 PM
*To:* A Mailing list for WIEN2k users
*Subject:* Re: [Wien] Structure optimization of Beta -Sn
Dear Dr. Tran,

                  I have used both plain GGA & nlvdw independently & 
in both cases the results are same. I have used Rmr*Kmax=9 for nlvdw 
& 7 for GGA. Do you think moving to larger Rmt*Kmax may solve the 
problem? I am currently going through the literature you have sent..


with regards,

On Wed, 3 Jun 2020 at 23:53, Tran, Fabien <mailto:fabien.t...@tuwien.ac.at>> wrote:


At first sight you struct file seems ok, but this is difficult to
help you without more details. For instance: Which functional
have you used? Are you keeping the c/a ratio fixed? Have you
looked into the literature:

https://aip.scitation.org/doi/abs/10.1063/1.4948434



*From:* Wien mailto:wien-boun...@zeus.theochem.tuwien.ac.at>> on behalf of
shamik chakrabarti mailto:shamik15041...@gmail.com>>
*Sent:* Wednesday, June 3, 2020 8:06 PM
*To:* A Mailing list for WIEN2k users
*Subject:* [Wien] Structure optimization of Beta -Sn
Dear wien2k users,

                                 I am trying to optimize the
structure of Beta - Sn. However, even after 20% increment of the
volume there is no sign of energy minima. I am attaching the
struct file herewith this mail for your consideration.

Looking forward to hearing from you.

with regards,

-- 
Dr. Shamik Chakrabarti

Research Fellow
Department of Physics
Indian Institute of Technology Patna
Bihta-801103
Patna
Bihar, India
___
Wien mailing list
Wien@

Re: [Wien] Structure optimization of Beta -Sn

2020-06-04 Thread Xavier Rocquefelte

Dear Shamik

I will do as I do with my colleagues who are learning how to use WIEN2k ;)

Use the cif file I previously sent. If you directory is named Beta-Sn, 
rename the cif file Beta-Sn.cif.


Then use cif2struct in this way: cif2struct Beta-Sn.cif. It will 
generate Beta-Sn.struct


Then initialize the calculation and read carefully what WIEN2k is saying.

You will see this message when doing sgroup:

>   sgroup    (08:56:30) 0.000u 0.000s 0:00.00 0.0%    0+0k 0+8io 0pf+0w
  Names of point group: -4m2   -4m2   D2d
warning: !!! Struct file is not consistent with space group found.
Number and name of space group: 141 (I 41/a m d) [origin choice 2]
warning: !!! Struct file is not consistent with space group found.
-> check in  TT.outputsgroup  for proper symmetry, compare
   with your struct file and later with  TT.outputs
   sgroup has also produced a new struct file based on your old one.
   If you see warnings above, consider to use the newly generated
   struct file, which you can view (edit) now.
-> continue with symmetry (old case.struct) or use/edit 
TT.struct_sgroup ? (c/e)

e

You see that WIEN2k is not happy with this setting and more specifically 
WIEN2k has only one origin choice for this space group (the second one) 
while the structure I sent you and the one you were using is based on 
the origin choice 1. But WIEN2k helps you and proposes to generate the 
structure file with the origin choice 2.


Thus you must use the new file.

e
-> Do you want to use the new struct file  ? (y/n)
y
 next is nn
>   nn    (08:57:51)  specify nn-bondlength factor: (usually=2) [and 
optionally dlimit, dstmax (about

  1.d-5, 20)]
2
 DSTMAX:   22.004768372
 iix,iiy,iiz   5   5   5 55.09969
   55.09969    30.059875000
 NAMED ATOM: Sn1   Z changed to IATNR+999 to determine equivalency

    ATOM  1  Sn1    ATOM  1  Sn1
 RMT(  1)=2.5 AND RMT(  1)=2.5
 SUMS TO 5.0  LT.  NN-DIST= 5.71128
NN ENDS
0.004u 0.000s 0:01.51 0.0%    0+0k 0+32io 0pf+0w
-> check in  TT.outputnn  for overlapping spheres,
   coordination and nearest neighbor distances
-> continue with sgroup or edit the TT.struct file (c/e)
c
 next is sgroup
>   sgroup    (08:58:02) 0.000u 0.000s 0:00.00 0.0%    0+0k 0+8io 0pf+0w
  Names of point group: -4m2   -4m2   D2d
Number and name of space group: 141 (I 41/a m d) [origin choice 2]
-> check in  TT.outputsgroup  for proper symmetry, compare
   with your struct file and later with  TT.outputs
   sgroup has also produced a new struct file based on your old one.
   If you see warnings above, consider to use the newly generated
   struct file, which you can view (edit) now.
-> continue with symmetry (old case.struct) or use/edit 
TT.struct_sgroup ? (c/e)

c

And as you can see you have no more warnings.

Best regards,

Xavier

___
Wien mailing list
Wien@zeus.theochem.tuwien.ac.at
http://zeus.theochem.tuwien.ac.at/mailman/listinfo/wien
SEARCH the MAILING-LIST at:  
http://www.mail-archive.com/wien@zeus.theochem.tuwien.ac.at/index.html


Re: [Wien] Structure optimization of Beta -Sn

2020-06-03 Thread Xavier Rocquefelte
Here is a cif file. You can see that Sn is on a Wyckoff position 4a, 
which means that you have 4 equivalent positions in the space group I 
41/a m d.


In WIEN2k, it leads to a case.struct file having only 2 equivalent 
positions, because only the positions in the primitive cell are shown in 
the case.struct file and here the cell is body-centered (I).



The fact that you have 4 equivalent position in the case.struct file you 
sent to the list is clearly an indication that the structure is not 
correct.


In addition, when starting a calculation you should first check the bond 
distances from the litterature, which is about 3.02 angtström for Sn-Sn. 
More precisely one tin is surrounded by 4 tin atoms with the same Sn-Sn 
distance.



In the structure you sent to the list the Sn-Sn distances were 2.2 
angström!!! It is too small for such an element. It is why the geometry 
optimization goes wrong. Thus, the first thing to due before doing 
"run_lapw" is to check your data. To do so, you must check carefully:



- case.outputnn --> bond distances and coordination

- case.outputs  --> point group for each inequivalent atoms

...


Regards

Xavier






Le 03/06/2020 à 23:54, Xavier Rocquefelte a écrit :


Dear Fabien,

I explained to Shamik that the structure he was using was not correct.

I also sent cif and struct file.

See below a proper case.struct file.

Shamik could you please send email only to the wienlist to avoid 
multiple answers from the list and many people trying to help you 
without having all the details?



blebleble
B   LATTICE,NONEQUIV.ATOMS:  1 141 I41/amd
MODE OF CALC=RELA unit=bohr
 11.019938 11.019938  6.011975 90.00 90.00 90.00
ATOM   1: X=0.5000 Y=0.2500 Z=0.1250
  MULT= 2  ISPLIT=-2
   1: X=0. Y=0.2500 Z=0.3750
Sn1    NPT=  781  R0=.1 RMT= 2.5 Z:  50.
LOCAL ROT MATRIX:    1.000 0.000 0.000
 0.000 1.000 0.000
 0.000 0.000 1.000
  16  NUMBER OF SYMMETRY OPERATIONS
 1 0 0 0.
 0 1 0 0.
 0 0 1 0.
   1
-1 0 0 0.5000
 0-1 0 0.
 0 0 1 0.5000
   2
 0-1 0 0.2500
 1 0 0 0.7500
 0 0 1 0.2500
   3
 0 1 0 0.2500
-1 0 0 0.2500
 0 0 1 0.7500
   4
-1 0 0 0.5000
 0 1 0 0.
 0 0-1 0.5000
   5
 1 0 0 0.
 0-1 0 0.
 0 0-1 0.
   6
 0 1 0 0.2500
 1 0 0 0.7500
 0 0-1 0.2500
   7
 0-1 0 0.2500
-1 0 0 0.2500
 0 0-1 0.7500
   8
-1 0 0 0.
 0-1 0 0.
 0 0-1 0.
   9
 1 0 0 0.5000
 0 1 0 0.
 0 0-1 0.5000
  10
 0 1 0 0.7500
-1 0 0 0.2500
 0 0-1 0.7500
  11
 0-1 0 0.7500
 1 0 0 0.7500
 0 0-1 0.2500
  12
 1 0 0 0.5000
 0-1 0 0.
 0 0 1 0.5000
  13
-1 0 0 0.
 0 1 0 0.
 0 0 1 0.
  14
 0-1 0 0.7500
-1 0 0 0.2500
 0 0 1 0.7500
  15
 0 1 0 0.7500
 1 0 0 0.7500
 0 0 1 0.2500
  16





Le 03/06/2020 à 21:38, Tran, Fabien a écrit :


Using a RKmax above 7 should not lead to completely wrong results. 
One important point is how the lattice constants a and c were varied. 
In a meaningful way?




*From:* Wien  on behalf of 
shamik chakrabarti 

*Sent:* Wednesday, June 3, 2020 8:40 PM
*To:* A Mailing list for WIEN2k users
*Subject:* Re: [Wien] Structure optimization of Beta -Sn
Dear Dr. Tran,

                  I have used both plain GGA & nlvdw independently & 
in both cases the results are same. I have used Rmr*Kmax=9 for nlvdw 
& 7 for GGA. Do you think moving to larger Rmt*Kmax may solve the 
problem? I am currently going through the literature you have sent..


with regards,

On Wed, 3 Jun 2020 at 23:53, Tran, Fabien <mailto:fabien.t...@tuwien.ac.at>> wrote:


At first sight you struct file seems ok, but this is difficult to
help you without more details. For instance: Which functional
have you used? Are you keeping the c/a ratio fixed? Have you
looked into the literature:

https://aip.scitation.org/doi/abs/10.1063/1.4948434



*From:* Wien mailto:wien-boun...@zeus.theochem.tuwien.ac.at>> on behalf of
shamik chakrabarti mailto:shamik15041...@gmail.com>>
*Sent:* Wednesday, June 3, 2020 8:06 PM
*To:* A Mailing list for WIEN2k users
*Subject:* [Wien] Structure optimization of Beta -Sn
Dear wien2k users,

                                 I am trying to optimize the
structure of Beta - Sn. However, even after 20% increment of the
volume there is no sign of energy minima. I am attaching the
struct file herewith this mail for your consideration.

Looking forward to hearing from you.

with regards,

Re: [Wien] Structure optimization of Beta -Sn

2020-06-03 Thread Xavier Rocquefelte

Dear Fabien,

I explained to Shamik that the structure he was using was not correct.

I also sent cif and struct file.

See below a proper case.struct file.

Shamik could you please send email only to the wienlist to avoid 
multiple answers from the list and many people trying to help you 
without having all the details?



blebleble
B   LATTICE,NONEQUIV.ATOMS:  1 141 I41/amd
MODE OF CALC=RELA unit=bohr
 11.019938 11.019938  6.011975 90.00 90.00 90.00
ATOM   1: X=0.5000 Y=0.2500 Z=0.1250
  MULT= 2  ISPLIT=-2
   1: X=0. Y=0.2500 Z=0.3750
Sn1    NPT=  781  R0=.1 RMT= 2.5 Z:  50.
LOCAL ROT MATRIX:    1.000 0.000 0.000
 0.000 1.000 0.000
 0.000 0.000 1.000
  16  NUMBER OF SYMMETRY OPERATIONS
 1 0 0 0.
 0 1 0 0.
 0 0 1 0.
   1
-1 0 0 0.5000
 0-1 0 0.
 0 0 1 0.5000
   2
 0-1 0 0.2500
 1 0 0 0.7500
 0 0 1 0.2500
   3
 0 1 0 0.2500
-1 0 0 0.2500
 0 0 1 0.7500
   4
-1 0 0 0.5000
 0 1 0 0.
 0 0-1 0.5000
   5
 1 0 0 0.
 0-1 0 0.
 0 0-1 0.
   6
 0 1 0 0.2500
 1 0 0 0.7500
 0 0-1 0.2500
   7
 0-1 0 0.2500
-1 0 0 0.2500
 0 0-1 0.7500
   8
-1 0 0 0.
 0-1 0 0.
 0 0-1 0.
   9
 1 0 0 0.5000
 0 1 0 0.
 0 0-1 0.5000
  10
 0 1 0 0.7500
-1 0 0 0.2500
 0 0-1 0.7500
  11
 0-1 0 0.7500
 1 0 0 0.7500
 0 0-1 0.2500
  12
 1 0 0 0.5000
 0-1 0 0.
 0 0 1 0.5000
  13
-1 0 0 0.
 0 1 0 0.
 0 0 1 0.
  14
 0-1 0 0.7500
-1 0 0 0.2500
 0 0 1 0.7500
  15
 0 1 0 0.7500
 1 0 0 0.7500
 0 0 1 0.2500
  16





Le 03/06/2020 à 21:38, Tran, Fabien a écrit :


Using a RKmax above 7 should not lead to completely wrong results. 
One important point is how the lattice constants a and c were varied. 
In a meaningful way?




*From:* Wien  on behalf of 
shamik chakrabarti 

*Sent:* Wednesday, June 3, 2020 8:40 PM
*To:* A Mailing list for WIEN2k users
*Subject:* Re: [Wien] Structure optimization of Beta -Sn
Dear Dr. Tran,

                  I have used both plain GGA & nlvdw independently & 
in both cases the results are same. I have used Rmr*Kmax=9 for nlvdw & 
7 for GGA. Do you think moving to larger Rmt*Kmax may solve the 
problem? I am currently going through the literature you have sent..


with regards,

On Wed, 3 Jun 2020 at 23:53, Tran, Fabien <mailto:fabien.t...@tuwien.ac.at>> wrote:


At first sight you struct file seems ok, but this is difficult to
help you without more details. For instance: Which functional have
you used? Are you keeping the c/a ratio fixed? Have you looked
into the literature:

https://aip.scitation.org/doi/abs/10.1063/1.4948434



*From:* Wien mailto:wien-boun...@zeus.theochem.tuwien.ac.at>> on behalf of
shamik chakrabarti mailto:shamik15041...@gmail.com>>
*Sent:* Wednesday, June 3, 2020 8:06 PM
*To:* A Mailing list for WIEN2k users
*Subject:* [Wien] Structure optimization of Beta -Sn
Dear wien2k users,

                                 I am trying to optimize the
structure of Beta - Sn. However, even after 20% increment of the
volume there is no sign of energy minima. I am attaching the
struct file herewith this mail for your consideration.

Looking forward to hearing from you.

with regards,

-- 
Dr. Shamik Chakrabarti

Research Fellow
Department of Physics
Indian Institute of Technology Patna
Bihta-801103
Patna
Bihar, India
___
Wien mailing list
Wien@zeus.theochem.tuwien.ac.at
<mailto:Wien@zeus.theochem.tuwien.ac.at>
http://zeus.theochem.tuwien.ac.at/mailman/listinfo/wien
SEARCH the MAILING-LIST at:
http://www.mail-archive.com/wien@zeus.theochem.tuwien.ac.at/index.html



--
Dr. Shamik Chakrabarti
Research Fellow
Department of Physics
Indian Institute of Technology Patna
Bihta-801103
Patna
Bihar, India

___
Wien mailing list
Wien@zeus.theochem.tuwien.ac.at
http://zeus.theochem.tuwien.ac.at/mailman/listinfo/wien
SEARCH the MAILING-LIST at:  
http://www.mail-archive.com/wien@zeus.theochem.tuwien.ac.at/index.html


--

Institut des Sciences Chimiques de Rennes (ISCR)
Univ Rennes - CNRS - UMR6226, France
https://iscr.univ-rennes1.fr/cti/people/permanent-staff/rocquefelte-xavier

ICAMM2019 : VASP Workshop and International Materials Modelling Conf
June 26-July 3 2019, Rennes France
icamm2019.sciencesconf.or

Re: [Wien] Structure optimization of Beta -Sn

2020-06-03 Thread Xavier Rocquefelte

Your structure file seems to be not correct!

You should have only one inequivalent position and the space group is I 
41/a m d.


Here is the structure:

http://som.web.cmu.edu/structures/S018-beta-Sn.html


Le 03/06/2020 à 20:40, shamik chakrabarti a écrit :

Dear Dr. Tran,

                  I have used both plain GGA & nlvdw independently & 
in both cases the results are same. I have used Rmr*Kmax=9 for nlvdw & 
7 for GGA. Do you think moving to larger Rmt*Kmax may solve the 
problem? I am currently going through the literature you have sent..


with regards,

On Wed, 3 Jun 2020 at 23:53, Tran, Fabien <mailto:fabien.t...@tuwien.ac.at>> wrote:


At first sight you struct file seems ok, but this is difficult to
help you without more details. For instance: Which functional have
you used? Are you keeping the c/a ratio fixed? Have you looked
into the literature:

https://aip.scitation.org/doi/abs/10.1063/1.4948434



*From:* Wien mailto:wien-boun...@zeus.theochem.tuwien.ac.at>> on behalf of
shamik chakrabarti mailto:shamik15041...@gmail.com>>
*Sent:* Wednesday, June 3, 2020 8:06 PM
*To:* A Mailing list for WIEN2k users
*Subject:* [Wien] Structure optimization of Beta -Sn
Dear wien2k users,

                                 I am trying to optimize the
structure of Beta - Sn. However, even after 20% increment of the
volume there is no sign of energy minima. I am attaching the
struct file herewith this mail for your consideration.

Looking forward to hearing from you.

with regards,

-- 
Dr. Shamik Chakrabarti

Research Fellow
Department of Physics
Indian Institute of Technology Patna
Bihta-801103
Patna
Bihar, India
___
Wien mailing list
Wien@zeus.theochem.tuwien.ac.at
<mailto:Wien@zeus.theochem.tuwien.ac.at>
http://zeus.theochem.tuwien.ac.at/mailman/listinfo/wien
SEARCH the MAILING-LIST at:
http://www.mail-archive.com/wien@zeus.theochem.tuwien.ac.at/index.html



--
Dr. Shamik Chakrabarti
Research Fellow
Department of Physics
Indian Institute of Technology Patna
Bihta-801103
Patna
Bihar, India

___
Wien mailing list
Wien@zeus.theochem.tuwien.ac.at
http://zeus.theochem.tuwien.ac.at/mailman/listinfo/wien
SEARCH the MAILING-LIST at:  
http://www.mail-archive.com/wien@zeus.theochem.tuwien.ac.at/index.html


--

Institut des Sciences Chimiques de Rennes (ISCR)
Univ Rennes - CNRS - UMR6226, France
https://iscr.univ-rennes1.fr/cti/people/permanent-staff/rocquefelte-xavier

ICAMM2019 : VASP Workshop and International Materials Modelling Conf
June 26-July 3 2019, Rennes France
icamm2019.sciencesconf.org  : Registration opening soon, see you there!

___
Wien mailing list
Wien@zeus.theochem.tuwien.ac.at
http://zeus.theochem.tuwien.ac.at/mailman/listinfo/wien
SEARCH the MAILING-LIST at:  
http://www.mail-archive.com/wien@zeus.theochem.tuwien.ac.at/index.html


Re: [Wien] Aligning magnetization direction along high symmetry crystallographic direction

2020-05-20 Thread Xavier Rocquefelte
And another comment. Applying the magnetization along a high symmetry 
direction does not mean that you will apply SOC in the direction for 
which the effect will be maximum... It is why, people study the 
magnetocrystalline anisotropy energy (MAE).


Best regards

Xavier

Le 20/05/2020 à 08:07, Peter Blaha a écrit :

I think nobody really understands your question.

Obviously for a hexagonal system the c-axis is a very high symmetry 
direction. This corresponds to  0 0 1


Of course, the a (or b) direction is also a special direction, but it 
will break hexagonal symmetry as the a and b directions will no longer 
be equivalent. So when you choose 1 0 0, in many cases the symmetry 
will be automatically reduced by symmetso.



Am 20.05.2020 um 05:30 schrieb Nileema Sharma:

Dear WIEN2k users and developers!!
I am working on a hexagonal system, I would like to know in which 
direction(s) should I apply SOC so that it would represent high 
symmetry crystallographic direction, for the calculation of the 
energy in that particular direction?

Best regards.
Thank you!!
Nileema Sharma

___
Wien mailing list
Wien@zeus.theochem.tuwien.ac.at
http://zeus.theochem.tuwien.ac.at/mailman/listinfo/wien
SEARCH the MAILING-LIST at: 
http://www.mail-archive.com/wien@zeus.theochem.tuwien.ac.at/index.html





--

Institut des Sciences Chimiques de Rennes (ISCR)
Univ Rennes - CNRS - UMR6226, France
https://iscr.univ-rennes1.fr/cti/people/permanent-staff/rocquefelte-xavier

ICAMM2019 : VASP Workshop and International Materials Modelling Conf
June 26-July 3 2019, Rennes France
icamm2019.sciencesconf.org  : Registration opening soon, see you there!

___
Wien mailing list
Wien@zeus.theochem.tuwien.ac.at
http://zeus.theochem.tuwien.ac.at/mailman/listinfo/wien
SEARCH the MAILING-LIST at:  
http://www.mail-archive.com/wien@zeus.theochem.tuwien.ac.at/index.html


Re: [Wien] MBJ fails to produce gap unto 0.6 for VO2 M1 phase

2020-05-12 Thread Xavier Rocquefelte
Just one comment about this paper. They compare total energies obtained 
from mBJLDA.


However, mBJ is not self-consistent with respect to the total energy. 
mBJ is a semilocal potential.


Thus Tables I and II contain errors.

Best wishes

Xavier

Le 13/05/2020 à 05:59, Wasim Raja Mondal a écrit :

Dear All,
            Sorry for my email. MBJ works for this material also.
Please see the reference if anybody interested: PHYSICAL REVIEW B 86, 
075149 (2012)


Thanks
Wasim

On Tue, May 12, 2020 at 2:02 AM Xavier Rocquefelte 
<mailto:xavier.rocquefe...@univ-rennes1.fr>> wrote:


Dear Wasim,

In VO2, vanadium is 4+ and thus we expect one electron in V(3d)
states, thus a gap based on d-d transitions.

MBJ alone is not expected to open the gap. You will need to put a
Hubbard term at least.

I recommend you to read the following article which deals with
DFT+DMFT and M1 phase of VO2.


  Phys. Rev. Lett. *117*, 056402 (2016)

Best regards

Xavier

Le 12/05/2020 à 03:09, Wasim Raja Mondal a écrit :

Dear Experts,
                     I am doing some calculations for VO2 M1
phase. To get the correct band gap value, I applied MBJ. But, I
am getting zero gap. To get the gap, I increased the c value.
with such large c value, there is no sign of convergence of my
calculation.

I appreciate if experts have any comments and suggestions.

Regards
Wasim

___
Wien mailing list
Wien@zeus.theochem.tuwien.ac.at  <mailto:Wien@zeus.theochem.tuwien.ac.at>
http://zeus.theochem.tuwien.ac.at/mailman/listinfo/wien
SEARCH the MAILING-LIST 
at:http://www.mail-archive.com/wien@zeus.theochem.tuwien.ac.at/index.html


-- 


Institut des Sciences Chimiques de Rennes (ISCR)
Univ Rennes - CNRS - UMR6226, France
https://iscr.univ-rennes1.fr/cti/people/permanent-staff/rocquefelte-xavier

ICAMM2019 : VASP Workshop and International Materials Modelling Conf
June 26-July 3 2019, Rennes France
icamm2019.sciencesconf.org  <http://icamm2019.sciencesconf.org>   : 
Registration opening soon, see you there!

___
Wien mailing list
Wien@zeus.theochem.tuwien.ac.at
<mailto:Wien@zeus.theochem.tuwien.ac.at>
http://zeus.theochem.tuwien.ac.at/mailman/listinfo/wien
SEARCH the MAILING-LIST at:
http://www.mail-archive.com/wien@zeus.theochem.tuwien.ac.at/index.html


___
Wien mailing list
Wien@zeus.theochem.tuwien.ac.at
http://zeus.theochem.tuwien.ac.at/mailman/listinfo/wien
SEARCH the MAILING-LIST at:  
http://www.mail-archive.com/wien@zeus.theochem.tuwien.ac.at/index.html


--

Institut des Sciences Chimiques de Rennes (ISCR)
Univ Rennes - CNRS - UMR6226, France
https://iscr.univ-rennes1.fr/cti/people/permanent-staff/rocquefelte-xavier

ICAMM2019 : VASP Workshop and International Materials Modelling Conf
June 26-July 3 2019, Rennes France
icamm2019.sciencesconf.org  : Registration opening soon, see you there!

___
Wien mailing list
Wien@zeus.theochem.tuwien.ac.at
http://zeus.theochem.tuwien.ac.at/mailman/listinfo/wien
SEARCH the MAILING-LIST at:  
http://www.mail-archive.com/wien@zeus.theochem.tuwien.ac.at/index.html


Re: [Wien] MBJ fails to produce gap unto 0.6 for VO2 M1 phase

2020-05-12 Thread Xavier Rocquefelte

Dear Wasim,

In VO2, vanadium is 4+ and thus we expect one electron in V(3d) states, 
thus a gap based on d-d transitions.


MBJ alone is not expected to open the gap. You will need to put a 
Hubbard term at least.


I recommend you to read the following article which deals with DFT+DMFT 
and M1 phase of VO2.



 Phys. Rev. Lett. *117*, 056402 (2016)

Best regards

Xavier

Le 12/05/2020 à 03:09, Wasim Raja Mondal a écrit :

Dear Experts,
                     I am doing some calculations for VO2 M1 phase. To 
get the correct band gap value, I applied MBJ. But, I am getting zero 
gap. To get the gap, I increased the c value. with such large c value, 
there is no sign of convergence of my calculation.


I appreciate if experts have any comments and suggestions.

Regards
Wasim

___
Wien mailing list
Wien@zeus.theochem.tuwien.ac.at
http://zeus.theochem.tuwien.ac.at/mailman/listinfo/wien
SEARCH the MAILING-LIST at:  
http://www.mail-archive.com/wien@zeus.theochem.tuwien.ac.at/index.html


--

Institut des Sciences Chimiques de Rennes (ISCR)
Univ Rennes - CNRS - UMR6226, France
https://iscr.univ-rennes1.fr/cti/people/permanent-staff/rocquefelte-xavier

ICAMM2019 : VASP Workshop and International Materials Modelling Conf
June 26-July 3 2019, Rennes France
icamm2019.sciencesconf.org  : Registration opening soon, see you there!

___
Wien mailing list
Wien@zeus.theochem.tuwien.ac.at
http://zeus.theochem.tuwien.ac.at/mailman/listinfo/wien
SEARCH the MAILING-LIST at:  
http://www.mail-archive.com/wien@zeus.theochem.tuwien.ac.at/index.html


Re: [Wien] LDA vs GGA

2020-04-10 Thread Xavier Rocquefelte

I usually apply the following recipe Ueff(LDA) = Ueff(GGA) + 2 in eV.

You could test many Ueff values and check how the magnetic moment evolves.

Here are papers which could help. In the first one they did LDA+U+SOC 
with U = 0, 1 and 2 eV.


https://www.nature.com/articles/srep30309

https://arxiv.org/ftp/arxiv/papers/1511/1511.06979.pdf

Cheers

Xavier

Le 10/04/2020 à 12:13, Amit Chauhan ph17d008 a écrit :
Yes, I have done LDA/GGA + U + SOC. And both are predicting the ground 
state properly. But my query is related to the magnetic moment. Why it 
is magnetic in GGA but non-magnetic in LDA for U values I have suggested.


शुक्र, 10 अप्रैल 2020, 15:37 को Xavier Rocquefelte 
<mailto:xavier.rocquefe...@univ-rennes1.fr>> ने लिखा:


One more comment, LDA+U (+SOC) will be better than everything in a
DFT level, because of iridium.


Le 10/04/2020 à 12:02, Xavier Rocquefelte a écrit :


I really recommend to look carefully at the litterature of this
system.

Using mbj or hybrid will work but for what?

In addition, the states at the Fermi to be properly described
need something you will not have in your functional.

Cheers

Xavier


Le 10/04/2020 à 12:00, Wasim Raja Mondal a écrit :

You can use mbj and hybrid functional. It is straight forward to
include spin-orbit in such calculations.

Of course, DMFT will be the best solution. For that, you can use
packages like TRIQS, Kerstn's full LDA+DMFT code is available in
his website.

https://triqs.github.io/triqs/latest/

http://hauleweb.rutgers.edu/tutorials/

On Fri, Apr 10, 2020 at 5:46 AM Xavier Rocquefelte
mailto:xavier.rocquefe...@univ-rennes1.fr>> wrote:

For such a system DFT is not sufficient. You must use DMFT.

In addition, you should include spin-orbit coupling.

It is thus a very difficult situation because DMFT+SOC is
not trivial at all.

Cheers

Xavier


Le 10/04/2020 à 11:39, Amit Chauhan ph17d008 a écrit :

Dear All:

I am working on a strongly spin-orbital coupled d^5 system,
SrIrO3 (orthorhombic). It's a Dirac semimetal with weakly
correlated (U=0.3-0.4 eV) non-magnetic ground state. With
GGA+U, for U=1eV, the system is showing a magnetic metal
phase but with LDA it is showing complely non-magnetic
state. I also checked for U=2 eV, but still GGA gave a
magnetic solution but LDA remains non-magnetic. I have also
used a very fine k-mesh (16x16x12) but still LDA remains
non-magnetic. I also checked it with PAW pseudopotential
method as implemented in VASP, for both GGA and LDA, and
results were consistent with the full potential GGA. Why
full potential LDA is not showing the magnetic state??. I
have properly checked the inorb and indmc files and the
parameters are fine.

Regards,

Amit Chauhan
Research Scholar
IIT Madras


___
Wien mailing list
Wien@zeus.theochem.tuwien.ac.at  
<mailto:Wien@zeus.theochem.tuwien.ac.at>
http://zeus.theochem.tuwien.ac.at/mailman/listinfo/wien
SEARCH the MAILING-LIST 
at:http://www.mail-archive.com/wien@zeus.theochem.tuwien.ac.at/index.html


-- 


Institut des Sciences Chimiques de Rennes (ISCR)
Univ Rennes - CNRS - UMR6226, France

https://iscr.univ-rennes1.fr/cti/people/permanent-staff/rocquefelte-xavier

ICAMM2019 : VASP Workshop and International Materials Modelling Conf
June 26-July 3 2019, Rennes France
icamm2019.sciencesconf.org  <http://icamm2019.sciencesconf.org>   : 
Registration opening soon, see you there!

___
Wien mailing list
Wien@zeus.theochem.tuwien.ac.at
<mailto:Wien@zeus.theochem.tuwien.ac.at>
http://zeus.theochem.tuwien.ac.at/mailman/listinfo/wien
SEARCH the MAILING-LIST at:
http://www.mail-archive.com/wien@zeus.theochem.tuwien.ac.at/index.html


___
Wien mailing list
Wien@zeus.theochem.tuwien.ac.at  <mailto:Wien@zeus.theochem.tuwien.ac.at>
http://zeus.theochem.tuwien.ac.at/mailman/listinfo/wien
SEARCH the MAILING-LIST 
at:http://www.mail-archive.com/wien@zeus.theochem.tuwien.ac.at/index.html
-- 


Institut des Sciences Chimiques de Rennes (ISCR)
Univ Rennes - CNRS - UMR6226, France
https://iscr.univ-rennes1.fr/cti/people/permanent-staff/rocquefelte-xavier

ICAMM2019 : VASP Workshop and International Materials Modelling Conf
June 26-July 3 2019, Rennes France
icamm2019.sciencesconf.org  <http://icamm2019.sciencesconf.or

Re: [Wien] LDA vs GGA

2020-04-10 Thread Xavier Rocquefelte
One more comment, LDA+U (+SOC) will be better than everything in a DFT 
level, because of iridium.



Le 10/04/2020 à 12:02, Xavier Rocquefelte a écrit :


I really recommend to look carefully at the litterature of this system.

Using mbj or hybrid will work but for what?

In addition, the states at the Fermi to be properly described need 
something you will not have in your functional.


Cheers

Xavier


Le 10/04/2020 à 12:00, Wasim Raja Mondal a écrit :
You can use mbj and hybrid functional. It is straight forward to 
include spin-orbit in such calculations.


Of course, DMFT will be the best solution. For that, you can use 
packages like TRIQS, Kerstn's full LDA+DMFT code is available in his 
website.


https://triqs.github.io/triqs/latest/

http://hauleweb.rutgers.edu/tutorials/

On Fri, Apr 10, 2020 at 5:46 AM Xavier Rocquefelte 
<mailto:xavier.rocquefe...@univ-rennes1.fr>> wrote:


For such a system DFT is not sufficient. You must use DMFT.

In addition, you should include spin-orbit coupling.

It is thus a very difficult situation because DMFT+SOC is not
trivial at all.

Cheers

Xavier


Le 10/04/2020 à 11:39, Amit Chauhan ph17d008 a écrit :

Dear All:

I am working on a strongly spin-orbital coupled d^5 system,
SrIrO3 (orthorhombic). It's a Dirac semimetal with weakly
correlated (U=0.3-0.4 eV) non-magnetic ground state. With GGA+U,
for U=1eV, the system is showing a magnetic metal phase but with
LDA it is showing complely non-magnetic state. I also checked
for U=2 eV, but still GGA gave a magnetic solution but LDA
remains non-magnetic. I have also used a very fine k-mesh
(16x16x12) but still LDA remains non-magnetic. I also checked it
with PAW pseudopotential method as implemented in VASP, for both
GGA and LDA, and results were consistent with the full potential
GGA. Why full potential LDA is not showing the magnetic state??.
I have properly checked the inorb and indmc files and the
parameters are fine.

Regards,

Amit Chauhan
Research Scholar
IIT Madras


___
Wien mailing list
Wien@zeus.theochem.tuwien.ac.at  <mailto:Wien@zeus.theochem.tuwien.ac.at>
http://zeus.theochem.tuwien.ac.at/mailman/listinfo/wien
SEARCH the MAILING-LIST 
at:http://www.mail-archive.com/wien@zeus.theochem.tuwien.ac.at/index.html


-- 


Institut des Sciences Chimiques de Rennes (ISCR)
Univ Rennes - CNRS - UMR6226, France
https://iscr.univ-rennes1.fr/cti/people/permanent-staff/rocquefelte-xavier

ICAMM2019 : VASP Workshop and International Materials Modelling Conf
June 26-July 3 2019, Rennes France
icamm2019.sciencesconf.org  <http://icamm2019.sciencesconf.org>   : 
Registration opening soon, see you there!

___
Wien mailing list
Wien@zeus.theochem.tuwien.ac.at
<mailto:Wien@zeus.theochem.tuwien.ac.at>
http://zeus.theochem.tuwien.ac.at/mailman/listinfo/wien
SEARCH the MAILING-LIST at:
http://www.mail-archive.com/wien@zeus.theochem.tuwien.ac.at/index.html


___
Wien mailing list
Wien@zeus.theochem.tuwien.ac.at
http://zeus.theochem.tuwien.ac.at/mailman/listinfo/wien
SEARCH the MAILING-LIST 
at:http://www.mail-archive.com/wien@zeus.theochem.tuwien.ac.at/index.html

--

Institut des Sciences Chimiques de Rennes (ISCR)
Univ Rennes - CNRS - UMR6226, France
https://iscr.univ-rennes1.fr/cti/people/permanent-staff/rocquefelte-xavier

ICAMM2019 : VASP Workshop and International Materials Modelling Conf
June 26-July 3 2019, Rennes France
icamm2019.sciencesconf.org  : Registration opening soon, see you there!

___
Wien mailing list
Wien@zeus.theochem.tuwien.ac.at
http://zeus.theochem.tuwien.ac.at/mailman/listinfo/wien
SEARCH the MAILING-LIST at:  
http://www.mail-archive.com/wien@zeus.theochem.tuwien.ac.at/index.html


--

Institut des Sciences Chimiques de Rennes (ISCR)
Univ Rennes - CNRS - UMR6226, France
https://iscr.univ-rennes1.fr/cti/people/permanent-staff/rocquefelte-xavier

ICAMM2019 : VASP Workshop and International Materials Modelling Conf
June 26-July 3 2019, Rennes France
icamm2019.sciencesconf.org  : Registration opening soon, see you there!

___
Wien mailing list
Wien@zeus.theochem.tuwien.ac.at
http://zeus.theochem.tuwien.ac.at/mailman/listinfo/wien
SEARCH the MAILING-LIST at:  
http://www.mail-archive.com/wien@zeus.theochem.tuwien.ac.at/index.html


Re: [Wien] LDA vs GGA

2020-04-10 Thread Xavier Rocquefelte

I really recommend to look carefully at the litterature of this system.

Using mbj or hybrid will work but for what?

In addition, the states at the Fermi to be properly described need 
something you will not have in your functional.


Cheers

Xavier


Le 10/04/2020 à 12:00, Wasim Raja Mondal a écrit :
You can use mbj and hybrid functional. It is straight forward to 
include spin-orbit in such calculations.


Of course, DMFT will be the best solution. For that, you can use 
packages like TRIQS, Kerstn's full LDA+DMFT code is available in his 
website.


https://triqs.github.io/triqs/latest/

http://hauleweb.rutgers.edu/tutorials/

On Fri, Apr 10, 2020 at 5:46 AM Xavier Rocquefelte 
<mailto:xavier.rocquefe...@univ-rennes1.fr>> wrote:


For such a system DFT is not sufficient. You must use DMFT.

In addition, you should include spin-orbit coupling.

It is thus a very difficult situation because DMFT+SOC is not
trivial at all.

Cheers

Xavier


Le 10/04/2020 à 11:39, Amit Chauhan ph17d008 a écrit :

Dear All:

I am working on a strongly spin-orbital coupled d^5 system,
SrIrO3 (orthorhombic). It's a Dirac semimetal with weakly
correlated (U=0.3-0.4 eV) non-magnetic ground state. With GGA+U,
for U=1eV, the system is showing a magnetic metal phase but with
LDA it is showing complely non-magnetic state. I also checked for
U=2 eV, but still GGA gave a magnetic solution but LDA remains
non-magnetic. I have also used a very fine k-mesh (16x16x12) but
still LDA remains non-magnetic. I also checked it with PAW
pseudopotential method as implemented in VASP, for both GGA and
LDA, and results were consistent with the full potential GGA. Why
full potential LDA is not showing the magnetic state??. I have
properly checked the inorb and indmc files and the parameters are
fine.

Regards,

Amit Chauhan
Research Scholar
IIT Madras


___
Wien mailing list
Wien@zeus.theochem.tuwien.ac.at  <mailto:Wien@zeus.theochem.tuwien.ac.at>
http://zeus.theochem.tuwien.ac.at/mailman/listinfo/wien
SEARCH the MAILING-LIST 
at:http://www.mail-archive.com/wien@zeus.theochem.tuwien.ac.at/index.html


-- 


Institut des Sciences Chimiques de Rennes (ISCR)
Univ Rennes - CNRS - UMR6226, France
https://iscr.univ-rennes1.fr/cti/people/permanent-staff/rocquefelte-xavier

ICAMM2019 : VASP Workshop and International Materials Modelling Conf
June 26-July 3 2019, Rennes France
icamm2019.sciencesconf.org  <http://icamm2019.sciencesconf.org>   : 
Registration opening soon, see you there!

___
Wien mailing list
Wien@zeus.theochem.tuwien.ac.at
<mailto:Wien@zeus.theochem.tuwien.ac.at>
http://zeus.theochem.tuwien.ac.at/mailman/listinfo/wien
SEARCH the MAILING-LIST at:
http://www.mail-archive.com/wien@zeus.theochem.tuwien.ac.at/index.html


___
Wien mailing list
Wien@zeus.theochem.tuwien.ac.at
http://zeus.theochem.tuwien.ac.at/mailman/listinfo/wien
SEARCH the MAILING-LIST at:  
http://www.mail-archive.com/wien@zeus.theochem.tuwien.ac.at/index.html


--

Institut des Sciences Chimiques de Rennes (ISCR)
Univ Rennes - CNRS - UMR6226, France
https://iscr.univ-rennes1.fr/cti/people/permanent-staff/rocquefelte-xavier

ICAMM2019 : VASP Workshop and International Materials Modelling Conf
June 26-July 3 2019, Rennes France
icamm2019.sciencesconf.org  : Registration opening soon, see you there!

___
Wien mailing list
Wien@zeus.theochem.tuwien.ac.at
http://zeus.theochem.tuwien.ac.at/mailman/listinfo/wien
SEARCH the MAILING-LIST at:  
http://www.mail-archive.com/wien@zeus.theochem.tuwien.ac.at/index.html


Re: [Wien] LDA vs GGA

2020-04-10 Thread Xavier Rocquefelte

For such a system DFT is not sufficient. You must use DMFT.

In addition, you should include spin-orbit coupling.

It is thus a very difficult situation because DMFT+SOC is not trivial at 
all.


Cheers

Xavier


Le 10/04/2020 à 11:39, Amit Chauhan ph17d008 a écrit :

Dear All:

I am working on a strongly spin-orbital coupled d^5 system, SrIrO3 
(orthorhombic). It's a Dirac semimetal with weakly correlated 
(U=0.3-0.4 eV) non-magnetic ground state. With GGA+U, for U=1eV, the 
system is showing a magnetic metal phase but with LDA it is showing 
complely non-magnetic state. I also checked for U=2 eV, but still GGA 
gave a magnetic solution but LDA remains non-magnetic. I have also 
used a very fine k-mesh (16x16x12) but still LDA remains non-magnetic. 
I also checked it with PAW pseudopotential method as implemented in 
VASP, for both GGA and LDA, and results were consistent with the full 
potential GGA. Why full potential LDA is not showing the magnetic 
state??. I have properly checked the inorb and indmc files and the 
parameters are fine.


Regards,

Amit Chauhan
Research Scholar
IIT Madras


___
Wien mailing list
Wien@zeus.theochem.tuwien.ac.at
http://zeus.theochem.tuwien.ac.at/mailman/listinfo/wien
SEARCH the MAILING-LIST at:  
http://www.mail-archive.com/wien@zeus.theochem.tuwien.ac.at/index.html


--

Institut des Sciences Chimiques de Rennes (ISCR)
Univ Rennes - CNRS - UMR6226, France
https://iscr.univ-rennes1.fr/cti/people/permanent-staff/rocquefelte-xavier

ICAMM2019 : VASP Workshop and International Materials Modelling Conf
June 26-July 3 2019, Rennes France
icamm2019.sciencesconf.org  : Registration opening soon, see you there!

___
Wien mailing list
Wien@zeus.theochem.tuwien.ac.at
http://zeus.theochem.tuwien.ac.at/mailman/listinfo/wien
SEARCH the MAILING-LIST at:  
http://www.mail-archive.com/wien@zeus.theochem.tuwien.ac.at/index.html


Re: [Wien] A basic question

2020-02-16 Thread Xavier Rocquefelte

Dear Shamik,

We can answer your question in many ways.

First of all, DFT is a theory of ground state properties. We thus do not 
expect good simulations of excited states properties!


However, we are commonly using DFT to reproduce excited states 
properties (optical, magnetic ...), while we must go beyond DFT (GW, 
TDDFT, HF-CI, ...).


Thus we have to be pragmatic because we want to investigate properties 
of solid state materials, which are usually based on large number of atoms.


Using specific functionals allow to describe a wide range of materials 
and properties, but it requires a deep knowledge of all these 
functionals (limitations ...).


Long time ago, researchers in solid state science were mainly using LDA 
and GGA, knowing their limitations and thus knowing what to do with 
these functionals and how to improve them when localized states and/or 
strong correlation were present (LDA+U, GGA+U). Now, we have more 
functionals and more to know.


In addition, you must be aware that mBJ is not a functional but a 
semilocal potential. It means that using mBJ you cannot use the energy 
to estimate the atomic forces for instance. However, using TB-mBJ you 
can really obtain band gaps of the quality of GW calculations with the 
price of a LDA calculation.


Pratically speaking, one today strategy is to use a functional for the 
geometry optimization and if your system is too large for an hybrid 
functional, use TB-mBJ for the estimation of the band gap, effective 
masses ...


To conclude, there is no unique answer to your question but the more 
important is to learn the subtle details beyond each functionals.


You will then see that you need only very few of them ... because the 
more important is not to be exact but to be able to understand and 
explain the relationship between the properties and the atomic structure 
through the electronic structure.


Best regards

Xavier



Le 17/02/2020 à 07:44, shamik chakrabarti a écrit :


Dear Wien2k users & Experts,

                                 I have a basic question regarding 
simulation using different functionals. I have learned that simulation 
using mbj would provide more accurate band gap than it is provided by 
either GGA or GGA+U or nlvdw. On the contrary, if we want to check 
accurate lattice parameters we go for rev-vdW-DF2 & again if we need 
to check accurate cohesive energy we go for SCAN. So, for checking 
different parameters we use different functionals while one property 
with one functional may be correct & at the same time one property 
with the same functional is not accurate enough.


Why is that so? Why there is no unique functional by using which we 
can get all the properties relatively accurately.


Looking forward to your esteemed advices.

with regards,

--
Dr. Shamik Chakrabarti
Research Fellow
Department of Physics
Indian Institute of Technology Patna
Bihta-801103
Patna
Bihar, India

___
Wien mailing list
Wien@zeus.theochem.tuwien.ac.at
http://zeus.theochem.tuwien.ac.at/mailman/listinfo/wien
SEARCH the MAILING-LIST at:  
http://www.mail-archive.com/wien@zeus.theochem.tuwien.ac.at/index.html


--

Institut des Sciences Chimiques de Rennes (ISCR)
Univ Rennes - CNRS - UMR6226, France
https://iscr.univ-rennes1.fr/cti/people/permanent-staff/rocquefelte-xavier

ICAMM2019 : VASP Workshop and International Materials Modelling Conf
June 26-July 3 2019, Rennes France
icamm2019.sciencesconf.org  : Registration opening soon, see you there!

___
Wien mailing list
Wien@zeus.theochem.tuwien.ac.at
http://zeus.theochem.tuwien.ac.at/mailman/listinfo/wien
SEARCH the MAILING-LIST at:  
http://www.mail-archive.com/wien@zeus.theochem.tuwien.ac.at/index.html


Re: [Wien] Beween non spin polarized and spin polarized calculations

2019-11-22 Thread Xavier Rocquefelte

I am working on similar compounds but not with Mn.

I would recommend to use PBE+U or PBE0 on-site hybrid.

In the first case (PBE+U) you must define an appropriate value for Ueff 
of Mn-3d states.


In the second case, use alpha = 0.25 for only M-3d states.

It should be a not to bad approximation. However, it can be tricky 
because of the metallic nature of the system.


Thus, the best would be to have experimental data to compare.

Regards

Xavier


Le 22/11/2019 à 11:52, djamel slamnia a écrit :

the chemical composition are Cu2MnZrSe4    Cu2MnZrS4



___
Wien mailing list
Wien@zeus.theochem.tuwien.ac.at
http://zeus.theochem.tuwien.ac.at/mailman/listinfo/wien
SEARCH the MAILING-LIST at:  
http://www.mail-archive.com/wien@zeus.theochem.tuwien.ac.at/index.html


Re: [Wien] Beween non spin polarized and spin polarized calculations

2019-11-22 Thread Xavier Rocquefelte

OK so you have transition metal elements surrounded by ligands (S, Se) ...

For copper, it depends if its oxidation state is +2 or +1, leading to 10 
and 9 electrons in the Cu-3d states, respectively.


For Manganese you have to use DFT+U or DFT+onsite hybrid functional or 
full hybrid if not metallic.


For zirconium, you should have no difficulties.

I cannot say more  without the chemical composition ...

Le 22/11/2019 à 09:13, djamel slamnia a écrit :

thanks Mr for replay

my compounds one of them is metalic and the second is half metalic. 
both compounds containing d-Orbitals of transition metal (Cu ,Mn , Zr 
, S , Se)

i thought that -hf module can be the best methode

What do you suggest  ??
thanks in advance

Le vendredi 22 novembre 2019 à 08:00:02 UTC+1, Peter Blaha 
 a écrit :



Please: Do all the examples from our wien2k-workshop (from
www.wien2k.at) first !  Learn how to run WIEN2k properly on small 
examples.


I guess you do all wrong. A wrong command (even one soingle letter) can
spoil the calculations.

a) If your compound is metallic, it is very doubtful that a hybrid
calculation is good. Hybrid-DFT is not good for metals.

b) runsp_lapw -hf -ec -f 1 0.0001 -cc 0.001 -NI

This command has several errors. What should that be:
-ec -f 1 0.0001    
And I doubt that a non-parallel (without -p) calculation runs
meaningfully for 200 cycles with -hf. This would run VERY long, unless
your k-mesh is very small.

c) You said you did geometry optimization "successfully" ! ?

did you use    runsp -min -fc 1 ...    for this purpose ???

Did you check that the forces were small at the end in your case.scf
file ???


Am 21.11.2019 um 20:51 schrieb djamel slamnia:
> I am very grateful to communicate with you
>
> my compound is quaternary all atoms are d-Orbitals of  transition metal
>
> i did the geometry optimisation successfuly
> Now i'm trying to do minimisation of position with hf
> after i intialisation init_hf_lapw  i filled nband i did this  (
> runsp_lapw -hf -ec -f 1 0.0001 -cc 0.001 -NI)
> but with more then 200 iterations never get convegence ???
>
> Le jeudi 21 novembre 2019 à 18:27:12 UTC+1, Xavier Rocquefelte
> <mailto:xavier.rocquefe...@univ-rennes1.fr>> a écrit :

>
>
> If you want our help we need to have details about your system.
>
> Then we can advice you concerning the best strategy.
>
> Best Regards
>
> Xavier
>
> Le 21/11/2019 à 18:02, djamel slamnia a écrit :
> thanks a lot Mr XAVIER for
>   i'm traying hard to do option 2.a minimisation position with hybrid
> functionel but the process did not converged after more than 200
> iteration ??
> can you Suggests me solutio
> Le jeudi 21 novembre 2019 à 16:51:45 UTC+1, Xavier Rocquefelte
> <mailto:xavier.rocquefe...@univ-rennes1.fr>>
> <mailto:xavier.rocquefe...@univ-rennes1.fr 
<mailto:xavier.rocquefe...@univ-rennes1.fr>> a écrit :

>
>
> Hummm ... Here are many options ... choose the one you like ;)
>
> 1/ You consider the importance to have hybrid functional to properly
> describe the atomic structure. If not, optimize using regular DFT and
> then estimate the property of interest with hybrid functional using to
> optimized structure in regular DFT.
>
> 2/ You really want to optimize the geometry in hybrid because your
> system requires such a level of accuracy:
>
> 2.a. you implement the optimization using hybrid functional in WIEN2k
> and all the community will be happy of such a contribution
>
> 2.b. you use a plane-wave code (the one you like) and you optimize in
> hybrid. Then you use this optimized structure in WIEN2k to simulate a
> specific property or to do an accurate analysis.
>
> Regards,
>
> Xavier
>
> Le 21/11/2019 à 16:41, djamel slamnia a écrit :
> so what  i have to do now ??? thanks in advance
>
> Le mercredi 20 novembre 2019 à 15:42:59 UTC+1, Tran, Fabien
> mailto:fabien.t...@tuwien.ac.at>> 
<mailto:fabien.t...@tuwien.ac.at <mailto:fabien.t...@tuwien.ac.at>> a 
écrit :

>
>
> If you are trying to do an optimization of atom positions, then you can
> not do it with hybrid functional (-hf option) because the forces are 
not

> implemented for hybrid functionals.
>
>
> 
> *From:* Wien <mailto:wien-boun...@zeus.theochem.tuwien.ac.at>>
> <mailto:wien-boun...@zeus.theochem.tuwien.ac.at 
<mailto:wien-boun...@zeus.theochem.tuwien.ac.at>> on behalf of djamel
> slamnia mailto:djamel4...@yahoo.fr>> 
<mailto:djamel4...@yahoo.fr <mailto:djamel4...@yahoo.fr>>

> *Sent:* Wednesday, November 20, 2019 2:41 PM
> *To:* A Mailing list for WIEN2k users
> *Subject:* Re: [Wien] Beween non spin polarized and spin polarized
> calculations
> i'm  u

Re: [Wien] Beween non spin polarized and spin polarized calculations

2019-11-21 Thread Xavier Rocquefelte

If you want our help we need to have details about your system.

Then we can advice you concerning the best strategy.

Best Regards

Xavier

Le 21/11/2019 à 18:02, djamel slamnia a écrit :

thanks a lot Mr XAVIER for
 i'm traying hard to do option 2.a minimisation position with hybrid  
functionel but the process did not converged after more than 200 
iteration ??

can you Suggests me solutio
Le jeudi 21 novembre 2019 à 16:51:45 UTC+1, Xavier Rocquefelte 
 a écrit :



Hummm ... Here are many options ... choose the one you like ;)

1/ You consider the importance to have hybrid functional to properly 
describe the atomic structure. If not, optimize using regular DFT and 
then estimate the property of interest with hybrid functional using to 
optimized structure in regular DFT.


2/ You really want to optimize the geometry in hybrid because your 
system requires such a level of accuracy:


2.a. you implement the optimization using hybrid functional in WIEN2k 
and all the community will be happy of such a contribution


2.b. you use a plane-wave code (the one you like) and you optimize in 
hybrid. Then you use this optimized structure in WIEN2k to simulate a 
specific property or to do an accurate analysis.


Regards,

Xavier

Le 21/11/2019 à 16:41, djamel slamnia a écrit :
so what  i have to do now ??? thanks in advance

Le mercredi 20 novembre 2019 à 15:42:59 UTC+1, Tran, Fabien 
 <mailto:fabien.t...@tuwien.ac.at> a écrit :



If you are trying to do an optimization of atom positions, then you 
can not do it with hybrid functional (-hf option) because the forces 
are not implemented for hybrid functionals.




*From:* Wien  
<mailto:wien-boun...@zeus.theochem.tuwien.ac.at> on behalf of djamel 
slamnia  <mailto:djamel4...@yahoo.fr>

*Sent:* Wednesday, November 20, 2019 2:41 PM
*To:* A Mailing list for WIEN2k users
*Subject:* Re: [Wien] Beween non spin polarized and spin polarized 
calculations
i'm  using hf hybrid functional i tryed to do minimisation MSR1 but i 
cant get convergence

what is the solution in your opinion ??   thank in advance

Le dimanche 17 novembre 2019 à 18:36:12 UTC+1, Abderrahmane Reggad 
 <mailto:abde.reg...@gmail.com> a écrit :



Thanks xavier for the interesting article and useful information

My new question is : what's the relationship between the unpaired 
electrons and the orbital contribution. I think that the unpaired 
electrons are always related to the spin contribution.


Best regards

--
Dr. Abderrahmane Reggad
Engineering Physics Laboratory
Faculty of Material Sciences, Ibn Khaldoun University, Tiaret, 14000, 
Algeria

Tel: +213(0)561861963 - Algeria
___
Wien mailing list
Wien@zeus.theochem.tuwien.ac.at <mailto:Wien@zeus.theochem.tuwien.ac.at>
http://zeus.theochem.tuwien.ac.at/mailman/listinfo/wien
SEARCH the MAILING-LIST at: 
http://www.mail-archive.com/wien@zeus.theochem.tuwien.ac.at/index.html


___
Wien mailing list
Wien@zeus.theochem.tuwien.ac.at <mailto:Wien@zeus.theochem.tuwien.ac.at>
http://zeus.theochem.tuwien.ac.at/mailman/listinfo/wien
SEARCH the MAILING-LIST at: 
http://www.mail-archive.com/wien@zeus.theochem.tuwien.ac.at/index.html


___ Wien mailing list 
Wien@zeus.theochem.tuwien.ac.at 
<mailto:Wien@zeus.theochem.tuwien.ac.at> 
http://zeus.theochem.tuwien.ac.at/mailman/listinfo/wien SEARCH the 
MAILING-LIST at: 
http://www.mail-archive.com/wien@zeus.theochem.tuwien.ac.at/index.html


--

Institut des Sciences Chimiques de Rennes (ISCR)
Univ Rennes - CNRS - UMR6226, France
https://iscr.univ-rennes1.fr/cti/people/permanent-staff/rocquefelte-xavier

ICAMM2019 : VASP Workshop and International Materials Modelling Conf
June 26-July 3 2019, Rennes France
icamm2019.sciencesconf.org  : Registration opening soon, see you there!
___
Wien mailing list
Wien@zeus.theochem.tuwien.ac.at <mailto:Wien@zeus.theochem.tuwien.ac.at>
http://zeus.theochem.tuwien.ac.at/mailman/listinfo/wien
SEARCH the MAILING-LIST at: 
http://www.mail-archive.com/wien@zeus.theochem.tuwien.ac.at/index.html


___
Wien mailing list
Wien@zeus.theochem.tuwien.ac.at
http://zeus.theochem.tuwien.ac.at/mailman/listinfo/wien
SEARCH the MAILING-LIST at:  
http://www.mail-archive.com/wien@zeus.theochem.tuwien.ac.at/index.html


--

Institut des Sciences Chimiques de Rennes (ISCR)
Univ Rennes - CNRS - UMR6226, France
https://iscr.univ-rennes1.fr/cti/people/permanent-staff/rocquefelte-xavier

ICAMM2019 : VASP Workshop and International Materials Modelling Conf
June 26-July 3 2019, Rennes France
icamm2019.sciencesconf.org  : Registration opening soon, see you there!


Re: [Wien] Beween non spin polarized and spin polarized calculations

2019-11-21 Thread Xavier Rocquefelte

Hummm ... Here are many options ... choose the one you like ;)

1/ You consider the importance to have hybrid functional to properly 
describe the atomic structure. If not, optimize using regular DFT and 
then estimate the property of interest with hybrid functional using to 
optimized structure in regular DFT.


2/ You really want to optimize the geometry in hybrid because your 
system requires such a level of accuracy:


2.a. you implement the optimization using hybrid functional in WIEN2k 
and all the community will be happy of such a contribution


2.b. you use a plane-wave code (the one you like) and you optimize in 
hybrid. Then you use this optimized structure in WIEN2k to simulate a 
specific property or to do an accurate analysis.


Regards,

Xavier

Le 21/11/2019 à 16:41, djamel slamnia a écrit :

so what  i have to do now ???  thanks in advance

Le mercredi 20 novembre 2019 à 15:42:59 UTC+1, Tran, Fabien 
 a écrit :



If you are trying to do an optimization of atom positions, then you 
can not do it with hybrid functional (-hf option) because the forces 
are not implemented for hybrid functionals.




*From:* Wien  on behalf of 
djamel slamnia 

*Sent:* Wednesday, November 20, 2019 2:41 PM
*To:* A Mailing list for WIEN2k users
*Subject:* Re: [Wien] Beween non spin polarized and spin polarized 
calculations
i'm  using hf hybrid functional i tryed to do minimisation MSR1 but i 
cant get convergence

what is the solution in your opinion ??   thank in advance

Le dimanche 17 novembre 2019 à 18:36:12 UTC+1, Abderrahmane Reggad 
 a écrit :



Thanks xavier for the interesting article and useful information

My new question is : what's the relationship between the unpaired 
electrons and the orbital contribution. I think that the unpaired 
electrons are always related to the spin contribution.


Best regards

--
Dr. Abderrahmane Reggad
Engineering Physics Laboratory
Faculty of Material Sciences, Ibn Khaldoun University, Tiaret, 14000, 
Algeria

Tel: +213(0)561861963 - Algeria
___
Wien mailing list
Wien@zeus.theochem.tuwien.ac.at <mailto:Wien@zeus.theochem.tuwien.ac.at>
http://zeus.theochem.tuwien.ac.at/mailman/listinfo/wien
SEARCH the MAILING-LIST at: 
http://www.mail-archive.com/wien@zeus.theochem.tuwien.ac.at/index.html

___
Wien mailing list
Wien@zeus.theochem.tuwien.ac.at <mailto:Wien@zeus.theochem.tuwien.ac.at>
http://zeus.theochem.tuwien.ac.at/mailman/listinfo/wien
SEARCH the MAILING-LIST at: 
http://www.mail-archive.com/wien@zeus.theochem.tuwien.ac.at/index.html


___
Wien mailing list
Wien@zeus.theochem.tuwien.ac.at
http://zeus.theochem.tuwien.ac.at/mailman/listinfo/wien
SEARCH the MAILING-LIST at:  
http://www.mail-archive.com/wien@zeus.theochem.tuwien.ac.at/index.html


--

Institut des Sciences Chimiques de Rennes (ISCR)
Univ Rennes - CNRS - UMR6226, France
https://iscr.univ-rennes1.fr/cti/people/permanent-staff/rocquefelte-xavier

ICAMM2019 : VASP Workshop and International Materials Modelling Conf
June 26-July 3 2019, Rennes France
icamm2019.sciencesconf.org  : Registration opening soon, see you there!

___
Wien mailing list
Wien@zeus.theochem.tuwien.ac.at
http://zeus.theochem.tuwien.ac.at/mailman/listinfo/wien
SEARCH the MAILING-LIST at:  
http://www.mail-archive.com/wien@zeus.theochem.tuwien.ac.at/index.html


Re: [Wien] Beween non spin polarized and spin polarized calculations

2019-11-17 Thread Xavier Rocquefelte

I recommend you the following article:

https://journals.aps.org/prl/abstract/10.1103/PhysRevLett.108.047201

Bulk gold is diamagnetic and it exhibits a tiny Pauli and Orbital 
paramagnetic state.


Pauli paramagnetism is due to the non-zero density at the Fermi level 
(metal) and the orbital paramagnetism to the presence of unpaired electrons.


Best regards

Xavier

Le 16/11/2019 à 20:59, Abderrahmane Reggad a écrit :
Since the ground state of the Au metal is non magnetic, does it mean 
that the Au metal is diamagnetic or paramagnetic and why ?



--
Dr. Abderrahmane Reggad
Engineering Physics Laboratory
Faculty of Material Sciences, Ibn Khaldoun University, Tiaret, 14000, 
Algeria

Tel: +213(0)561861963 - Algeria

___
Wien mailing list
Wien@zeus.theochem.tuwien.ac.at
http://zeus.theochem.tuwien.ac.at/mailman/listinfo/wien
SEARCH the MAILING-LIST at:  
http://www.mail-archive.com/wien@zeus.theochem.tuwien.ac.at/index.html


--

Institut des Sciences Chimiques de Rennes (ISCR)
Univ Rennes - CNRS - UMR6226, France
https://iscr.univ-rennes1.fr/cti/people/permanent-staff/rocquefelte-xavier

ICAMM2019 : VASP Workshop and International Materials Modelling Conf
June 26-July 3 2019, Rennes France
icamm2019.sciencesconf.org  : Registration opening soon, see you there!

___
Wien mailing list
Wien@zeus.theochem.tuwien.ac.at
http://zeus.theochem.tuwien.ac.at/mailman/listinfo/wien
SEARCH the MAILING-LIST at:  
http://www.mail-archive.com/wien@zeus.theochem.tuwien.ac.at/index.html


Re: [Wien] Error while treating Sm 4f states as core

2019-10-20 Thread Xavier Rocquefelte

Dear Anup,

The method you want to use is an "old" one which is now not anymore 
used, except if you really have no other choice.


It is preferred to use an Hubbard (or an on-site hybrid) correction 
which will allow to correct the DFT error concerning the treatment of 
the 4f states of Sm.


I recommend you to redo the calculation using LDA+U or GGA+U with a 
large U value. Look at the litterature of previous DFT+U calculations 
for Sm. I imagine that a value of about 8 eV should do the job if you 
don't need an exact position of the 4f states but just want to push them 
in the semi-core region. In such a case, you will have to do 
spin-polarized calculations.


Best regards

Xavier

Le 19/10/2019 à 17:45, Anup Shakya a écrit :

Dear All,

I am trying to perform non-magnetic calculations for SmBi treating Sm 
4f electrons in the core region. I have followed the example given for 
Yb by Prof. Blaha. The steps that I have performed are as follows.


I performed a normal non-magnetic scf calculation using run_lapw and 
from there found out where the Sm 4f states are from the case. scf 
file. In case.inc file, I added two lines 4, 3, 5 and 4, -4, 0, (to 
keep Sm 4f 5/2 and Sm 4f 7/2 states in the core and since Sm in +3 
configuration has 5 electrons, so I have kept the occupations no as 5 
for Sm 4f 5/2 and 0 for Sm 4f 7/2) increased the number of orbitals 
from 14 to 16 and added a downward shift of 0.6 Ry. In case.in1, I 
changed the value for Sm 4f states from


3    0.30     0.0010 CONT 1
to
3   -1.00   0.0010 CONT 1,
so that the Sm 4f states will not be found by lapw1.
After that in case.in2 I removed 5 electrons and reduced the number NE
TOT             (TOT,FOR,QTL,EFG,FERMI)
   -12.0    31.0   0.50 0.05  1   EMIN, NE, ESEPERMIN, ESEPER0, iqtlsave

from 31 to 26 as follows:

OT             (TOT,FOR,QTL,EFG,FERMI)
   -12.0    26.0   0.50 0.05  1   EMIN, NE, ESEPERMIN, ESEPER0, iqtlsave
After this I ran dstart first and then did a normal scf run. For the 
first 39 cycles there was no problem but then an error came in the 
40th scf cycle as follows:


LAPW2 - FERMI; weights written
L2main - QTL-B Error
L2main - QTL-B Error

cat *.error
 'l2main' - QTL-B.GT.15., Ghostbands, check scf files
 'l2main' - QTL-B.GT.15., Ghostbands, check scf files

lse
4 -rw-r--r-- 1 root root 54 Oct 19 00:58 uplapw2_29.error
4 -rw-r--r-- 1 root root 54 Oct 19 00:58 uplapw2_2.error
4 -rw-r--r-- 1 root root 54 Oct 19 00:58 uplapw2_30.error
4 -rw-r--r-- 1 root root 54 Oct 19 00:58 uplapw2_31.error
4 -rw-r--r-- 1 root root 54 Oct 19 00:58 uplapw2_32.error
4 -rw-r--r-- 1 root root 54 Oct 19 00:58 uplapw2_33.error
4 -rw-r--r-- 1 root root 54 Oct 19 00:58 uplapw2_34.error
4 -rw-r--r-- 1 root root 54 Oct 19 00:58 uplapw2_35.error
4 -rw-r--r-- 1 root root 54 Oct 19 00:58 uplapw2_36.error

:FER  : F E R M I - ENERGY(TETRAH.M.)=   0.0628473771
I followed previous threads and then tried everything as suggested by 
Prof. Blaha in the link as follows:
http://susi.theochem.tuwien.ac.at/reg_user/faq/qtlb.html. But I have 
not been able to overcome this problem.


I have two questions.

1) Is there any mistake in the steps which I have performed for 
treating Sm 4f as core states. Should I do spin polarized calculations?


2) If there is no problems with my steps, then how should I overcome 
this error. I have tried everything possible from my side. I would be 
grateful if anyone in the forum could help me to overcome this 
problem. If you need any more information, please let me know. Looking 
forward to hearing from you.


Sincerely,
Anup Pradhan Sakhya (Ph.D.)
Visiting Post-Doctoral Fellow
DCMP, TIFR, Mumbai
Alternative Email:anup.sak...@tifr.res.in 
<mailto:rajib.mon...@tifr.res.in>


___
Wien mailing list
Wien@zeus.theochem.tuwien.ac.at
http://zeus.theochem.tuwien.ac.at/mailman/listinfo/wien
SEARCH the MAILING-LIST at:  
http://www.mail-archive.com/wien@zeus.theochem.tuwien.ac.at/index.html


--

Institut des Sciences Chimiques de Rennes (ISCR)
Univ Rennes - CNRS - UMR6226, France
https://iscr.univ-rennes1.fr/cti/people/permanent-staff/rocquefelte-xavier

ICAMM2019 : VASP Workshop and International Materials Modelling Conf
June 26-July 3 2019, Rennes France
icamm2019.sciencesconf.org  : Registration opening soon, see you there!

___
Wien mailing list
Wien@zeus.theochem.tuwien.ac.at
http://zeus.theochem.tuwien.ac.at/mailman/listinfo/wien
SEARCH the MAILING-LIST at:  
http://www.mail-archive.com/wien@zeus.theochem.tuwien.ac.at/index.html


Re: [Wien] Regarding Relaxation calculation

2019-05-29 Thread Xavier Rocquefelte

See the WIEN2k userguide at pages 177, 178 ...

One can constrain individual positions incase.inMor define linear 
constrains for several po-sitions  usingcase.constraint(thanks to  
B.Yanchitsky  (Kiev,  y...@imag.kiev.ua);  for  detailssee comments in 
the SRCtemplates/template.constraint file).  In case of calculations 
with linearconstrains one should use NEW1 (incase.inM). When 
constraining individual positions and us-ing PORT, one should after 
modifications incase.inMrerunx pairhess -copy(which 
copies.minpairto.minrestartand.minhess).






Le 29/05/2019 à 13:12, Shamim Sk a écrit :

Dear Prof. P. Blaha & WIEN2k Community,

  In relaxation calculation using "run_lapw -min -fc 0.5 -ec 0.0001 
-cc 0.001" , is it possible to keep the positions of some of the atoms 
in the supercell fixed  ?




Thank you,
Shamim Sk
IIT Mandi, HP, India.

___
Wien mailing list
Wien@zeus.theochem.tuwien.ac.at
http://zeus.theochem.tuwien.ac.at/mailman/listinfo/wien
SEARCH the MAILING-LIST at:  
http://www.mail-archive.com/wien@zeus.theochem.tuwien.ac.at/index.html


--

Institut des Sciences Chimiques de Rennes (ISCR)
Univ Rennes - CNRS - UMR6226, France
https://iscr.univ-rennes1.fr/cti/people/permanent-staff/rocquefelte-xavier

ICAMM2019 : VASP Workshop and International Materials Modelling Conf
June 26-July 3 2019, Rennes France
icamm2019.sciencesconf.org  : Registration opening soon, see you there!

___
Wien mailing list
Wien@zeus.theochem.tuwien.ac.at
http://zeus.theochem.tuwien.ac.at/mailman/listinfo/wien
SEARCH the MAILING-LIST at:  
http://www.mail-archive.com/wien@zeus.theochem.tuwien.ac.at/index.html


Re: [Wien] Layer and orbital resolved MAE contribution

2019-05-02 Thread Xavier Rocquefelte
You can use a trick consisting to replace part of the magnetic atoms by 
non-magnetic atoms (having similar radii).


For instance, Zinc for copper ... It works really nicely and a way to 
check the consistency is to do the sum of the individual contributions 
and see if you recover the total signal.


In the past, we did such calculations in many cases, allowing to analyze 
our results.


Best regards,

Xavier

Le 02/05/2019 à 12:36, Vojtech Chlan a écrit :

Hi,

in order to determine contribution of individual atoms or groups of 
atoms to MAE, you can try switching off the spin-orbit interaction for 
some atoms (using the last line in the case.inso file).
In particular, one can switch off all but one atom and thus obtain the 
single-ion contribution to MAE of such individual atom. However, this 
can be a bit confusing, because summing up all the single-ion 
contributions one by one will not give the same MAE as in the standard 
case when one has s-o on all atoms. This is due to neglecting of the 
contribution of pseudodipolar (or higher ranks) interactions to MAE. 
But it can be overcome by combining the results of such a 
single-ion-calculation (spinorbit on n-th atom only, others off) and 
an inverse scenario (s-o switched off for n-th atom, the other atoms 
with s-o). The individual atom then contributes to MAE by half of the 
sum of energies of these two calculations... (Works for pseudodipolar; 
for pseudoquadrupolar etc I guess this might get a bit more complicated.)


To separate contributions of states with different orbital number, you 
might try specifying some energy window (emin/emax) in the case.inso 
file to calculate s-o eigenvalues for some band(s) only, but I never 
tried that and don't know whether it works that way.


Best wishes,
Vojtech


On 02-May-19 11:49, ctiusan wrote:

Dear wien2k users,

I am dealing with magnetic anisotropy calculations in multilatered 
supercells.
From total energy an/or force theorem, I could calculate total MAE 
values. It works quite well.


However, for deeper analysis of surface perpendicular anisotropy 
issues, I would like to get layer and/or orbital contributions to the 
total MAE.
For layer contribution one possibility would be to use the Bruno’s 
 formula, but this only works when splin flip is neglected...that is 
not allways the case...as clearly seen by looking  at density matrix 
components generated by lapwdm when SO is present...


Is there any way in wien2k to get acces to layer and orbital 
decomposition of the total energy, so that to be able to directly 
calculate corresponding resolved MAE?

Could someone help me with some advice related to this?

With thanks in advance,
C. Tiusan

——
Prof. Phd. Eng. Habil. Coriolan Viorel TIUSAN

Department of Physics and Chemistry
Technical University of Cluj-Napoca
Center of Superconductivity, Spintronics and Surface Science
Str. Memorandumului No. 28, RO-400114
 Cluj-Napoca, ROMANIA
    Tel: +40-264-401-465  Fax: +40-264-592-055 


    Cell: +40-732-893-750 
    e-mail: coriolan.tiusan@ 
<mailto:coriolan.tiu...@phys.utcluj.ro>phys.utcluj.ro 
<http://phys.utcluj.ro>

—
Senior Researcher
National Center of Scientific Research - FRANCE
 web: http://www.c4s.utcluj.ro/webperso/tiusan/welcome.html
_



___
Wien mailing list
Wien@zeus.theochem.tuwien.ac.at
http://zeus.theochem.tuwien.ac.at/mailman/listinfo/wien
SEARCH the MAILING-LIST 
at:http://www.mail-archive.com/wien@zeus.theochem.tuwien.ac.at/index.html




___
Wien mailing list
Wien@zeus.theochem.tuwien.ac.at
http://zeus.theochem.tuwien.ac.at/mailman/listinfo/wien
SEARCH the MAILING-LIST at:  
http://www.mail-archive.com/wien@zeus.theochem.tuwien.ac.at/index.html


--

Institut des Sciences Chimiques de Rennes (ISCR)
Univ Rennes - CNRS - UMR6226, France
https://iscr.univ-rennes1.fr/cti/people/permanent-staff/rocquefelte-xavier

ICAMM2019 : VASP Workshop and International Materials Modelling Conf
June 26-July 3 2019, Rennes France
icamm2019.sciencesconf.org  : Registration opening soon, see you there!

___
Wien mailing list
Wien@zeus.theochem.tuwien.ac.at
http://zeus.theochem.tuwien.ac.at/mailman/listinfo/wien
SEARCH the MAILING-LIST at:  
http://www.mail-archive.com/wien@zeus.theochem.tuwien.ac.at/index.html


Re: [Wien] Augmented Plane Wave

2019-03-22 Thread Xavier Rocquefelte

Dear Pablo,

My understanding is that you have a mixed basis set, with plane wave 
outside the atomic spheres which are augmented by atomic functions 
inside the sphere. You have a good explanation in the David Singh book 
but also in the WIEN2k userguide I believe.


Best Regards,

Xavier


Le 22/03/2019 à 02:49, delamora a écrit :

Dear Wien users,
I have a question about the name of
"Augmented Plane Wave"
I had the idea that when the wave enters the Muffin Tin sphere the 
amplitude of the wave increased.

Trying to see this I found that when a wave crosses a step function,
https://quantummechanics.ucsd.edu/ph130a/130_notes/node149.html
When the incoming wave
exp(ikx)
reaches an upwards step function there is a reflected wave
R exp(-ikx)
and a transmitted wave
T exp(ik'x)
what this article shows is;
1 + R = T
That is, the amplitudes of the incoming wave and the reflected wave 
add to the amplitude of the transmitted wave
If I take this into a square well then I would understand that the 
waves inside the well have the total amplitude equal to the incoming 
and transmitted wave. That is, when the wave enters the Muffin Tin the 
amplitude of wave is not AUGMENTED. So why is this method called 
"Augmented Plane Wave"?


Saludos

Pablo


___
Wien mailing list
Wien@zeus.theochem.tuwien.ac.at
http://zeus.theochem.tuwien.ac.at/mailman/listinfo/wien
SEARCH the MAILING-LIST at:  
http://www.mail-archive.com/wien@zeus.theochem.tuwien.ac.at/index.html


--

Institut des Sciences Chimiques de Rennes (ISCR)
Univ Rennes - CNRS - UMR6226, France
https://iscr.univ-rennes1.fr/cti/people/permanent-staff/rocquefelte-xavier

ICAMM2019 : VASP Workshop and International Materials Modelling Conf
June 26-July 3 2019, Rennes France
icamm2019.sciencesconf.org  : Registration opening soon, see you there!

___
Wien mailing list
Wien@zeus.theochem.tuwien.ac.at
http://zeus.theochem.tuwien.ac.at/mailman/listinfo/wien
SEARCH the MAILING-LIST at:  
http://www.mail-archive.com/wien@zeus.theochem.tuwien.ac.at/index.html


Re: [Wien] problems with convergence of SCF for AFM HoPtBi

2018-11-01 Thread Xavier Rocquefelte
Thank you so much Laurence for this explanation which definitely enlight 
my understanding!


Could you just give me more hints. What do you mean by "fixed point 
solution" when you are speaking about the spins?


Brainstorming session:

I am a chemist but I can propose an idea without a clear understanding! 
The rotations are non-linear and we should find a way to express them in 
such a way to be linear.


Is a Fourier transform (or fractional Fourier transform or something 
related) useful in such a case?


Cheers

Xavier




Le 01/11/2018 à 17:19, Laurence Marks a écrit :
Quite a few things are going on "Under the hood" in these systems, so 
let me explain a little.


For heavy atoms with large RMTs the initial pseudocharge (PW density 
inside the RMTs) from dstart is quite bad. In addition, unless you are 
careful to chose a "good" initial spin state in case.inst, the spins 
can be quite far from the fixed point solution. While it might appear 
that the density and energy (:DIS and :ENE) are oscillating, in 
reality the pseudocharge and the spin/orbital moments are rotating 
slowly in a N-Dimensional space. Rotations are non-linear, and the 
mixer (all versions in all DFT codes to my knowledge) use linear 
expansions. It can therefore take a long time for these rotations 
tocomplete. What you will often see if you look at the pseudocharge 
(grep :CPC in 18.1, the pseudocharge was not monited in earlier 
versions as its role was not known) you will often see that it is 
slowly "walking" -- similarly the spin moments etc.


When you add SOC and/or +U (or -eece) there will be large rotations of 
the spins and/or spin moments.


In an ideal world some transformation would be applied to the Wien2k 
variables to separate the pseudocharge and convert the rotations to 
linear variables. I do not know how to do this, and simple ideas such 
as using the sum and difference of spins don't work. (As expected, 
since this is a linear transformation.) I am open to (mathematical) 
suggestions/inspirations. It has to be math not fudge -- the mixer 
only works when the math is right!


On Thu, Nov 1, 2018 at 9:34 AM Xavier Rocquefelte 
<mailto:xavier.rocquefe...@univ-rennes1.fr>> wrote:


I have exactly the same experience. It is really a matter of
cooking to
my point of view and strongly depends on the system and what is
happening at the Fermi level.

In my case I am using DFT+U+SOC for such systems. The hubbard term
indeed allows to separate the 4f states. However, the SOC is also
essential and leads to large effects in these systems.
In the case of HoPtBi, all the elements requires SOC. In addition
it is
an intermetallic compound, meaning that you have both localized and
delocalized electrons!

Did you try the following procedure:
- DFT+SOC in a self-consistent manner
- DFT+SOC+U on top of the first calculation

Sometimes the lift of degeneracy due to SOC is sufficient to converge
more efficiently DFT+SOC and then after DFT+SOC+U. And sometimes
it is
the other way around;)

Best wishes
Xavier


-- 


Institut des Sciences Chimiques de Rennes (ISCR)
Univ Rennes - CNRS - UMR6226, France

https://urldefense.proofpoint.com/v2/url?u=https-3A__iscr.univ-2Drennes1.fr_cti_people_permanent-2Dstaff_rocquefelte-2Dxavier=DwICAg=yHlS04HhBraes5BQ9ueu5zKhE7rtNXt_d012z2PA6ws=U_T4PL6jwANfAy4rnxTj8IUxm818jnvqKFdqWLwmqg0=V1_Z6_elFWMevjRRzb6Er-1CwhwNLWJ5nHzKWh26Fo0=adjkiyhXqoBS6b7SN1a8EitYiDIMwrzO8SLBzLqxJ0M=

ICAMM2019 : VASP Workshop and International Materials Modelling Conf
June 26-July 3 2019, Rennes France
icamm.2019.sciencesconf.org <http://icamm.2019.sciencesconf.org> 
: Registration opening soon, see you there!


___
Wien mailing list
Wien@zeus.theochem.tuwien.ac.at
<mailto:Wien@zeus.theochem.tuwien.ac.at>

https://urldefense.proofpoint.com/v2/url?u=http-3A__zeus.theochem.tuwien.ac.at_mailman_listinfo_wien=DwICAg=yHlS04HhBraes5BQ9ueu5zKhE7rtNXt_d012z2PA6ws=U_T4PL6jwANfAy4rnxTj8IUxm818jnvqKFdqWLwmqg0=V1_Z6_elFWMevjRRzb6Er-1CwhwNLWJ5nHzKWh26Fo0=bHKBKCqE5r9Ol5EZoCphMsvUqWp9VPpa4qmZjlZ76MY=
SEARCH the MAILING-LIST at:

https://urldefense.proofpoint.com/v2/url?u=http-3A__www.mail-2Darchive.com_wien-40zeus.theochem.tuwien.ac.at_index.html=DwICAg=yHlS04HhBraes5BQ9ueu5zKhE7rtNXt_d012z2PA6ws=U_T4PL6jwANfAy4rnxTj8IUxm818jnvqKFdqWLwmqg0=V1_Z6_elFWMevjRRzb6Er-1CwhwNLWJ5nHzKWh26Fo0=QFcXH13Qr3d3q8iYQbINXbHncLwDP5UPJWJ9QJbhqIA=



--
Professor Laurence Marks
"Research is to see what everybody else has seen, and to think what 
nobody else has thought", Albert Szent-Gyorgi
www.numis.northwestern.edu <http://www.numis.northwestern.edu> ; 
Corrosion in 4D: MURI4D.numis.northwestern.edu 
<http://MURI4D.numis.northwestern.edu>

Re: [Wien] problems with convergence of SCF for AFM HoPtBi

2018-11-01 Thread Xavier Rocquefelte
I have exactly the same experience. It is really a matter of cooking to 
my point of view and strongly depends on the system and what is 
happening at the Fermi level.


In my case I am using DFT+U+SOC for such systems. The hubbard term 
indeed allows to separate the 4f states. However, the SOC is also 
essential and leads to large effects in these systems.
In the case of HoPtBi, all the elements requires SOC. In addition it is 
an intermetallic compound, meaning that you have both localized and 
delocalized electrons!


Did you try the following procedure:
- DFT+SOC in a self-consistent manner
- DFT+SOC+U on top of the first calculation

Sometimes the lift of degeneracy due to SOC is sufficient to converge 
more efficiently DFT+SOC and then after DFT+SOC+U. And sometimes it is 
the other way around;)


Best wishes
Xavier


--

Institut des Sciences Chimiques de Rennes (ISCR)
Univ Rennes - CNRS - UMR6226, France
https://iscr.univ-rennes1.fr/cti/people/permanent-staff/rocquefelte-xavier

ICAMM2019 : VASP Workshop and International Materials Modelling Conf
June 26-July 3 2019, Rennes France
icamm.2019.sciencesconf.org  : Registration opening soon, see you there!

___
Wien mailing list
Wien@zeus.theochem.tuwien.ac.at
http://zeus.theochem.tuwien.ac.at/mailman/listinfo/wien
SEARCH the MAILING-LIST at:  
http://www.mail-archive.com/wien@zeus.theochem.tuwien.ac.at/index.html


Re: [Wien] problems with convergence of SCF for AFM HoPtBi

2018-11-01 Thread Xavier Rocquefelte
=  781  R0=.1 RMT= 2.5 Z:  67.
LOCAL ROT MATRIX:1.000 0.000 0.000
 0.000 1.000 0.000
 0.000 0.000 1.000
ATOM  -4: X=0.5000 Y=0.5000 Z=0.5000
  MULT= 1  ISPLIT= 4
Ho2NPT=  781  R0=.1 RMT= 2.5 Z:  67.
LOCAL ROT MATRIX:1.000 0.000 0. 0.000 
0.000 1.000
ATOM  -5: X=0.7500 Y=0.7500 Z=0.7500
  MULT= 1  ISPLIT= 4
Bi1NPT=  781  R0=.05000 RMT= 2.5 Z:  83.
LOCAL ROT MATRIX:1.000 0.000 0.000
 0.000 1.000 0.000
 0.000 0.000 1.000
ATOM  -6: X=0.2500 Y=0.2500 Z=0.2500
  MULT= 1  ISPLIT= 4
Bi2NPT=  781  R0=.05000 RMT= 2.5 Z:  83.
LOCAL ROT MATRIX:1.000 0.000 0.000
 0.000 1.000 0.000
 0.000 0.000 1.000
   6  NUMBER OF SYMMETRY OPERATIONS
1 0 0 0.
0 1 0 0.
0 0 1 0.
   1
0 0 1 0.
1 0 0 0.
0 1 0 0.
   2
0 1 0 0.
0 0 1 0.
1 0 0 0.
   3
0 1 0 0.
1 0 0 0.
0 0 1 0.
   4
1 0 0 0.
0 0 1 0.
0 1 0 0.
   5
0 0 1 0.
0 1 0 0.
1 0 0 0.
   6

Best,
Wang



___
Wien mailing list
Wien@zeus.theochem.tuwien.ac.at
http://zeus.theochem.tuwien.ac.at/mailman/listinfo/wien
SEARCH the MAILING-LIST at:  
http://www.mail-archive.com/wien@zeus.theochem.tuwien.ac.at/index.html


--

Institut des Sciences Chimiques de Rennes (ISCR)
Univ Rennes - CNRS - UMR6226, France
https://iscr.univ-rennes1.fr/cti/people/permanent-staff/rocquefelte-xavier

ICAMM2019 : VASP Workshop and International Materials Modelling Conf
June 26-July 3 2019, Rennes France
icamm.2019.sciencesconf.org  : Registration opening soon, see you there!

___
Wien mailing list
Wien@zeus.theochem.tuwien.ac.at
http://zeus.theochem.tuwien.ac.at/mailman/listinfo/wien
SEARCH the MAILING-LIST at:  
http://www.mail-archive.com/wien@zeus.theochem.tuwien.ac.at/index.html


Re: [Wien] The parameters to obtain an accurate small gap

2018-09-12 Thread Xavier Rocquefelte

Dear Xiangyan,

10 meV is more than very small!

However, accuracy and parameters to adjust (or tune) will really depend 
on the material, the size of the unit cell, the nature of the bonding...


If you want a more accurate answer, we need a more accurate question ;)

Cheers

Xavier



Le 12/09/2018 à 11:33, 薄祥䶮 a écrit :

Dear Dr. P. Blaha and Wien2k users,

I am trying to calculate a material with a very small gap about 10 
meV, and I want to get the value of the gap to be as accurate as possible.
How should I do it? For example, what setting parameters should be 
set? I know I should use a large K-mesh and set the convergence LIMIT 
to be small. What else can I do?


Thank you,
Xiangyan Bo




___
Wien mailing list
Wien@zeus.theochem.tuwien.ac.at
http://zeus.theochem.tuwien.ac.at/mailman/listinfo/wien
SEARCH the MAILING-LIST at:  
http://www.mail-archive.com/wien@zeus.theochem.tuwien.ac.at/index.html


___
Wien mailing list
Wien@zeus.theochem.tuwien.ac.at
http://zeus.theochem.tuwien.ac.at/mailman/listinfo/wien
SEARCH the MAILING-LIST at:  
http://www.mail-archive.com/wien@zeus.theochem.tuwien.ac.at/index.html


Re: [Wien] optics broken symmetry

2018-09-05 Thread Xavier Rocquefelte
As you Laurence, I was thinking about the effect of shifting or not the 
kmesh!


Peter, do you think it will lead to a better convergence?

Cheers

Xavier


Le 03/09/2018 à 13:59, Laurence Marks a écrit :
What you are doing "should" work -- I have done similar things myself. 
I have also managed to do it "not quite right" in the past as well. 
The most obvious possibility is user error.


One thing I would check is shifting the k-mesh. For reasons that I do 
not fully understand this can break symmetry. A good thing to check is 
no shift, this may solve the problem. You may want to test using TEMPS.


You may also need to tighten the convergence. Breaking of symmetry can 
be a very soft mode, and be only satisfied when one pushes to really 
well converged results.


On Sun, Sep 2, 2018 at 10:33 PM, Oleg Rubel > wrote:


Dear Wien2k community,

I try to compute opto-elastic properties of InP (zinc-blend
structure).
It is related to a change of the dielectric constant (real part) in
response to an applied strain. There are no problems with a
response to
a hydrostatic strain, and results agree well with experiments. A
problem
occurs with a uniaxial strain (strained along X-axis only by 0.05%).
Computed change in the dielectric constant is too large (~ an
order of
magnitude).

Trying to trace back the problem, I did the following:
First, I initialize a tetragonaly-distorted zinc-blend structure
(init_lapw -b -vxc 19 -ecut -6.5 -numk 800) with the following
lattice
parameters

F   LATTICE,NONEQUIV.ATOMS:  2

MODE OF CALC=RELA unit=ang

  11.095785 11.090240 11.090240 90.00 90.00 90.00

Then I set the lattice parameters back to the cubic lattice

F   LATTICE,NONEQUIV.ATOMS:  2

MODE OF CALC=RELA unit=ang

  11.090240 11.090240 11.090240 90.00 90.00 90.00

and rerun (x dstart). This allows me to preserve the symmetry of a
distorted structure (see the structure file below).

Next, I run SCF (run_lapw -ec 0.1 -cc 0.0001) and optics with
20x20x20 k-mesh. The results for Re and Im parts of the dielectric
constant are here:

[oleg@feynman InP-w2k]$ head InP-w2k.epsilon
#

# Lorentzian broadening with gamma= 0.10  [eV]
# Im(epsilon) shifted by   0.7860   [eV]
# No intraband contributions added
#
# Energy [eV] Re_eps_xx Im_eps_xx Re_eps_yy  Im_eps_yy
#
0.013610  0.940850E+01  0.988634E-01  0.947674E+01 0.100908E+00
0.040820  0.940928E+01  0.100340E+00  0.947756E+01 0.102453E+00
0.068030  0.941084E+01  0.101855E+00  0.947919E+01 0.104042E+00

It seems that the symmetry is broken, which causes later problems
with
opto-elastic coefficients as change of 0.07 in the second decimal
point
of Re_eps for such a small strain is too much.

Once again, there are no problems when the strain tensor does not
break
the zinc-blend cubic symmetry.

Any thoughts are highly appreciated.


Thank you in advance
Oleg

-- 
Oleg Rubel (PhD, PEng)

Department of Materials Science and Engineering
McMaster University
JHE 359, 1280 Main Street West, Hamilton, Ontario L8S 4L8, Canada
Email: rub...@mcmaster.ca 
Tel: +1-905-525-9140, ext. 24094
Web:

https://urldefense.proofpoint.com/v2/url?u=http-3A__olegrubel.mcmaster.ca=DwICAg=yHlS04HhBraes5BQ9ueu5zKhE7rtNXt_d012z2PA6ws=U_T4PL6jwANfAy4rnxTj8IUxm818jnvqKFdqWLwmqg0=WpMS0L_jtI19kiPXo_QJrB9iBMQX1L9TeNePUM_x2Lw=YKfgMS8xLL_yM_B62Rds1s_GApfimaNcCR8kQSU2LLw=



P.S. I run WIEN2k_16.1 (Release 11/17/2016). Our main cluster is down
for maintenance, so I was not able to check with the newest
version of
Wien2k.

P.P.S. Here is the cubic structure file with a distorted symmetry
that I
run to get the data.

InP

F   LATTICE,NONEQUIV.ATOMS:  2

MODE OF CALC=RELA unit=ang

  11.090240 11.090240 11.090240 90.00 90.00 90.00

ATOM  -1: X=0. Y=0. Z=0.
   MULT= 1  ISPLIT=-2
In NPT=  781  R0=0.1000 RMT=2.   Z: 49.000

LOCAL ROT MATRIX:0.000 0.000 1.000
  1.000 0.000 0.000
  0.000 1.000 0.000
ATOM  -2: X=0.2500 Y=0.2500 Z=0.2500
   MULT= 1  ISPLIT=-2
P  NPT=  781  R0=0.0001 RMT=2.   Z: 15.000

LOCAL ROT MATRIX:0.000 0.000 1.000
  1.000 0.000 0.000
  0.000 1.000 

Re: [Wien] LDA+U +SOC calculations

2018-03-20 Thread Xavier Rocquefelte

Dear Sylwia

I will give you a chemist answer, which appears to work properly in many 
cases.


If you have 3d elements, the crystal field (CF) is usually larger than 
SOC, thus I will do first orb, and then include SO.


If you have 4f elements, the SOC is usually larger than the CF, thus I 
will do first SO and then add orb.


It is not a general recipe. It must be adapted to the system and you 
have to try.


Best Regards

Xavier


Le 20/03/2018 à 18:01, Sylwia Golab a écrit :

Dear Mr Blaha and others,

my question seems to be trivial and I am sorry about that, but I 
cannot find clear answer:
what should we do first: calculations with SOC (without orb) and then 
include orb or calculations with orb (without SOC) and then include SOC?


Best regards,
Sylwia


___
Wien mailing list
Wien@zeus.theochem.tuwien.ac.at
http://zeus.theochem.tuwien.ac.at/mailman/listinfo/wien
SEARCH the MAILING-LIST at:  
http://www.mail-archive.com/wien@zeus.theochem.tuwien.ac.at/index.html


___
Wien mailing list
Wien@zeus.theochem.tuwien.ac.at
http://zeus.theochem.tuwien.ac.at/mailman/listinfo/wien
SEARCH the MAILING-LIST at:  
http://www.mail-archive.com/wien@zeus.theochem.tuwien.ac.at/index.html


Re: [Wien] SO in 3d and 4f elements

2018-03-13 Thread Xavier Rocquefelte

Dear Pablo

It really depends on the property you are considering:

1. In terms of chemical bond, the effect of SO on 3d and 4f elements 
will be negligeable.


2. In terms of spectroscopy it can be significant if you have access to 
d3/2 and d5/2 states, but also p1/2 and p3/2 ... and similar for f states


3. In terms of magnetic properties, it will be negligeable for the 
magnetic exchange couplings but not for the anisotropy and thus the easy 
magnetization direction.



Thus it really depends on the calculation you plan to do and the 
property you are looking for.


Best Regards

Xavier


Le 13/03/2018 à 02:44, delamora a écrit :

Dear WIEN2k community,
I know that spin-orbit coupling in 4d and 5d is important,
but how important is SOC for 3d and 4f elements?

Yours

Pablo de la Mora



___
Wien mailing list
Wien@zeus.theochem.tuwien.ac.at
http://zeus.theochem.tuwien.ac.at/mailman/listinfo/wien
SEARCH the MAILING-LIST at:  
http://www.mail-archive.com/wien@zeus.theochem.tuwien.ac.at/index.html


___
Wien mailing list
Wien@zeus.theochem.tuwien.ac.at
http://zeus.theochem.tuwien.ac.at/mailman/listinfo/wien
SEARCH the MAILING-LIST at:  
http://www.mail-archive.com/wien@zeus.theochem.tuwien.ac.at/index.html


Re: [Wien] Importing .cif files to struct

2018-03-06 Thread Xavier Rocquefelte

Your cif file contains 3 settings explaining why cif2struct cannot work.

Here is the first setting of the cif file.

Cheers

Xavier


Le 06/03/2018 à 12:27, Sherif Yehia a écrit :

Dear  Users

 The attached  .cif file  data have been compiled from

the crystallographic data sheet for
"HoCo3 Crystal Structure" taken from SpringerMaterials 
(sm_isp_sd_0452309).


When I try to use cif2struct   I get the following error

_Cell dimension(s) missing!_
_ F F F_

Can I get some advice  to get  case.struct

Thank you


___
Wien mailing list
Wien@zeus.theochem.tuwien.ac.at
http://zeus.theochem.tuwien.ac.at/mailman/listinfo/wien
SEARCH the MAILING-LIST at:  
http://www.mail-archive.com/wien@zeus.theochem.tuwien.ac.at/index.html




Stc1.cif
Description: application/vnd.multiad.creator.cif
___
Wien mailing list
Wien@zeus.theochem.tuwien.ac.at
http://zeus.theochem.tuwien.ac.at/mailman/listinfo/wien
SEARCH the MAILING-LIST at:  
http://www.mail-archive.com/wien@zeus.theochem.tuwien.ac.at/index.html


Re: [Wien] regarding k-mesh and tolf

2018-01-28 Thread Xavier Rocquefelte
Here is a link towards a paper which treats such systems using GGA and 
HSE06. Indeed, with GGA the band gap is nearly zero for some of them.


https://arxiv.org/pdf/1706.08674.pdf

The point is that it seems to me that it will not be too much 
problematic although your band gap is zero if the bands are simply 
touching at the Fermi level.


You must check your band structure first.

In the paper they show HSEO6 band structures which clearly confirm that 
GGA+U or onsite-hybrid will not be useful.


One option for you could be to do band structure using GGA and 
Full-hybrid (HSE06 or PBE0) to compare and discuss the differences. Then 
if the difference is only band gap reduction, you could use GGA for the 
phonons calculations. In addition, it will allow you to estimate the 
residual forces using both GGA and hybrid, which will be one more 
confirmation (or not) that GGA is sufficient for your system.


Cheers

Xavier


Le 27/01/2018 à 16:01, chin Sabsu a écrit :

Sir, it is A2BX6,  x is halogen.

Sent from Yahoo Mail on Android 
<https://overview.mail.yahoo.com/mobile/?.src=Android>


On Sat, Jan 27, 2018 at 7:23 PM, Xavier Rocquefelte
<xavier.rocquefe...@univ-rennes1.fr> wrote:
___
Wien mailing list
Wien@zeus.theochem.tuwien.ac.at
<mailto:Wien@zeus.theochem.tuwien.ac.at>
http://zeus.theochem.tuwien.ac.at/mailman/listinfo/wien
SEARCH the MAILING-LIST at:
http://www.mail-archive.com/wien@zeus.theochem.tuwien.ac.at/index.html



___
Wien mailing list
Wien@zeus.theochem.tuwien.ac.at
http://zeus.theochem.tuwien.ac.at/mailman/listinfo/wien
SEARCH the MAILING-LIST at:  
http://www.mail-archive.com/wien@zeus.theochem.tuwien.ac.at/index.html


___
Wien mailing list
Wien@zeus.theochem.tuwien.ac.at
http://zeus.theochem.tuwien.ac.at/mailman/listinfo/wien
SEARCH the MAILING-LIST at:  
http://www.mail-archive.com/wien@zeus.theochem.tuwien.ac.at/index.html


Re: [Wien] regarding k-mesh and tolf

2018-01-27 Thread Xavier Rocquefelte
Could you give the chemical formula? It will really help answering your 
question from my side.


Cheers

Xavier


Le 27/01/2018 à 14:34, chin Sabsu a écrit :

Dear Respected Xavaier Sir
Thank you very much for detailed explanation.
My system is not having any d-d interaction (no TM element) so I do 
not think I should go for  DFT+U, Onsite Hybrid functional and Full 
hybrid calculations!


With TB-mBJ it is showing a band gap. The VBM is widely dispersed, 
flat for a wide range, near the VBM. So this is a semiconductor.


As we can not relax the structure using mBJ so there is no way to use 
it for the phonopy purpose (phonon bands).


Due to lack of good computational facilities, I want to avoid hybrid 
calculation which will never finish if I go for Phonopy.


What I see it, I use the combination of tolf and -fc such that it 
gives minimum forces. But I am sure the forces will not be less than 
2mRy/Bohr. Again this process I see a contradiction with Peter Sir's 
statement: forces should be less than 1mRy/bohr for phonon 
calculations and which I will not be having!!
How much the ~2-3mRy/bohr forces on the atom affects the phonon band 
structure?
can I proceed for phonon bands with the structure having forces ~2-3 
mRy/Bohr?


Thank you very much!

Chin S.




On Saturday 27 January 2018, 6:31:35 PM IST, Xavier Rocquefelte 
<xavier.rocquefe...@univ-rennes1.fr> wrote:



Dear Chin S.

Your system is metallic from the point of view of DFT which is not a 
surprise.


In your last calculation, which is the more accurate you simply catch 
this fact ... In other words in your previous calculations you had 0.1 
eV gap due to less kpoints and/or less accurate calculations. To 
really understand what is happening you should draw the DOS and 
bandstructure to see if you have band crossing at the Fermi level 
which could lead to occupancy of an expected empty band. If yes, it 
will have an impact on the chemical bond description and also on the 
phonons.


Depending on the system you will have to consider methods going beyond 
DFT, which in WIEN2k could be based on:


- DFT+U

- Onsite Hybrid functional

- Full hybrid

- TB-mBJ potential

If your band gap is based on charge transfer transitions you cannot 
used DFT+U and onsite-hybrid.
Thus the less expensive approach will be TB-mBJ potential which will 
give the band gap but not the forces.


Thus only one option remains which is full hybrid.

If your system is based on d-d transitions, or highly correlated 
states, then you can used DFT+U or Onsite-hybrid.


Hope it will help you.

Cheers

Xavier


Le 27/01/2018 à 12:28, chin Sabsu a écrit :

Dear Stefaan Sir

Below are my updates:

A. For mesh size
I got your point and in all different distorted structure (with 
different inequivalent atoms) I am having the same number of 
electrons while the number of inequivalent atoms are different. NE 
varies from 154 to 616(1x1x1 cell) to 4928 (2x2x2 cell).
So I will divide xxx numk by 4 and 32 in 1x1x1 and 2x2x2 supercell 
structure. Thanks for this excellent recommendation!!!
For 2x2x2 supercell in phonopy calculations, we need to go for mpi 
jobs for certain cases!!


B. Now I have done some tests for my cubic perovskite and below are 
my observations:


B1. For tolf 5 and -fc1 I for FOR003 = ~4 mRy / bohr   with band gap 
~0.1 eV
B2. For tolf 2 and - fc 0.1 forces are almost same. and the band gap 
is still ~0.1ev


B3. Now to reduce the forces less then 0.1mRy/ bohr , I reduced the 
tolf to 0.5 and - fc 0.1 - 0.01. I got forces at the desired accurecy 
but the band gap vanished and the system becomes metallic. The 
experimentally reported band gap is 1.2eV

I used RKMAX 8, numk 2000, vxc 19.

What is your suggestion for point number B, why I am getting the 
system metallic?

Can I use this relaxed structure for Phonopy?

Kind regards

Chin S.


For t olf



___
Wien mailing list
Wien@zeus.theochem.tuwien.ac.at <mailto:Wien@zeus.theochem.tuwien.ac.at>
http://zeus.theochem.tuwien.ac.at/mailman/listinfo/wien
SEARCH the MAILING-LIST 
at:http://www.mail-archive.com/wien@zeus.theochem.tuwien.ac.at/index.html


___
Wien mailing list
Wien@zeus.theochem.tuwien.ac.at <mailto:Wien@zeus.theochem.tuwien.ac.at>
http://zeus.theochem.tuwien.ac.at/mailman/listinfo/wien
SEARCH the MAILING-LIST at: 
http://www.mail-archive.com/wien@zeus.theochem.tuwien.ac.at/index.html



___
Wien mailing list
Wien@zeus.theochem.tuwien.ac.at
http://zeus.theochem.tuwien.ac.at/mailman/listinfo/wien
SEARCH the MAILING-LIST at:  
http://www.mail-archive.com/wien@zeus.theochem.tuwien.ac.at/index.html


___
Wien mailing list
Wien@zeus.theochem.tuwien.ac.at
http://zeus.theochem.tuwien.ac.at/mailman/listinfo/wien
SEARCH the MAILING-LIST at:  
http://www.mail-archive.com/wien@zeus.theochem.tuwien.ac.at/index.html


Re: [Wien] regarding k-mesh and tolf

2018-01-27 Thread Xavier Rocquefelte

Dear Chin S.

Your system is metallic from the point of view of DFT which is not a 
surprise.


In your last calculation, which is the more accurate you simply catch 
this fact ... In other words in your previous calculations you had 0.1 
eV gap due to less kpoints and/or less accurate calculations. To really 
understand what is happening you should draw the DOS and bandstructure 
to see if you have band crossing at the Fermi level which could lead to 
occupancy of an expected empty band. If yes, it will have an impact on 
the chemical bond description and also on the phonons.


Depending on the system you will have to consider methods going beyond 
DFT, which in WIEN2k could be based on:


- DFT+U

- Onsite Hybrid functional

- Full hybrid

- TB-mBJ potential

If your band gap is based on charge transfer transitions you cannot used 
DFT+U and onsite-hybrid.
Thus the less expensive approach will be TB-mBJ potential which will 
give the band gap but not the forces.


Thus only one option remains which is full hybrid.

If your system is based on d-d transitions, or highly correlated states, 
then you can used DFT+U or Onsite-hybrid.


Hope it will help you.

Cheers

Xavier


Le 27/01/2018 à 12:28, chin Sabsu a écrit :

Dear Stefaan Sir

Below are my updates:

A. For mesh size
I got your point and in all different distorted structure (with 
different inequivalent atoms) I am having the same number of electrons 
while the number of inequivalent atoms are different. NE varies from 
154 to 616(1x1x1 cell) to 4928 (2x2x2 cell).
So I will divide xxx numk by 4 and 32 in 1x1x1 and 2x2x2 supercell 
structure. Thanks for this excellent recommendation!!!
For 2x2x2 supercell in phonopy calculations, we need to go for mpi 
jobs for certain cases!!


B. Now I have done some tests for my cubic perovskite and below are my 
observations:


B1. For tolf 5 and -fc1 I for FOR003 = ~4 mRy/bohr  with band gap ~0.1 eV
B2. For tolf 2 and -fc 0.1 forces are almost same. and the band gap is 
still ~0.1ev


B3. Now to reduce the forces less then 0.1mRy/bohr, I reduced the tolf 
to 0.5 and -fc 0.1 - 0.01. I got forces at the desired accurecy but 
the band gap vanished and the system becomes metallic. The 
experimentally reported band gap is 1.2eV

I used RKMAX 8, numk 2000, vxc 19.

What is your suggestion for point number B, why I am getting the 
system metallic?

Can I use this relaxed structure for Phonopy?

Kind regards

Chin S.


For tolf



___
Wien mailing list
Wien@zeus.theochem.tuwien.ac.at
http://zeus.theochem.tuwien.ac.at/mailman/listinfo/wien
SEARCH the MAILING-LIST at:  
http://www.mail-archive.com/wien@zeus.theochem.tuwien.ac.at/index.html


___
Wien mailing list
Wien@zeus.theochem.tuwien.ac.at
http://zeus.theochem.tuwien.ac.at/mailman/listinfo/wien
SEARCH the MAILING-LIST at:  
http://www.mail-archive.com/wien@zeus.theochem.tuwien.ac.at/index.html


Re: [Wien] Charged cells using hybrid functionals

2018-01-23 Thread Xavier Rocquefelte

Dear Marcelo

Another option would be to do a chemical substitution of one element far 
from the probed atom, for which you want to estimate EFG, in such a way 
to compensate the charge.


This will allow you to use hybrid. Indeed, EFG is a local probe and thus 
using a supercell with a reasonnable separation between the probe and 
the compensation charge element (10 angström) you will have relevant 
results.


Cheers

Xavier


Le 24/01/2018 à 04:27, Marcelo Barbosa a écrit :

Dear Gavin,

Thank you very much for your answer.

Since hybrid functional calculations for charged cells is still not 
possible, do you think that a plausible approach to compare the EFG of 
different charged states using hybrid functionals could be done by 
calculating the variation in each EFG orbital component from PBE to 
hybrid and then consider that a similar variation in each component 
could be expected for the charged state, therefore enabling an 
estimation of the EFG for the charged state?


Best regards,
Marcelo

On 12 Jan 2018, at 10:23, Gavin Abo > wrote:


In SRC_lapw0/inputpars.F, it looks like SWITCH3 corresponds with 
putting either KXC or screened exchange-correlation energy and no 
exchange-correlation potential (e.g., EX_SPBE VX_SPBE VX_NONE 
VC_NONE) in case.in0_grr.


The message "Charged cell and SWITCH3 not possible", and "Charged 
cells AND E-tot decomposition not possible" message that you likely 
also have in case.output0_grr, seems to be the same as the post at 
the link you provided.  So yes, it seems that hybrid functional 
calculations for charged cells is still not possible.


On 1/11/2018 12:08 AM, Marcelo Barbosa wrote:

Dear Sirs,

I’m trying to calculate a charged cell using hybrid functionals but 
the following error shows up:


"Charged cell and SWITCH3 not possible”

Looking at the mailing list archive, I only found this link from 
2013 about this subject 
(https://www.mail-archive.com/wien@zeus.theochem.tuwien.ac.at/msg09546.html) 
but I’m not sure if the problem is related since I don’t know what 
SWITCH3 corresponds to.

Is it still not possible to use hybrid functionals for charged cells?

Thank you for your help.

Best regards,
Marcelo

___
Wien mailing list
Wien@zeus.theochem.tuwien.ac.at 
http://zeus.theochem.tuwien.ac.at/mailman/listinfo/wien
SEARCH the MAILING-LIST at: 
 http://www.mail-archive.com/wien@zeus.theochem.tuwien.ac.at/index.html




___
Wien mailing list
Wien@zeus.theochem.tuwien.ac.at
http://zeus.theochem.tuwien.ac.at/mailman/listinfo/wien
SEARCH the MAILING-LIST at:  
http://www.mail-archive.com/wien@zeus.theochem.tuwien.ac.at/index.html


___
Wien mailing list
Wien@zeus.theochem.tuwien.ac.at
http://zeus.theochem.tuwien.ac.at/mailman/listinfo/wien
SEARCH the MAILING-LIST at:  
http://www.mail-archive.com/wien@zeus.theochem.tuwien.ac.at/index.html


Re: [Wien] Magnetocrystalline anisotropy

2018-01-16 Thread Xavier Rocquefelte

Dear Peter and Laurence

Thank you for your replies. Using the following strategy it seems to 
work nicely (preliminary results):


runsp_lapw -eece -p -ec 0.1 -NI

Then for larger EMAX and kmesh without SO we do :

x lapw1 -up -c -orb
x lapw1 -dn -c -orb

And for the different magnetization directions we do:

x lapwso -c -up
x lapw2 -so -up  -c
x lapw2 -so -dn  -c


In other words, we do not put -orb in lapwso. It seems that this option 
was leading to problems depending on the magnetic system.


Best Regards
Xavier

Le 16/01/2018 à 17:57, William Lafargue-dit-Hauret a écrit :


Dear Profs. Blaha and Marks,

Thank you for your advices. I have a question regarding your comment 
on the EECE case.


The resulting effective potential of LDA+U and EECE corrections is 
decomposed in vorb files, but I don't understand why the spin-coupling 
file case.vorbud shouldn't appear in the second case.


I have a second question regarding the reading of these orbital 
potential files. We noticed the orb program read only the real part, 
neglecting the imaginary one (Prof. Marks already noticed it in the 
source code). Due to our energy scale which is tiny, could we expect 
an impact of such approximation ? Considering your last comments, it 
seems not due to the execution of the orb program only in lapw1 and 
not during the perturbative procedure.


Thank you by advance.

All the best,

William Lafargue-dit-Hauret


Le 16/01/2018 à 17:05, Xavier Rocquefelte a écrit :

I was not clear Peter. I clarify the way we proceed.

We do runsp_lapw -eece -p -ec 0.1 -NI

Then for larger EMAX and kmesh without SO we do :

x lapw1 -up -c -orb
x lapw1 -dn -c -orb

And for the different magnetization directions we do:

x lapwso -c -up  -orb
x lapw2 -so -up  -c
x lapw2 -so -dn  -c




Le 16/01/2018 à 16:58, Peter Blaha a écrit :

Hups: If this is true, you are counting the orbital potential twice !

-orb should only be present in the lapwso step.
(And in fact, the lapw1 steps need to be done just once for the 
increased k-mesh; but not when changing the M-direction in case.inso)


Please check the presence of case.vorbud. It must not be there for 
EECE.


You could also test Laurence suggestion, running:

x lapw1 -up/dn -orb
x lapwso -up  (no -orb !!!)

and see of it makes a difference.

On 01/16/2018 04:52 PM, Xavier Rocquefelte wrote:

Dear Peter

You are totally correct. We are doing SO non-selfconsistent by 
using a standard procedure for EECE calculations:


runsp_lapw -eece -p -ec 0.1 -NI

and then we estimate the MAE using this non-SCF procedure :

Increase EMAX in case.in1c - increase kmesh if needed 

x lapw1 -up -c -orb
x lapw1 -dn -c -orb
x lapwso -c -up  -orb
x lapw2 -so -up  -c
x lapw2 -so -dn  -c

Such a procedure was working nicely in previous WIEN2k versions.
Best Regards
Xavier



Le 16/01/2018 à 16:34, Peter Blaha a écrit :

Hallo Xavier,

Looks rather strange.

Eventually I would have expected problems both, in 16.1 and 17.1 
(but not 14.2) due to the off-diagonal density matrices. But this 
should concern ONLY LDA+U, not -eece.


Just to be sure:

I expect you do SO non-selfconsistent, so vorbup/dn(du) files are 
always the same ??  (just running lapwso and lapw2 -so)


Did you make sure that for -eece -so, case.vorbud is NOT present 
(from previous LDA+U).


Peter


On 01/16/2018 02:50 PM, Xavier Rocquefelte wrote:

Here is a document showing the results graphically.

https://filesender.renater.fr/?s=download=8ac3a214-edfa-4894-fa1f-27aba5a5522f 



It really looks like the problem we had before (using bad kmesh).

We test it on two different compounds and in both cases WIEN2k_16 
gives a correct picture and not WIEN2k_17.


We are now comparing the two versions of the code.

Regards

Xavier


Le 16/01/2018 à 12:10, Xavier Rocquefelte a écrit :


Dear All

Finally the problem is not completely solved.

More precisely, when we are doing GGA+SO calculations and using 
a correct kmesh (no temporal symmetry), we obtain a symmetric 
magnetocrystalline anisotropy, namely same MAE along [0 1 0] and 
[0 -1 0].


In contrast, when we are doing GGA+U+SO or EECE+SO with a 
correct kmesh we still obtain non-symmetric MAE, namely MAE 
along [0 1 0] and [0 -1 0] are different.


In addition, the so obtained MAE looks similar to the ones 
obtained in GGA+SO with a bad kmesh (including temporal symmetry).


At this moment, we are checking all the recent modifications in 
SRC_ORB and SRC_LAPW2 related to the manipulation of 
case.vorbup, case.vorbdn and case.vorbud files.


Surprisingly, the EECE+SO calculations in WIEN2k_16 are 
symmetric, while not in WIEN2k_17.


Next soon ... I hope.

Xavier


Le 10/01/2018 à 15:10, Xavier Rocquefelte a écrit :


Dear All

The problem is solved and was related to one stupid human mistake.

It was necessary to generate a kmesh without adding inversion 
(time-inversion symmetry).

Indeed, as mentionned in the userguide when using kgen program:

# *"add inve

Re: [Wien] Magnetocrystalline anisotropy

2018-01-16 Thread Xavier Rocquefelte

I was not clear Peter. I clarify the way we proceed.

We do runsp_lapw -eece -p -ec 0.1 -NI

Then for larger EMAX and kmesh without SO we do :

x lapw1 -up -c -orb
x lapw1 -dn -c -orb

And for the different magnetization directions we do:

x lapwso -c -up  -orb
x lapw2 -so -up  -c
x lapw2 -so -dn  -c




Le 16/01/2018 à 16:58, Peter Blaha a écrit :

Hups: If this is true, you are counting the orbital potential twice !

-orb should only be present in the lapwso step.
(And in fact, the lapw1 steps need to be done just once for the 
increased k-mesh; but not when changing the M-direction in case.inso)


Please check the presence of case.vorbud. It must not be there for EECE.

You could also test Laurence suggestion, running:

x lapw1 -up/dn -orb
x lapwso -up  (no -orb !!!)

and see of it makes a difference.

On 01/16/2018 04:52 PM, Xavier Rocquefelte wrote:

Dear Peter

You are totally correct. We are doing SO non-selfconsistent by using 
a standard procedure for EECE calculations:


runsp_lapw -eece -p -ec 0.1 -NI

and then we estimate the MAE using this non-SCF procedure :

Increase EMAX in case.in1c - increase kmesh if needed 

x lapw1 -up -c -orb
x lapw1 -dn -c -orb
x lapwso -c -up  -orb
x lapw2 -so -up  -c
x lapw2 -so -dn  -c

Such a procedure was working nicely in previous WIEN2k versions.
Best Regards
Xavier



Le 16/01/2018 à 16:34, Peter Blaha a écrit :

Hallo Xavier,

Looks rather strange.

Eventually I would have expected problems both, in 16.1 and 17.1 
(but not 14.2) due to the off-diagonal density matrices. But this 
should concern ONLY LDA+U, not -eece.


Just to be sure:

I expect you do SO non-selfconsistent, so vorbup/dn(du) files are 
always the same ??  (just running lapwso and lapw2 -so)


Did you make sure that for -eece -so, case.vorbud is NOT present 
(from previous LDA+U).


Peter


On 01/16/2018 02:50 PM, Xavier Rocquefelte wrote:

Here is a document showing the results graphically.

https://filesender.renater.fr/?s=download=8ac3a214-edfa-4894-fa1f-27aba5a5522f 



It really looks like the problem we had before (using bad kmesh).

We test it on two different compounds and in both cases WIEN2k_16 
gives a correct picture and not WIEN2k_17.


We are now comparing the two versions of the code.

Regards

Xavier


Le 16/01/2018 à 12:10, Xavier Rocquefelte a écrit :


Dear All

Finally the problem is not completely solved.

More precisely, when we are doing GGA+SO calculations and using a 
correct kmesh (no temporal symmetry), we obtain a symmetric 
magnetocrystalline anisotropy, namely same MAE along [0 1 0] and 
[0 -1 0].


In contrast, when we are doing GGA+U+SO or EECE+SO with a correct 
kmesh we still obtain non-symmetric MAE, namely MAE along [0 1 0] 
and [0 -1 0] are different.


In addition, the so obtained MAE looks similar to the ones 
obtained in GGA+SO with a bad kmesh (including temporal symmetry).


At this moment, we are checking all the recent modifications in 
SRC_ORB and SRC_LAPW2 related to the manipulation of case.vorbup, 
case.vorbdn and case.vorbud files.


Surprisingly, the EECE+SO calculations in WIEN2k_16 are symmetric, 
while not in WIEN2k_17.


Next soon ... I hope.

Xavier


Le 10/01/2018 à 15:10, Xavier Rocquefelte a écrit :


Dear All

The problem is solved and was related to one stupid human mistake.

It was necessary to generate a kmesh without adding inversion 
(time-inversion symmetry).

Indeed, as mentionned in the userguide when using kgen program:

# *"add inversion" ?* This is asked only when inversion is NOT 
present.


  * Say *"YES"* in all cases except when you do *spin-polarized
(magnetic) calculations WITH spin-orbit coupling * (this breaks
time-inversion symmetry and thus one MUST NOT add inversion
symmetry (eigenvalues at +k and -k may be different).

If you properly generate the kmesh for the spin-orbit 
calculations by doing : x kgen -fbz, then you obtain a symmetric 
magnetic anisotrop. In conclusion the asymmetry I obtained was 
due to an improper definition of the kmesh (adding artificially 
time-inversion).


I want to thank all the participants who answered to my question. 
It was essential to identify such a mistake which has a huge 
impact on the results.


Best wishes

Xavier



Le 10/01/2018 à 10:47, Xavier Rocquefelte a écrit :

Dear Lyudmila

The fact we have a small angle with axes is expected (also 
observed experimentally). It is related to the monoclinic 
symmetry of the system which permits it. However, you gave me an 
idea that I will test now and comment soon ;)


Cheers

Xavier

Le 10/01/2018 à 10:40, Lyudmila a écrit :

10.01.2018 13:36, Lyudmila wrote:
I see in the FM calculation also a slightly non-symmetric 
curve, isn't it?


I meant the small angle with axes.


Best wishes
Lyudmila Dobysheva

___
Wien mailing list
Wien@zeus.theochem.tuwien.ac.at
http://zeus.theochem.tuwien.ac.at/mailman/listinfo/wien
SEARCH the MAILING-LIST 

Re: [Wien] Magnetocrystalline anisotropy

2018-01-16 Thread Xavier Rocquefelte

OK we will check it. In EECE we have no case.vorbud file (empty file).


Le 16/01/2018 à 16:58, Peter Blaha a écrit :

Hups: If this is true, you are counting the orbital potential twice !

-orb should only be present in the lapwso step.
(And in fact, the lapw1 steps need to be done just once for the 
increased k-mesh; but not when changing the M-direction in case.inso)


Please check the presence of case.vorbud. It must not be there for EECE.

You could also test Laurence suggestion, running:

x lapw1 -up/dn -orb
x lapwso -up  (no -orb !!!)

and see of it makes a difference.

On 01/16/2018 04:52 PM, Xavier Rocquefelte wrote:

Dear Peter

You are totally correct. We are doing SO non-selfconsistent by using 
a standard procedure for EECE calculations:


runsp_lapw -eece -p -ec 0.1 -NI

and then we estimate the MAE using this non-SCF procedure :

Increase EMAX in case.in1c - increase kmesh if needed 

x lapw1 -up -c -orb
x lapw1 -dn -c -orb
x lapwso -c -up  -orb
x lapw2 -so -up  -c
x lapw2 -so -dn  -c

Such a procedure was working nicely in previous WIEN2k versions.
Best Regards
Xavier



Le 16/01/2018 à 16:34, Peter Blaha a écrit :

Hallo Xavier,

Looks rather strange.

Eventually I would have expected problems both, in 16.1 and 17.1 
(but not 14.2) due to the off-diagonal density matrices. But this 
should concern ONLY LDA+U, not -eece.


Just to be sure:

I expect you do SO non-selfconsistent, so vorbup/dn(du) files are 
always the same ??  (just running lapwso and lapw2 -so)


Did you make sure that for -eece -so, case.vorbud is NOT present 
(from previous LDA+U).


Peter


On 01/16/2018 02:50 PM, Xavier Rocquefelte wrote:

Here is a document showing the results graphically.

https://filesender.renater.fr/?s=download=8ac3a214-edfa-4894-fa1f-27aba5a5522f 



It really looks like the problem we had before (using bad kmesh).

We test it on two different compounds and in both cases WIEN2k_16 
gives a correct picture and not WIEN2k_17.


We are now comparing the two versions of the code.

Regards

Xavier


Le 16/01/2018 à 12:10, Xavier Rocquefelte a écrit :


Dear All

Finally the problem is not completely solved.

More precisely, when we are doing GGA+SO calculations and using a 
correct kmesh (no temporal symmetry), we obtain a symmetric 
magnetocrystalline anisotropy, namely same MAE along [0 1 0] and 
[0 -1 0].


In contrast, when we are doing GGA+U+SO or EECE+SO with a correct 
kmesh we still obtain non-symmetric MAE, namely MAE along [0 1 0] 
and [0 -1 0] are different.


In addition, the so obtained MAE looks similar to the ones 
obtained in GGA+SO with a bad kmesh (including temporal symmetry).


At this moment, we are checking all the recent modifications in 
SRC_ORB and SRC_LAPW2 related to the manipulation of case.vorbup, 
case.vorbdn and case.vorbud files.


Surprisingly, the EECE+SO calculations in WIEN2k_16 are symmetric, 
while not in WIEN2k_17.


Next soon ... I hope.

Xavier


Le 10/01/2018 à 15:10, Xavier Rocquefelte a écrit :


Dear All

The problem is solved and was related to one stupid human mistake.

It was necessary to generate a kmesh without adding inversion 
(time-inversion symmetry).

Indeed, as mentionned in the userguide when using kgen program:

# *"add inversion" ?* This is asked only when inversion is NOT 
present.


  * Say *"YES"* in all cases except when you do *spin-polarized
(magnetic) calculations WITH spin-orbit coupling * (this breaks
time-inversion symmetry and thus one MUST NOT add inversion
symmetry (eigenvalues at +k and -k may be different).

If you properly generate the kmesh for the spin-orbit 
calculations by doing : x kgen -fbz, then you obtain a symmetric 
magnetic anisotrop. In conclusion the asymmetry I obtained was 
due to an improper definition of the kmesh (adding artificially 
time-inversion).


I want to thank all the participants who answered to my question. 
It was essential to identify such a mistake which has a huge 
impact on the results.


Best wishes

Xavier



Le 10/01/2018 à 10:47, Xavier Rocquefelte a écrit :

Dear Lyudmila

The fact we have a small angle with axes is expected (also 
observed experimentally). It is related to the monoclinic 
symmetry of the system which permits it. However, you gave me an 
idea that I will test now and comment soon ;)


Cheers

Xavier

Le 10/01/2018 à 10:40, Lyudmila a écrit :

10.01.2018 13:36, Lyudmila wrote:
I see in the FM calculation also a slightly non-symmetric 
curve, isn't it?


I meant the small angle with axes.


Best wishes
Lyudmila Dobysheva

___
Wien mailing list
Wien@zeus.theochem.tuwien.ac.at
http://zeus.theochem.tuwien.ac.at/mailman/listinfo/wien
SEARCH the MAILING-LIST at: 
http://www.mail-archive.com/wien@zeus.theochem.tuwien.ac.at/index.html 







___
Wien mailing list
Wien@zeus.theochem.tuwien.ac.at
http://zeus.theochem.tuwien.ac.at/mailman/listinfo/wie

Re: [Wien] Magnetocrystalline anisotropy

2018-01-16 Thread Xavier Rocquefelte

Dear Peter

You are totally correct. We are doing SO non-selfconsistent by using a 
standard procedure for EECE calculations:


runsp_lapw -eece -p -ec 0.1 -NI

and then we estimate the MAE using this non-SCF procedure :

Increase EMAX in case.in1c - increase kmesh if needed 

x lapw1 -up -c -orb
x lapw1 -dn -c -orb
x lapwso -c -up  -orb
x lapw2 -so -up  -c
x lapw2 -so -dn  -c

Such a procedure was working nicely in previous WIEN2k versions.
Best Regards
Xavier



Le 16/01/2018 à 16:34, Peter Blaha a écrit :

Hallo Xavier,

Looks rather strange.

Eventually I would have expected problems both, in 16.1 and 17.1 (but 
not 14.2) due to the off-diagonal density matrices. But this should 
concern ONLY LDA+U, not -eece.


Just to be sure:

I expect you do SO non-selfconsistent, so vorbup/dn(du) files are 
always the same ??  (just running lapwso and lapw2 -so)


Did you make sure that for -eece -so, case.vorbud is NOT present (from 
previous LDA+U).


Peter


On 01/16/2018 02:50 PM, Xavier Rocquefelte wrote:

Here is a document showing the results graphically.

https://filesender.renater.fr/?s=download=8ac3a214-edfa-4894-fa1f-27aba5a5522f 



It really looks like the problem we had before (using bad kmesh).

We test it on two different compounds and in both cases WIEN2k_16 
gives a correct picture and not WIEN2k_17.


We are now comparing the two versions of the code.

Regards

Xavier


Le 16/01/2018 à 12:10, Xavier Rocquefelte a écrit :


Dear All

Finally the problem is not completely solved.

More precisely, when we are doing GGA+SO calculations and using a 
correct kmesh (no temporal symmetry), we obtain a symmetric 
magnetocrystalline anisotropy, namely same MAE along [0 1 0] and [0 
-1 0].


In contrast, when we are doing GGA+U+SO or EECE+SO with a correct 
kmesh we still obtain non-symmetric MAE, namely MAE along [0 1 0] 
and [0 -1 0] are different.


In addition, the so obtained MAE looks similar to the ones obtained 
in GGA+SO with a bad kmesh (including temporal symmetry).


At this moment, we are checking all the recent modifications in 
SRC_ORB and SRC_LAPW2 related to the manipulation of case.vorbup, 
case.vorbdn and case.vorbud files.


Surprisingly, the EECE+SO calculations in WIEN2k_16 are symmetric, 
while not in WIEN2k_17.


Next soon ... I hope.

Xavier


Le 10/01/2018 à 15:10, Xavier Rocquefelte a écrit :


Dear All

The problem is solved and was related to one stupid human mistake.

It was necessary to generate a kmesh without adding inversion 
(time-inversion symmetry).

Indeed, as mentionned in the userguide when using kgen program:

# *"add inversion" ?* This is asked only when inversion is NOT 
present.


  * Say *"YES"* in all cases except when you do *spin-polarized
(magnetic) calculations WITH spin-orbit coupling * (this breaks
time-inversion symmetry and thus one MUST NOT add inversion
symmetry (eigenvalues at +k and -k may be different).

If you properly generate the kmesh for the spin-orbit calculations 
by doing : x kgen -fbz, then you obtain a symmetric magnetic 
anisotrop. In conclusion the asymmetry I obtained was due to an 
improper definition of the kmesh (adding artificially time-inversion).


I want to thank all the participants who answered to my question. 
It was essential to identify such a mistake which has a huge impact 
on the results.


Best wishes

Xavier



Le 10/01/2018 à 10:47, Xavier Rocquefelte a écrit :

Dear Lyudmila

The fact we have a small angle with axes is expected (also 
observed experimentally). It is related to the monoclinic symmetry 
of the system which permits it. However, you gave me an idea that 
I will test now and comment soon ;)


Cheers

Xavier

Le 10/01/2018 à 10:40, Lyudmila a écrit :

10.01.2018 13:36, Lyudmila wrote:
I see in the FM calculation also a slightly non-symmetric curve, 
isn't it?


I meant the small angle with axes.


Best wishes
Lyudmila Dobysheva

___
Wien mailing list
Wien@zeus.theochem.tuwien.ac.at
http://zeus.theochem.tuwien.ac.at/mailman/listinfo/wien
SEARCH the MAILING-LIST at: 
http://www.mail-archive.com/wien@zeus.theochem.tuwien.ac.at/index.html 







___
Wien mailing list
Wien@zeus.theochem.tuwien.ac.at
http://zeus.theochem.tuwien.ac.at/mailman/listinfo/wien
SEARCH the MAILING-LIST 
at:http://www.mail-archive.com/wien@zeus.theochem.tuwien.ac.at/index.html




___
Wien mailing list
Wien@zeus.theochem.tuwien.ac.at
http://zeus.theochem.tuwien.ac.at/mailman/listinfo/wien
SEARCH the MAILING-LIST 
at:http://www.mail-archive.com/wien@zeus.theochem.tuwien.ac.at/index.html




___
Wien mailing list
Wien@zeus.theochem.tuwien.ac.at
http://zeus.theochem.tuwien.ac.at/mailman/listinfo/wien
SEARCH the MAILING-LIST at: 
http://www.mail-archive.com/wien@zeus.theochem.tuw

Re: [Wien] Magnetocrystalline anisotropy

2018-01-16 Thread Xavier Rocquefelte

Dear Laurence

Here is the point. Our results show that in version 17 a problem occurs 
related to the time-reversal, which appears only if we do GGA+U or EECE.


a) The calculations we showed are already done in P1. Indeed, I wanted 
to avoid any problems related to symmetry.


b) we (me and my PhD student William Lafargue-dit-Hauret) were exactly 
working along this line ... our preliminary results are not consistent.


c) we didn't try this option ... we will try

Thank you for your quick reply Laurence

Best Regards

Xavier



Le 16/01/2018 à 15:03, Laurence Marks a écrit :

I may annoy Peter with the comment below, so it goes

Personally I have some reservations about the consistency between the
orbital potential and time-reversal operations in -so. Three possible
tests:

a) Reduce the symmetry to P1 and run. While this is slower, I believe
your test case is small so this should not matter.
b) Hack the runsp_lapw script so that -orb is applied to lapw1, not in lapwso
c) Run without the up/dn component (-noorbud)

On Tue, Jan 16, 2018 at 7:50 AM, Xavier Rocquefelte
<xavier.rocquefe...@univ-rennes1.fr> wrote:

Here is a document showing the results graphically.

https://filesender.renater.fr/?s=download=8ac3a214-edfa-4894-fa1f-27aba5a5522f

It really looks like the problem we had before (using bad kmesh).

We test it on two different compounds and in both cases WIEN2k_16 gives a
correct picture and not WIEN2k_17.

We are now comparing the two versions of the code.

Regards

Xavier


Le 16/01/2018 à 12:10, Xavier Rocquefelte a écrit :

Dear All

Finally the problem is not completely solved.

More precisely, when we are doing GGA+SO calculations and using a correct
kmesh (no temporal symmetry), we obtain a symmetric magnetocrystalline
anisotropy, namely same MAE along [0 1 0] and [0 -1 0].

In contrast, when we are doing GGA+U+SO or EECE+SO with a correct kmesh we
still obtain non-symmetric MAE, namely MAE along [0 1 0] and [0 -1 0] are
different.

In addition, the so obtained MAE looks similar to the ones obtained in
GGA+SO with a bad kmesh (including temporal symmetry).

At this moment, we are checking all the recent modifications in SRC_ORB and
SRC_LAPW2 related to the manipulation of case.vorbup, case.vorbdn and
case.vorbud files.

Surprisingly, the EECE+SO calculations in WIEN2k_16 are symmetric, while not
in WIEN2k_17.

Next soon ... I hope.

Xavier


Le 10/01/2018 à 15:10, Xavier Rocquefelte a écrit :

Dear All

The problem is solved and was related to one stupid human mistake.

It was necessary to generate a kmesh without adding inversion
(time-inversion symmetry).
Indeed, as mentionned in the userguide when using kgen program:

"add inversion" ? This is asked only when inversion is NOT present.

Say "YES" in all cases except when you do spin-polarized (magnetic)
calculations WITH spin-orbit coupling (this breaks time-inversion symmetry
and thus one MUST NOT add inversion symmetry (eigenvalues at +k and -k may
be different).

If you properly generate the kmesh for the spin-orbit calculations by doing
: x kgen -fbz, then you obtain a symmetric magnetic anisotrop. In conclusion
the asymmetry I obtained was due to an improper definition of the kmesh
(adding artificially time-inversion).

I want to thank all the participants who answered to my question. It was
essential to identify such a mistake which has a huge impact on the results.

Best wishes

Xavier



Le 10/01/2018 à 10:47, Xavier Rocquefelte a écrit :

Dear Lyudmila

The fact we have a small angle with axes is expected (also observed
experimentally). It is related to the monoclinic symmetry of the system
which permits it. However, you gave me an idea that I will test now and
comment soon ;)

Cheers

Xavier

Le 10/01/2018 à 10:40, Lyudmila a écrit :

10.01.2018 13:36, Lyudmila wrote:

I see in the FM calculation also a slightly non-symmetric curve, isn't it?


I meant the small angle with axes.

Best wishes
Lyudmila Dobysheva

___
Wien mailing list
Wien@zeus.theochem.tuwien.ac.at
http://zeus.theochem.tuwien.ac.at/mailman/listinfo/wien
SEARCH the MAILING-LIST at:
http://www.mail-archive.com/wien@zeus.theochem.tuwien.ac.at/index.html





___
Wien mailing list
Wien@zeus.theochem.tuwien.ac.at
http://zeus.theochem.tuwien.ac.at/mailman/listinfo/wien
SEARCH the MAILING-LIST at:
http://www.mail-archive.com/wien@zeus.theochem.tuwien.ac.at/index.html




___
Wien mailing list
Wien@zeus.theochem.tuwien.ac.at
http://zeus.theochem.tuwien.ac.at/mailman/listinfo/wien
SEARCH the MAILING-LIST at:
http://www.mail-archive.com/wien@zeus.theochem.tuwien.ac.at/index.html







___
Wien mailing list
Wien@zeus.theochem.tuwien.ac.at
http://zeus.theochem.tuwien.ac.at/mailman/listinfo/wien
SEARCH the MAILING-LIST at:  
http://www.mail-archive.com/wien@zeus.theochem.tuwien.ac.at/index.html


Re: [Wien] Magnetocrystalline anisotropy

2018-01-16 Thread Xavier Rocquefelte

Here is a document showing the results graphically.

https://filesender.renater.fr/?s=download=8ac3a214-edfa-4894-fa1f-27aba5a5522f

It really looks like the problem we had before (using bad kmesh).

We test it on two different compounds and in both cases WIEN2k_16 gives 
a correct picture and not WIEN2k_17.


We are now comparing the two versions of the code.

Regards

Xavier


Le 16/01/2018 à 12:10, Xavier Rocquefelte a écrit :


Dear All

Finally the problem is not completely solved.

More precisely, when we are doing GGA+SO calculations and using a 
correct kmesh (no temporal symmetry), we obtain a symmetric 
magnetocrystalline anisotropy, namely same MAE along [0 1 0] and [0 -1 
0].


In contrast, when we are doing GGA+U+SO or EECE+SO with a correct 
kmesh we still obtain non-symmetric MAE, namely MAE along [0 1 0] and 
[0 -1 0] are different.


In addition, the so obtained MAE looks similar to the ones obtained in 
GGA+SO with a bad kmesh (including temporal symmetry).


At this moment, we are checking all the recent modifications in 
SRC_ORB and SRC_LAPW2 related to the manipulation of case.vorbup, 
case.vorbdn and case.vorbud files.


Surprisingly, the EECE+SO calculations in WIEN2k_16 are symmetric, 
while not in WIEN2k_17.


Next soon ... I hope.

Xavier


Le 10/01/2018 à 15:10, Xavier Rocquefelte a écrit :


Dear All

The problem is solved and was related to one stupid human mistake.

It was necessary to generate a kmesh without adding inversion 
(time-inversion symmetry).

Indeed, as mentionned in the userguide when using kgen program:

# *"add inversion" ?* This is asked only when inversion is NOT present.

  * Say *"YES"* in all cases except when you do *spin-polarized
(magnetic) calculations WITH spin-orbit coupling * (this breaks
time-inversion symmetry and thus one MUST NOT add inversion
symmetry (eigenvalues at +k and -k may be different).

If you properly generate the kmesh for the spin-orbit calculations by 
doing : x kgen -fbz, then you obtain a symmetric magnetic anisotrop. 
In conclusion the asymmetry I obtained was due to an improper 
definition of the kmesh (adding artificially time-inversion).


I want to thank all the participants who answered to my question. It 
was essential to identify such a mistake which has a huge impact on 
the results.


Best wishes

Xavier



Le 10/01/2018 à 10:47, Xavier Rocquefelte a écrit :

Dear Lyudmila

The fact we have a small angle with axes is expected (also observed 
experimentally). It is related to the monoclinic symmetry of the 
system which permits it. However, you gave me an idea that I will 
test now and comment soon ;)


Cheers

Xavier

Le 10/01/2018 à 10:40, Lyudmila a écrit :

10.01.2018 13:36, Lyudmila wrote:
I see in the FM calculation also a slightly non-symmetric curve, 
isn't it?


I meant the small angle with axes.


Best wishes
Lyudmila Dobysheva

___
Wien mailing list
Wien@zeus.theochem.tuwien.ac.at
http://zeus.theochem.tuwien.ac.at/mailman/listinfo/wien
SEARCH the MAILING-LIST at: 
http://www.mail-archive.com/wien@zeus.theochem.tuwien.ac.at/index.html






___
Wien mailing list
Wien@zeus.theochem.tuwien.ac.at
http://zeus.theochem.tuwien.ac.at/mailman/listinfo/wien
SEARCH the MAILING-LIST 
at:http://www.mail-archive.com/wien@zeus.theochem.tuwien.ac.at/index.html




___
Wien mailing list
Wien@zeus.theochem.tuwien.ac.at
http://zeus.theochem.tuwien.ac.at/mailman/listinfo/wien
SEARCH the MAILING-LIST at:  
http://www.mail-archive.com/wien@zeus.theochem.tuwien.ac.at/index.html


___
Wien mailing list
Wien@zeus.theochem.tuwien.ac.at
http://zeus.theochem.tuwien.ac.at/mailman/listinfo/wien
SEARCH the MAILING-LIST at:  
http://www.mail-archive.com/wien@zeus.theochem.tuwien.ac.at/index.html


Re: [Wien] Magnetocrystalline anisotropy

2018-01-16 Thread Xavier Rocquefelte

Dear All

Finally the problem is not completely solved.

More precisely, when we are doing GGA+SO calculations and using a 
correct kmesh (no temporal symmetry), we obtain a symmetric 
magnetocrystalline anisotropy, namely same MAE along [0 1 0] and [0 -1 0].


In contrast, when we are doing GGA+U+SO or EECE+SO with a correct kmesh 
we still obtain non-symmetric MAE, namely MAE along [0 1 0] and [0 -1 0] 
are different.


In addition, the so obtained MAE looks similar to the ones obtained in 
GGA+SO with a bad kmesh (including temporal symmetry).


At this moment, we are checking all the recent modifications in SRC_ORB 
and SRC_LAPW2 related to the manipulation of case.vorbup, case.vorbdn 
and case.vorbud files.


Surprisingly, the EECE+SO calculations in WIEN2k_16 are symmetric, while 
not in WIEN2k_17.


Next soon ... I hope.

Xavier


Le 10/01/2018 à 15:10, Xavier Rocquefelte a écrit :


Dear All

The problem is solved and was related to one stupid human mistake.

It was necessary to generate a kmesh without adding inversion 
(time-inversion symmetry).

Indeed, as mentionned in the userguide when using kgen program:

# *"add inversion" ?* This is asked only when inversion is NOT present.

  * Say *"YES"* in all cases except when you do *spin-polarized
(magnetic) calculations WITH spin-orbit coupling * (this breaks
time-inversion symmetry and thus one MUST NOT add inversion
symmetry (eigenvalues at +k and -k may be different).

If you properly generate the kmesh for the spin-orbit calculations by 
doing : x kgen -fbz, then you obtain a symmetric magnetic anisotrop. 
In conclusion the asymmetry I obtained was due to an improper 
definition of the kmesh (adding artificially time-inversion).


I want to thank all the participants who answered to my question. It 
was essential to identify such a mistake which has a huge impact on 
the results.


Best wishes

Xavier



Le 10/01/2018 à 10:47, Xavier Rocquefelte a écrit :

Dear Lyudmila

The fact we have a small angle with axes is expected (also observed 
experimentally). It is related to the monoclinic symmetry of the 
system which permits it. However, you gave me an idea that I will 
test now and comment soon ;)


Cheers

Xavier

Le 10/01/2018 à 10:40, Lyudmila a écrit :

10.01.2018 13:36, Lyudmila wrote:
I see in the FM calculation also a slightly non-symmetric curve, 
isn't it?


I meant the small angle with axes.


Best wishes
Lyudmila Dobysheva

___
Wien mailing list
Wien@zeus.theochem.tuwien.ac.at
http://zeus.theochem.tuwien.ac.at/mailman/listinfo/wien
SEARCH the MAILING-LIST at: 
http://www.mail-archive.com/wien@zeus.theochem.tuwien.ac.at/index.html






___
Wien mailing list
Wien@zeus.theochem.tuwien.ac.at
http://zeus.theochem.tuwien.ac.at/mailman/listinfo/wien
SEARCH the MAILING-LIST at:  
http://www.mail-archive.com/wien@zeus.theochem.tuwien.ac.at/index.html


___
Wien mailing list
Wien@zeus.theochem.tuwien.ac.at
http://zeus.theochem.tuwien.ac.at/mailman/listinfo/wien
SEARCH the MAILING-LIST at:  
http://www.mail-archive.com/wien@zeus.theochem.tuwien.ac.at/index.html


Re: [Wien] Magnetocrystalline anisotropy

2018-01-10 Thread Xavier Rocquefelte

Dear Lyudmila

The fact we have a small angle with axes is expected (also observed 
experimentally). It is related to the monoclinic symmetry of the system 
which permits it. However, you gave me an idea that I will test now and 
comment soon ;)


Cheers

Xavier

Le 10/01/2018 à 10:40, Lyudmila a écrit :

10.01.2018 13:36, Lyudmila wrote:
I see in the FM calculation also a slightly non-symmetric curve, 
isn't it?


I meant the small angle with axes.


Best wishes
Lyudmila Dobysheva

___
Wien mailing list
Wien@zeus.theochem.tuwien.ac.at
http://zeus.theochem.tuwien.ac.at/mailman/listinfo/wien
SEARCH the MAILING-LIST at: 
http://www.mail-archive.com/wien@zeus.theochem.tuwien.ac.at/index.html


___
Wien mailing list
Wien@zeus.theochem.tuwien.ac.at
http://zeus.theochem.tuwien.ac.at/mailman/listinfo/wien
SEARCH the MAILING-LIST at:  
http://www.mail-archive.com/wien@zeus.theochem.tuwien.ac.at/index.html


Re: [Wien] Magnetocrystalline anisotropy

2018-01-09 Thread Xavier Rocquefelte

Dear Jaroslav

Thank you for your quick and detailled reply.

It seems to me that there is one important difference between my 
calculations and yours.


Indeed, my calculations are done using P1 symmetry. In addition, the 
unit cell for FM and AFM1 magnetic orders are exactly the same. I only 
modify the case.inst file. Thus if we have a problem of local rotation 
matrix it should appear in both cases. However, it seems that the 
problem only appear in the case of AFM order calculation. I never had 
curious MAE for FM calculations.


I should admit that when estimating MAE for FM we have energies larger 
of one order of magnitude compared to AFM ones.
However, as shown in the previous document we have no noise in our 
calculated MAE values in both FM and AFM cases, suggesting that these 
calculations are already converged.


The problem seems to be still opened to suggestions but I will look at 
your idea in more details in the afternoon.


Thank you again

Cheers

Xavier





Le 09/01/2018 à 10:49, Jaroslav Hamrle a écrit :

Dear Xavier,

your problem somewhat resembles me my problem I had when calculating 
magnetic linear dochroism (MLD) on bcc Fe. The similarity is that we 
both want to see small changes in electronic structure when rotating 
magnetic field direction.


What help me:
1) run fine convergence criteria, such as runsp_lapw -p -cc 0.01 
-ec 0.01



2) as suggested Prof. Blaha, it was important to increase k-mesh (in 
my case up to 100), and apply fine BZ integration (TEMP or TEMPS) with 
small value as 0.001, not default TETRA.

for example change case.in2 by using command
sed '3s/^/TEMP0.001   /' $file.in2 > 
$file.in2_TEMPnew

more here"
https://www.mail-archive.com/wien@zeus.theochem.tuwien.ac.at/msg16815.html 

https://www.mail-archive.com/wien@zeus.theochem.tuwien.ac.at/msg16844.html 




3) for some magnetization direction, I had problem with either wrong, 
either suspicious values of local rotation matrix,
https://www.mail-archive.com/wien@zeus.theochem.tuwien.ac.at/msg16894.html 



Problem was that for some external magnetization directions, the local 
direction of magnetization was not [001].
In one case (external M along [-111]), the local magnetization 
direction was [-0.94281 0   -0.3] which I think is not correct, 
and MLD was wrong too.
In some case, local magnetization direction was along x or along y, 
which I dont know if it is correct. On one hand, eigenenergies agreed 
perfectly, but anyway I saw small change in MLD in those cases.


But as a blind suggestion for you, try if local rotation matrices are 
correct. Namely try if

mag_glob*R = mag_loc
where mag_glob is your (external i.e. in global coordinates) 
magnetization direction, R is local rotation matrix for each atom (can 
be found in case.struct or case.outsymso) and mag_loc is local 
magnetization direction, which in my (maybe naive and wrong) 
understanding should be [001].


Hoping it helps
With my best regards

Jaroslav


On 09/01/18 09:44, Xavier Rocquefelte wrote:

Dear Colleagues

I recently obtained a surprising result concerning the calculation of 
the magnetocrystalline anisotropy energy (MAE) of SeCuO3.


This compound has a monoclinic symmetry (SG. P21/n) and is known to 
be antiferromagnetically ordered at low temperature.


Here I provide the results obtained for two magnetic orders, named FM 
and AFM1 (see attached document) :


https://filesender.renater.fr/?s=download=1da93a22-9592-3a7e-ba2e-1533fcae45d2 



These calculations have been done using WIEN2k_17, GGA = PBE, RKMAX = 
6, kmesh = 5 4 4 and in P1 symmetry. The results are the same using 
RKMAX = 7.


The AFM1 order is the more stable one, as expected.

However, as shown in the document the MAE of AFM1 order is not 
symmetric, which is not expected. In contrast the MAE for FM order is 
symmetric.


Based on the recent discussion "zigzag potential", it seems to me 
that the AFM1 MAE should be symmetric, because the magnetic moment is 
a pseudo-vector. Is it possible that the present problem is related 
to the fact that in the present implementation of the spin-orbit 
coupling we neglect the off-diagonal terms? Do you have any idea 
about the problem we are facing? Does someone observe such unusual 
MAE for other systems?


Best Regards

Xavier

___
Wien mailing list
Wien@zeus.theochem.tuwien.ac.at
http://zeus.theochem.tuwien.ac.at/mailman/listinfo/wien
SEARCH the MAILING-LIST at: 
http://www.mail-archive.com/wien@zeus.theochem.tuwien.ac.at/index.html





___
Wien mailing list
Wien@zeus.theochem.tuwien.ac.at
http://zeus.theochem.tuwien.ac.at/mailman/listinfo/wien
SEARCH the MAILING-LIST at:  
http://www.mail-archive.com/wien@zeus.theochem.tuwien.ac.at/index.html


[Wien] Magnetocrystalline anisotropy

2018-01-09 Thread Xavier Rocquefelte

Dear Colleagues

I recently obtained a surprising result concerning the calculation of 
the magnetocrystalline anisotropy energy (MAE) of SeCuO3.


This compound has a monoclinic symmetry (SG. P21/n) and is known to be 
antiferromagnetically ordered at low temperature.


Here I provide the results obtained for two magnetic orders, named FM 
and AFM1 (see attached document) :


https://filesender.renater.fr/?s=download=1da93a22-9592-3a7e-ba2e-1533fcae45d2

These calculations have been done using WIEN2k_17, GGA = PBE, RKMAX = 6, 
kmesh = 5 4 4 and in P1 symmetry. The results are the same using RKMAX = 7.


The AFM1 order is the more stable one, as expected.

However, as shown in the document the MAE of AFM1 order is not 
symmetric, which is not expected. In contrast the MAE for FM order is 
symmetric.


Based on the recent discussion "zigzag potential", it seems to me that 
the AFM1 MAE should be symmetric, because the magnetic moment is a 
pseudo-vector. Is it possible that the present problem is related to the 
fact that in the present implementation of the spin-orbit coupling we 
neglect the off-diagonal terms? Do you have any idea about the problem 
we are facing? Does someone observe such unusual MAE for other systems?


Best Regards

Xavier

___
Wien mailing list
Wien@zeus.theochem.tuwien.ac.at
http://zeus.theochem.tuwien.ac.at/mailman/listinfo/wien
SEARCH the MAILING-LIST at:  
http://www.mail-archive.com/wien@zeus.theochem.tuwien.ac.at/index.html


Re: [Wien] nband should be an integer

2018-01-07 Thread Xavier Rocquefelte

Indeed, it seems that there is an error in the read_input.f file of SRC_hf.

If SCREENED if TRUE everything is fine, but not when SCREENED is FALSE, 
because in this case the program does not read SMU.


Here I did a simple modification (in red) of read_input.f :

...

  read(4,*) screened
  if ((screened .neqv. .true.) .and. (screened .neqv. .false.)) &
stop 'error in case.inhf: screening should be true or false'
  if (screened .eqv. .true.) then
read(4,*) smu
if ((smu .lt. 1d-4) .or. (smu .gt. 5d0)) &
  stop 'error in case.inhf: to avoid numerical instabilities, &
  do not use lambda smaller than 0.0001 or larger than 5'
  elseif (screened .eqv. .false.) then
*read(4,*) smu*
smu = 0d0
  endif



It should solve your problem.

Cheers

Xavier


Le 07/01/2018 à 11:25, 24h Nhảm a écrit :


Dear all,
I am trying to do a full Hybrid calculation for ZnSe.
1. I calculated pbe and save_lapw

2. i executed init_hf_lapw

3. I got nband from scf file (:BAN)
:BAN00018: 18 -0.234925 -0.215645 1.
:BAN00019: 19 -0.217062 -0.211819 1.
:BAN00020: 20 -0.215708 -0.210135 1.
:BAN00021: 21 -0.213675 -0.201748 1.
:BAN00022: 22 -0.213675 -0.191175 1.
:BAN00023: 23 -0.135689 0.023997 1.
:BAN00024: 24 -0.127789 0.179782 1.
:BAN00025: 25 0.008400 0.210094 1.
:BAN00026: 26 0.025266 0.244240 1.
:BAN00027: 27 0.091871 0.244240 1.
:BAN00028: 28 0.128341 0.246515 1.
:BAN00029: 29 0.350172 0.576997 0.
:BAN00030: 30 0.409942 0.595638 0.
:BAN00031: 31 0.583334 0.700133 0.
:BAN00032: 32 0.583334 0.732585 0.
:BAN00033: 33 0.672046 0.873424 0.
Energy to separate low and high energystates: -0.29268

:NOE : NUMBER OF ELECTRONS = 56.000

:FER : F E R M I - ENERGY(TETRAH.M.)= 0.2465154254
:GMA : POTENTIAL AND CHARGE CUT-OFF 12.00 Ry**.5

nband should be at least equal to the number of
(partially) occupied bands plus one. So i choose nband =34

4. i increase the EMAX from 1.5 to 6.0 in case.in1

K-VECTORS FROM UNIT:4 -9.0 6.0 117 emin / de (emax=Ef+de) / nband

5. In case.inc the print switch has to be ”1” for all atoms such that 
the core orbitals are printed


7 0.00 1 NUMBER OF ORBITALS (EXCLUDING SPIN), SHIFT, IPRINT
1,-1,2 ( N,KAPPA,OCCUP)
2,-1,2 ( N,KAPPA,OCCUP)
2, 1,2 ( N,KAPPA,OCCUP)
2,-2,4 ( N,KAPPA,OCCUP)
3,-1,2 ( N,KAPPA,OCCUP)
3, 1,2 ( N,KAPPA,OCCUP)
3,-2,4 ( N,KAPPA,OCCUP)
7 0.00 1 NUMBER OF ORBITALS (EXCLUDING SPIN), SHIFT, IPRINT
1,-1,2 ( N,KAPPA,OCCUP)
2,-1,2 ( N,KAPPA,OCCUP)
2, 1,2 ( N,KAPPA,OCCUP)
2,-2,4 ( N,KAPPA,OCCUP)
3,-1,2 ( N,KAPPA,OCCUP)
3, 1,2 ( N,KAPPA,OCCUP)
3,-2,4 ( N,KAPPA,OCCUP)
0

6. My file .inhf for PBE0

0.25 alpha
F screened (T) or unscreened (F)
0.165 lambda
34 nband
6 gmax
3 lmaxe
3 lmaxv
1d-3 tolu

8. runsp_lapw -hf and i got error "error in case.inhf: nband should be 
an integer"


Please help me. I don't know where i make mistake. Maybe nband.



___
Wien mailing list
Wien@zeus.theochem.tuwien.ac.at
http://zeus.theochem.tuwien.ac.at/mailman/listinfo/wien
SEARCH the MAILING-LIST at:  
http://www.mail-archive.com/wien@zeus.theochem.tuwien.ac.at/index.html


___
Wien mailing list
Wien@zeus.theochem.tuwien.ac.at
http://zeus.theochem.tuwien.ac.at/mailman/listinfo/wien
SEARCH the MAILING-LIST at:  
http://www.mail-archive.com/wien@zeus.theochem.tuwien.ac.at/index.html


Re: [Wien] zigzag potential interpretation

2018-01-03 Thread Xavier Rocquefelte

Dear Stefaan

I reconsider more carefully your case.struct file.

Indeed, in your case you have 8 symmetry operations.

As mentionned by Laurence :

"The equivalence may be related to how -so is implemented, with an 
implicit time-reversal for some symmetry operations. You may have to dig 
through the code to see if the vector values being output are time 
forward or backwards."


What you observe is certainly due to this aspect because point group of 
the iron atoms in this cell is 4mm.


Cheers

Xavier



Le 03/01/2018 à 15:41, Xavier Rocquefelte a écrit :

Dear Gerhard

One clarification is needed I think. The discussion was about applying 
an external ELECTRIC field (not a magnetic one).


Thus one part of your answer concerns something else which is also 
interesting :) Indeed, my PhD student has written a modification of 
WIEN2k to take into account the effect of an external magnetic field 
and we are testing this new version with Peter and Pavel at this moment.


Happy new year to you Gerhard

Xavier



Le 03/01/2018 à 14:24, Fecher, Gerhard a écrit :

Dear Stefaan,
I am not realy sure what difference you expect,
I do not see why at two seemingly same surfaces the size of the 
magnetic moment (orbital or spin) should depend on their orientation 
in the sense that it is parallel or antiparallel to the surface normal.


I wonder about the interpretation where the magnetic moment points to 
(in an absolute sense), if you change from 001 to 00-1 then the sign 
of the magnetic moment does not change,
however, if you change the sign of the magnetisation from m to -m  
(instgen) then the quantisation axis and the magnetisation may not 
longer be parallel

(the different situations are found in case.scfdmup).
The same might happen when applying an external magnetic field, it 
seems that it is never checked that all quantisation axes are 
consistent,
that means it is not checked  whether m or H parallel or antiparallel 
to the SO quantisation axes,

without SO it seems that H doesn't change the symmetry at all (!?).

If there is a difference in the wave functions it may be only in the 
sign of the phase such that it is lost in all cases where you use the 
absolute square.


Such differences in the phase enter effects that depend on the 
interferrence of waves as appear in all kinds of circular dichroism, 
you will not see them in pure intenities (square of wave functions 
but only in differences, what reminds me on Jaroslavs recent 
questions before X-mas).


Analysing the wave functions one needs to have a look on the spinors.
Note that only s up, s down correspond to |1/2,1/2>, |1/2, -1/2> ==> 
mj = ml + ms is either 0+1/2 or 0-1/2 because of ml=0 if l=0
for all higher angular momenta (l>0) mj = ml + ms may be reached by 
differen spin orientation e.g. mj = 3/2 = 1+1/2 = 2-1/2  (here you 
may have ml=0, 1, 2 for l=2)


The situation becomes worth if the quantisation axis is not along z 
(001, 00-1) but along x or y,
in the latter case one either needs to rotate the wave functions 
(leading to numerical issues) or one has additional off-diaogonal 
terms in the coupling matrices.
(note that the treatment in the ncm version of Wien2k differs from 
the regular one)


Coming back to my starting point, just something that will be 
different: If you think about XMCD then you have to change the 
direction of photons to hit the two different surfaces.

(and this might reverse the circular polarisation and thus the XMCD)



Ciao
Gerhard

DEEP THOUGHT in D. Adams; Hitchhikers Guide to the Galaxy:
"I think the problem, to be quite honest with you,
is that you have never actually known what the question is."


Dr. Gerhard H. Fecher
Institut of Inorganic and Analytical Chemistry
Johannes Gutenberg - University
55099 Mainz
and
Max Planck Institute for Chemical Physics of Solids
01187 Dresden

Von: Wien [wien-boun...@zeus.theochem.tuwien.ac.at] im Auftrag von 
Stefaan Cottenier [stefaan.cotten...@ugent.be]

Gesendet: Mittwoch, 3. Januar 2018 12:26
An: A Mailing list for WIEN2k users
Betreff: Re: [Wien] zigzag potential interpretation


Provide a indmc file as for lda+u (d-states and 0 0 at the end)
OK, done that, and now I see the vectorial information. Which 
confirms the same picture as ever before: these two surfaces are 
fully equivalent. The question remains: why...?


:ORB001:  ORBITAL MOMENT:  0.0  0.0  0.09334 PROJECTION ON M  
0.09334
:SPI001:  SPIN MOMENT:   0.0   0.0   3.00530 PROJECTION ON M  
3.00530


:ORB002:  ORBITAL MOMENT:  0.0  0.0  0.09334 PROJECTION ON M  
0.09334
:SPI002:  SPIN MOMENT:   0.0   0.0   3.00531 PROJECTION ON M  
3.00531


Stefaan




On 01/03/2018 12:02 PM, Stefaan Cottenier wrote:

Runx lapwdm -so -up

and look at the spin and orbital moments (vectorial) of the atoms 
there.

Hello Peter,

See underneath. I don't see vectorial informatio

Re: [Wien] zigzag potential interpretation

2018-01-02 Thread Xavier Rocquefelte

A piece of paper will be useful to discuss this point ;)

To my point of you, the picture is correct: Fe moment point inward and 
outward. However, I think that for a given direction (c direction) the 
001 and 00-1 orientation will lead to inward and outward respectively, 
which will give the same spin moment and orbital moment. It is due to 
the fact that the SO-effect will split the 3d orbitals similarly for the 
001 and 00-1 orientations. Doing two calculations with 001 and 00-1 
magnetization direction will lead to reverse the Fe moment for a given 
surface, and thus you will have inward and outward, respectively.


In your calculations, you have both (inward and outward) for one 
magnetization direction due to the surface termination.


The only limitation I see here is related to the definition of the Fermi 
level which can lead to difficulties to properly distinguish the two 
surfaces. Would it be possible that here is the problem? Are the partial 
DOS exactly the same?


Best Regards

Xavier





Le 02/01/2018 à 16:08, Stefaan Cottenier a écrit :


Hello Xavier,

You touch some of the points I have been pondering, indeed.

For bulk bcc-Fe, there would be no problem. Having spin-orbit along 
001 or along 00-1 must lead to the same result. In my naive picture, 
this is equivalent to having the Fe-moment pointing along 001 or along 
00-1, and for an infinite bulk lattice this is identical.


For a slab, the situation is slightly different. My expectation was 
that all global properties (e.g. total energy) would not depend on the 
choice between 001 or 00-1: there would be two inequivalent surfaces, 
but taking the other orientation for the moment would just interchange 
the two surfaces. The sum of both, would not change. What does 
surprise me, however, is that the two surfaces are *not* inequivalent: 
not only global properties yet also local properties (spin moment, 
EFG,…) are identical for the two surfaces.


When I forget about the electric field of the initial question, and 
use the unit cell suggested by sgroup, then the two surface layers 
become equivalent. Even after ‘breaking’ the symmetry by initso_lapw. 
That suggests it’s a general property, and not related to a particular 
orbital occupation as you suggest in your second post.


I suspect my naive interpretation of the Fe moment pointing ‘inward’ 
for one surface layer and pointing ‘outward’ for the other layer, is 
not correct. Yet I don’t see why.


Thanks!

Stefaan

*Van:*Wien [mailto:wien-boun...@zeus.theochem.tuwien.ac.at] *Namens 
*Xavier Rocquefelte

*Verzonden:* dinsdag 2 januari 2018 15:38
*Aan:* wien@zeus.theochem.tuwien.ac.at
*Onderwerp:* Re: [Wien] zigzag potential interpretation

Dear Stefaan

As always it is very nice to read your posts :)

I will only react on your "Thought 3". What will happen if you do the 
same calculation along 00-1? To my point of view, you will obtain the 
same result. Indeed, the magnetic anisotropy (MAE) of bulk-Fe must be 
symmetric. Here you break the symmetry, it could be seen considering 2 
local pictures (for each slab surface):

- one experiencing a magnetization direction along 001
- one along 00-1.
These two directions must lead to the same SO effects and thus the 
same spin moments, orbital moments and EFG.


Here is one plausible interpretation ;) I hope it will help you.

I wish you all the best and HAPPY NEW YEAR to you and your familly.
Xavier



Le 02/01/2018 à 14:33, Stefaan Cottenier a écrit :

Dear wien2k mailing list,

I know that the Berry phase approach is the recommended way
nowadays for applying an external electric field in wien2k.
However, for a quick test I resorted to the old zigzag potential
that is described in the usersguide, sec. 7.1.

It works, but I have some questions to convince me that I’m
interpreting it the right way.

The test situation I try to reproduce is from this paper
(https://doi.org/10.1103/PhysRevLett.101.137201), in particular
this picture

(https://journals.aps.org/prl/article/10.1103/PhysRevLett.101.137201/figures/1/medium).
It’s a free-standing slab of bcc-Fe layers, with an electric field
perpendicular to the slab. For convenience, I use only 7
Fe-monolayers (case.struct is pasted underneath). Spin orbit
coupling is used, and the Fe spin moments point in the positive
z-direction.

This is the input I used in case.in0 (the last line triggers the
electric field) :

TOT  XC_PBE (XC_LDA,XC_PBESOL,XC_WC,XC_MBJ,XC_REVTPSS)

NR2V  IFFT (R2V)

   30   30  360 2.00  1min IFFT-parameters, enhancement
factor, iprint

30 1.266176 1.

Question 1: The usersguide tells “The electric field (in Ry/bohr)
corresponds to EFIELD/c, where c is your c lattice parameter.” In
my example, EFIELD=1.266176 and c=65.082193 b, hence the electric
field should be 0.019455 Ry/bohr. That’s 0.5 V/Angstrom. However,
by comparing the dependence of 

Re: [Wien] zigzag potential interpretation

2018-01-02 Thread Xavier Rocquefelte

To be honest I also feel that something is missing in my last arguments.

What is the electronic configuration of Fe at the surface? The orbital 
occupancy could play a role in the understanding of the present 
observation.



Le 02/01/2018 à 15:37, Xavier Rocquefelte a écrit :


Dear Stefaan

As always it is very nice to read your posts :)

I will only react on your "Thought 3". What will happen if you do the 
same calculation along 00-1? To my point of view, you will obtain the 
same result. Indeed, the magnetic anisotropy (MAE) of bulk-Fe must be 
symmetric. Here you break the symmetry, it could be seen considering 2 
local pictures (for each slab surface):

- one experiencing a magnetization direction along 001
- one along 00-1.
These two directions must lead to the same SO effects and thus the 
same spin moments, orbital moments and EFG.


Here is one plausible interpretation ;) I hope it will help you.

I wish you all the best and HAPPY NEW YEAR to you and your familly.
Xavier




Le 02/01/2018 à 14:33, Stefaan Cottenier a écrit :


Dear wien2k mailing list,

I know that the Berry phase approach is the recommended way nowadays 
for applying an external electric field in wien2k. However, for a 
quick test I resorted to the old zigzag potential that is described 
in the usersguide, sec. 7.1.


It works, but I have some questions to convince me that I’m 
interpreting it the right way.


The test situation I try to reproduce is from this paper 
(https://doi.org/10.1103/PhysRevLett.101.137201), in particular this 
picture 
(https://journals.aps.org/prl/article/10.1103/PhysRevLett.101.137201/figures/1/medium). 
It’s a free-standing slab of bcc-Fe layers, with an electric field 
perpendicular to the slab. For convenience, I use only 7 
Fe-monolayers (case.struct is pasted underneath). Spin orbit coupling 
is used, and the Fe spin moments point in the positive z-direction.


This is the input I used in case.in0 (the last line triggers the 
electric field) :


TOT  XC_PBE (XC_LDA,XC_PBESOL,XC_WC,XC_MBJ,XC_REVTPSS)

NR2V  IFFT (R2V)

   30   30  360 2.00  1min IFFT-parameters, enhancement factor, 
iprint


30 1.266176 1.

Question 1: The usersguide tells “The electric field (in Ry/bohr) 
corresponds to EFIELD/c, where c is your c lattice parameter.” In my 
example, EFIELD=1.266176 and c=65.082193 b, hence the electric field 
should be 0.019455 Ry/bohr. That’s 0.5 V/Angstrom. However, by 
comparing the dependence of the moment on the field with the paper 
cited above, it looks like that value for field is just half of what 
it should be (=the moment changed as if it were subject to a field of 
1.0 V/Angstrom). When looking at the definition of the atomic unit of 
electric field (https://physics.nist.gov/cgi-bin/cuu/Value?auefld), I 
see it is defined with Hartree, not Rydberg. This factor 2 would 
explain it. Does someone know whether 2*EFIELD/c is the proper way to 
get the value of the applied electric field in WIEN2k?


Question 2: It is not clear from the userguide where the extrema in 
the zigzagpotential are. Are they at z=0 and z=0.5, as in fig. 6 of 
http://dx.doi.org/10.1103/PhysRevB.63.165205 ? I assumed so, that’s 
why the slab in my case struct is positioned around z=0.25. Adding 
this information to the usersguide or to the documentation in the 
code would be useful. (or alternatively, printing the zigzag 
potential as function of z by default would help too)


Thought 3: This is not related to the electric field as such, but 
when playing with the slab underneath, I notice that in the absence 
of an electric field all properties of atoms 1 and 2 – the ‘left’ and 
‘right’ terminating slab surfaces – are identical. Same spin moment, 
same orbital moment, same EFG,… I didn’t expect this, as with 
magnetism and spin-orbit coupling along 001, the magnetic moments of 
the atoms are pointing in the positive z-direction. That means ‘from 
the vacuum to the bulk’ for atom 1, and ‘from the bulk to the vacuum’ 
for atom 2. That’s not the same situation, so why does it lead to 
exactly the same properties? What do I miss here? (The forces (:FGL) 
for atoms 1 and 2 are opposite, as expected.  And when the electric 
field is switched on, atoms 1 and 2 do become different, as expected.)


Thanks for your insight,

Stefaan

bleblebles-o calc. M||  0.00  0.00  1.00

P 7 99 P

 RELA

  5.423516  5.423516 65.082193 90.00 90.00 90.00

ATOM  -1: X=0. Y=0. Z=0.1250

MULT= 1  ISPLIT=-2

Fe1NPT=  781  R0=.5 RMT= 2.22000   Z:  26.0

LOCAL ROT MATRIX:1.000 0.000 0.000

 0.000 1.000 0.000

 0.000 0.000 1.000

ATOM  -2: X=0. Y=0. Z=0.3750

  MULT= 1  ISPLIT=-2

Fe2NPT=  781  R0=.5 RMT= 2.22000   Z:  26.0

LOCAL ROT MATRIX:1.000 0.000

Re: [Wien] zigzag potential interpretation

2018-01-02 Thread Xavier Rocquefelte

Dear Stefaan

As always it is very nice to read your posts :)

I will only react on your "Thought 3". What will happen if you do the 
same calculation along 00-1? To my point of view, you will obtain the 
same result. Indeed, the magnetic anisotropy (MAE) of bulk-Fe must be 
symmetric. Here you break the symmetry, it could be seen considering 2 
local pictures (for each slab surface):

- one experiencing a magnetization direction along 001
- one along 00-1.
These two directions must lead to the same SO effects and thus the same 
spin moments, orbital moments and EFG.


Here is one plausible interpretation ;) I hope it will help you.

I wish you all the best and HAPPY NEW YEAR to you and your familly.
Xavier




Le 02/01/2018 à 14:33, Stefaan Cottenier a écrit :


Dear wien2k mailing list,

I know that the Berry phase approach is the recommended way nowadays 
for applying an external electric field in wien2k. However, for a 
quick test I resorted to the old zigzag potential that is described in 
the usersguide, sec. 7.1.


It works, but I have some questions to convince me that I’m 
interpreting it the right way.


The test situation I try to reproduce is from this paper 
(https://doi.org/10.1103/PhysRevLett.101.137201), in particular this 
picture 
(https://journals.aps.org/prl/article/10.1103/PhysRevLett.101.137201/figures/1/medium). 
It’s a free-standing slab of bcc-Fe layers, with an electric field 
perpendicular to the slab. For convenience, I use only 7 Fe-monolayers 
(case.struct is pasted underneath). Spin orbit coupling is used, and 
the Fe spin moments point in the positive z-direction.


This is the input I used in case.in0 (the last line triggers the 
electric field) :


TOT  XC_PBE (XC_LDA,XC_PBESOL,XC_WC,XC_MBJ,XC_REVTPSS)

NR2V  IFFT (R2V)

   30   30  3602.00 1min IFFT-parameters, enhancement factor, 
iprint


30 1.266176 1.

Question 1: The usersguide tells “The electric field (in Ry/bohr) 
corresponds to EFIELD/c, where c is your c lattice parameter.” In my 
example, EFIELD=1.266176 and c=65.082193 b, hence the electric field 
should be 0.019455 Ry/bohr. That’s 0.5 V/Angstrom. However, by 
comparing the dependence of the moment on the field with the paper 
cited above, it looks like that value for field is just half of what 
it should be (=the moment changed as if it were subject to a field of 
1.0 V/Angstrom). When looking at the definition of the atomic unit of 
electric field (https://physics.nist.gov/cgi-bin/cuu/Value?auefld), I 
see it is defined with Hartree, not Rydberg. This factor 2 would 
explain it. Does someone know whether 2*EFIELD/c is the proper way to 
get the value of the applied electric field in WIEN2k?


Question 2: It is not clear from the userguide where the extrema in 
the zigzagpotential are. Are they at z=0 and z=0.5, as in fig. 6 of 
http://dx.doi.org/10.1103/PhysRevB.63.165205 ? I assumed so, that’s 
why the slab in my case struct is positioned around z=0.25. Adding 
this information to the usersguide or to the documentation in the code 
would be useful. (or alternatively, printing the zigzag potential as 
function of z by default would help too)


Thought 3: This is not related to the electric field as such, but when 
playing with the slab underneath, I notice that in the absence of an 
electric field all properties of atoms 1 and 2 – the ‘left’ and 
‘right’ terminating slab surfaces – are identical. Same spin moment, 
same orbital moment, same EFG,… I didn’t expect this, as with 
magnetism and spin-orbit coupling along 001, the magnetic moments of 
the atoms are pointing in the positive z-direction. That means ‘from 
the vacuum to the bulk’ for atom 1, and ‘from the bulk to the vacuum’ 
for atom 2. That’s not the same situation, so why does it lead to 
exactly the same properties? What do I miss here? (The forces (:FGL) 
for atoms 1 and 2 are opposite, as expected.  And when the electric 
field is switched on, atoms 1 and 2 do become different, as expected.)


Thanks for your insight,

Stefaan

blebleble   s-o calc. M||  0.00  0.00 1.00

P 7 99 P

RELA

5.423516  5.423516 65.082193 90.00 90.00 90.00

ATOM -1: X=0. Y=0. Z=0.1250

MULT= 1  ISPLIT=-2

Fe1 NPT=  781  R0=.5 RMT=   2.22000   Z:  26.0

LOCAL ROT MATRIX:1.000 0.000 0.000

0.000 1.000 0.000

0.000 0.000 1.000

ATOM -2: X=0. Y=0. Z=0.3750

MULT= 1  ISPLIT=-2

Fe2 NPT=  781  R0=.5 RMT=   2.22000   Z:  26.0

LOCAL ROT MATRIX:1.000 0.000 0.000

0.000 1.000 0.000

   0.000 0.000 1.000

ATOM -3: X=0. Y=0. Z=0.2083

MULT= 1  ISPLIT=-2

Fe3 NPT=  781  R0=.5 RMT=   2.22000   Z:  26.0

LOCAL ROT MATRIX:1.000 0.000 0.000

 0.000 1.000 0.000

0.000 0.000 1.000

ATOM -4: X=0. Y=0. 

Re: [Wien] stability issue

2017-12-22 Thread Xavier Rocquefelte

Dear Chin

Your question is a little aside from the WIENLIST. Indeed, it is 
thermodynamics.


In equation 1, you evaluate the enthalpie of formation of the compound 
ABO3 from the reactants AO and BO2 using internal energies. Indeed, you 
neglect the volumetric effect (P deltaV). From your DFT calculation, if 
correct, you can simply conclude that the following reaction is 
endothermic.


Equation 2 gives you a different reaction and in your case is 
exothermic. We cannot answer in more details based on your message which 
do not provide any details. I can simply say that depending on the 
experimental conditions, the atmosphere can be oxygen rich / oxygen poor 
... you can start from pure solids A and B ... or other compounds ... In 
all case you will estimate a total energy difference which will give you 
an idea of the endo/exo-thermic nature of the reaction. You will not 
estimate the Gibbs energy (because you also neglect the entropic part T 
deltaS). However, in many articles we consider that estimation of 
internal energies are sufficient for solids because both deltaV and 
deltaS are negligeable (but such an approximation can be really bad).


Hope it will help you.

Cheers

Xavier




Le 22/12/2017 à 10:32, chin Sabsu a écrit :

Dear Sir,

I am working on the system ABO3 .

I have calculated formation energy in two ways:

Delta_E= Et[ABO3]  - Et[AO]  - Et[BO2]  -- (1)

 Delta_E= Et[ABO3] - Et[A] - Et[B] - 3/2Et[O2]- (2)

The Delta_E is positive 0.04 Ry from Eq (1) while is negative from Eq 2.

Why is the energy difference calculated from Eq (1) positive?


Thanks in advance!


Regards

Chin



___
Wien mailing list
Wien@zeus.theochem.tuwien.ac.at
http://zeus.theochem.tuwien.ac.at/mailman/listinfo/wien
SEARCH the MAILING-LIST at:  
http://www.mail-archive.com/wien@zeus.theochem.tuwien.ac.at/index.html


___
Wien mailing list
Wien@zeus.theochem.tuwien.ac.at
http://zeus.theochem.tuwien.ac.at/mailman/listinfo/wien
SEARCH the MAILING-LIST at:  
http://www.mail-archive.com/wien@zeus.theochem.tuwien.ac.at/index.html


Re: [Wien] The number of bands is less than what I want

2017-07-15 Thread Xavier Rocquefelte

Dear Bingrui Peng

What are the eigenvalues and how many bands do you have in the valence 
states.


When you did "x lapw1 -band" what was the energy range of your calculation.

Look at the case.output1 file, it contains the eigenvalues for each 
k-points. You will then see the number of bands and energy range.


Cheers

Xavier


Le 15/07/2017 à 12:57, Peng Bingrui a écrit :

Dear wien2k community

I'm running WIEN2K of 13 version on Linux system.

My case.insp is like this:
---
### Figure configuration
 5.0   3.0 # paper offset of plot
10.0  15.0   # xsize,ysize [cm]
 1.0   4# major ticks, minor ticks
 1.0   1# character height, font switch
 1.1   24  # line width, line switch, 
color switch

### Data configuration
-14.0  28.0  2  # energy range, energy switch 
(1:Ry, 2:eV)

1  0.6847884066# Fermi switch,  Fermi-level (in Ry units)
1  99   # number of bands for heavier 
plotting   1,1
0  11.0# jatom, jtype, size  of 
heavier plotting

---

I want to get 99 bands plotted, but there are only 80 bands shown when 
I open case.bands.agr. And note that although I set the energy range 
to be (-14.0, 28.0) , the highest band, is below 12.0 eV, which is 
much lower than 28.0 eV. What should I do to get more bands ?


Thank you very much for your attention.

Sincerely yours,
Bingrui Peng
from the Department of Physics, Nanjing University,
China


___
Wien mailing list
Wien@zeus.theochem.tuwien.ac.at
http://zeus.theochem.tuwien.ac.at/mailman/listinfo/wien
SEARCH the MAILING-LIST at:  
http://www.mail-archive.com/wien@zeus.theochem.tuwien.ac.at/index.html


___
Wien mailing list
Wien@zeus.theochem.tuwien.ac.at
http://zeus.theochem.tuwien.ac.at/mailman/listinfo/wien
SEARCH the MAILING-LIST at:  
http://www.mail-archive.com/wien@zeus.theochem.tuwien.ac.at/index.html


Re: [Wien] Ferromagnetic compound has non-magnetic moments

2017-06-12 Thread Xavier Rocquefelte

Dear Timothy

The main problem here is that in such intermetallic compound you must 
have both itinerant and localized magnetisms which is not so trivial to 
treat from DFT and adding an Hubbard correction will not help. To my 
point of view the problem can be solved by looking at the experimental 
data to better understand which kind of magnetic properties are 
expected. Is the compound metallic?


Best wishes

Xavier


Le 12/06/2017 à 18:39, Tim Hackett a écrit :

Greetings Prof. Blaha and all WIEN2k community,

I have a compound which consists of 29 atoms and is made up of Nb-Ni-B 
[Mg_3 Ni_20 B_6 -type, /cF/116, /Fm//-/3/m/ (No. 225)] that has been 
found experimentally to be magnetic.
But in my WIEN2k calculations using GGA and GGA+U with and without SO 
coupling, I found almost zero moment on each element with all formalism.


I used an RKmax = 7.5 in case.in1 (as per suggestion in the FAQ), 1000 
k points (I also used 2000 and there is no difference) and I converged 
with at least 10^-6 for both energy and charge criteria.


I searched extensively through the mailing list and have found no 
solution on this issue.


Please suggest how can I improve my calculations to match the 
experimental finding.


Best Regards-
--

Timothy A. Hackett

Assistant Scientist I

Division of Materials Science & Engineering

Ames Laboratory, Iowa State University, Ames, IA 50011



___
Wien mailing list
Wien@zeus.theochem.tuwien.ac.at
http://zeus.theochem.tuwien.ac.at/mailman/listinfo/wien
SEARCH the MAILING-LIST at:  
http://www.mail-archive.com/wien@zeus.theochem.tuwien.ac.at/index.html


___
Wien mailing list
Wien@zeus.theochem.tuwien.ac.at
http://zeus.theochem.tuwien.ac.at/mailman/listinfo/wien
SEARCH the MAILING-LIST at:  
http://www.mail-archive.com/wien@zeus.theochem.tuwien.ac.at/index.html


Re: [Wien] Ferromagnetic compound has non-magnetic moments

2017-06-12 Thread Xavier Rocquefelte

Dear Timothy

The main problem here is that in such intermetallic compound you must 
have both itinerant and localized magnetisms which is not so trivial to 
treat from DFT and adding an Hubbard correction will not help. To my 
point of view the problem can be solved by looking at the experimental 
data to better understand which kind of magnetic properties are 
expected. Is the compound metallic?


Best wishes

Xavier


Le 12/06/2017 à 18:39, Tim Hackett a écrit :

Greetings Prof. Blaha and all WIEN2k community,

I have a compound which consists of 29 atoms and is made up of Nb-Ni-B 
[Mg_3 Ni_20 B_6 -type, /cF/116, /Fm//-/3/m/ (No. 225)] that has been 
found experimentally to be magnetic.
But in my WIEN2k calculations using GGA and GGA+U with and without SO 
coupling, I found almost zero moment on each element with all formalism.


I used an RKmax = 7.5 in case.in1 (as per suggestion in the FAQ), 1000 
k points (I also used 2000 and there is no difference) and I converged 
with at least 10^-6 for both energy and charge criteria.


I searched extensively through the mailing list and have found no 
solution on this issue.


Please suggest how can I improve my calculations to match the 
experimental finding.


Best Regards-
--

Timothy A. Hackett

Assistant Scientist I

Division of Materials Science & Engineering

Ames Laboratory, Iowa State University, Ames, IA 50011



___
Wien mailing list
Wien@zeus.theochem.tuwien.ac.at
http://zeus.theochem.tuwien.ac.at/mailman/listinfo/wien
SEARCH the MAILING-LIST at:  
http://www.mail-archive.com/wien@zeus.theochem.tuwien.ac.at/index.html


___
Wien mailing list
Wien@zeus.theochem.tuwien.ac.at
http://zeus.theochem.tuwien.ac.at/mailman/listinfo/wien
SEARCH the MAILING-LIST at:  
http://www.mail-archive.com/wien@zeus.theochem.tuwien.ac.at/index.html


Re: [Wien] How to know the user id & password for graphical interface of wien2k

2017-04-01 Thread Rocquefelte Xavier
Indeed the intetface is a realization of Joachim Luitz ... I remember  
a Wien-in-a-boat interface (the grand father of w2web) .. an ideal  
boat to catch a 1st april fish :)




Le sam., 01 avril 2017, "Fecher, Gerhard" <fec...@uni-mainz.de> a écrit :


You installed Wien2k
You started W2web
You had to give a user id and a password and typed it in

And now you ask the forum about the user id and the password ?

Thanks, its April 1st

Ciao
Gerhard

DEEP THOUGHT in D. Adams; Hitchhikers Guide to the Galaxy:
"I think the problem, to be quite honest with you,
is that you have never actually known what the question is."


Dr. Gerhard H. Fecher
Institut of Inorganic and Analytical Chemistry
Johannes Gutenberg - University
55099 Mainz
and
Max Planck Institute for Chemical Physics of Solids
01187 Dresden

Von: Wien [wien-boun...@zeus.theochem.tuwien.ac.at] im Auftrag von  
shamik chakrabarti [shamik...@gmail.com]

Gesendet: Samstag, 1. April 2017 09:09
An: A Mailing list for WIEN2k users
Betreff: Re: [Wien] How to know the user id & password for graphical  
interface of wien2k


Dear Xavier,

   if without deleting the .w2web file, if I run  
./userconfig_lapw, whether that will give me new id & password for  
graphical interface?...As the installation of wien2k itself is a  
tough job, I do not want to take risk of deleting any files, as long  
as we have other ways to do it.


with regards,

On Sat, Apr 1, 2017 at 12:11 AM, Xavier Rocquefelte  
<xavier.rocquefe...@univ-rennes1.fr<mailto:xavier.rocquefe...@univ-rennes1.fr>>  
wrote:


Dear Shamik

The userid and password are generated when you start for the first  
time the w2web interface by typing in the unix terminal:


w2web

If it has been already generated, you have then created a directory  
named .w2web which contains configuration files.


One easy solution is to remove this directory: rm -f .w2web

Then type: w2web and answer the questions which will lead to  
recreate the directory with the userid and password you will choose.


Best Regards

Xavier

Le 31/03/2017 à 18:29, shamik chakrabarti a écrit :
Dear Wien2k users,

  I am now try to work in a system, in which  
while giving the port number to the address bar of a browser & it is  
then asking for user id & password to open graphical interface of  
wien2k. Unfortunately I do not know the id & the password for the  
interface to open. Is there any file or directory in which I can get  
the log in information for graphical interface.


Looking forward to your response in this regard.

with regards,

--
Dr. Shamik Chakrabarti
Research Associate
Electroceramics Lab
Dept. of Metallurgical & Materials Engineering
IIT Kharagpur
Kharagpur 721302
INDIA



___
Wien mailing list
Wien@zeus.theochem.tuwien.ac.at<mailto:Wien@zeus.theochem.tuwien.ac.at>
http://zeus.theochem.tuwien.ac.at/mailman/listinfo/wien
SEARCH the MAILING-LIST at:   
http://www.mail-archive.com/wien@zeus.theochem.tuwien.ac.at/index.html




___
Wien mailing list
Wien@zeus.theochem.tuwien.ac.at<mailto:Wien@zeus.theochem.tuwien.ac.at>
http://zeus.theochem.tuwien.ac.at/mailman/listinfo/wien
SEARCH the MAILING-LIST at:   
http://www.mail-archive.com/wien@zeus.theochem.tuwien.ac.at/index.html





--
Dr. Shamik Chakrabarti
Post Doctoral Research Associate
Dept. of Condensed Matter Physics and  Material Science
S N Bose National Centre for Basic Sciences
Kolkata-700098
INDIA
___
Wien mailing list
Wien@zeus.theochem.tuwien.ac.at
http://zeus.theochem.tuwien.ac.at/mailman/listinfo/wien
SEARCH the MAILING-LIST at:   
http://www.mail-archive.com/wien@zeus.theochem.tuwien.ac.at/index.html



___
Wien mailing list
Wien@zeus.theochem.tuwien.ac.at
http://zeus.theochem.tuwien.ac.at/mailman/listinfo/wien
SEARCH the MAILING-LIST at:  
http://www.mail-archive.com/wien@zeus.theochem.tuwien.ac.at/index.html


Re: [Wien] How to know the user id & password for graphical interface of wien2k

2017-04-01 Thread Rocquefelte Xavier

There is no risk. This is nicely explained in the userguide for more details.
Cheers




Le sam., 01 avril 2017, shamik chakrabarti <shamik...@gmail.com> a écrit :


Dear Xavier,

   if without deleting the .w2web file, if I run
./userconfig_lapw, whether that will give me new id & password for
graphical interface?...As the installation of wien2k itself is a tough job,
I do not want to take risk of deleting any files, as long as we have other
ways to do it.

with regards,

On Sat, Apr 1, 2017 at 12:11 AM, Xavier Rocquefelte <
xavier.rocquefe...@univ-rennes1.fr> wrote:


Dear Shamik

The userid and password are generated when you start for the first time
the w2web interface by typing in the unix terminal:

w2web

If it has been already generated, you have then created a directory named
.w2web which contains configuration files.

One easy solution is to remove this directory: rm -f .w2web

Then type: w2web and answer the questions which will lead to recreate the
directory with the userid and password you will choose.

Best Regards

Xavier

Le 31/03/2017 à 18:29, shamik chakrabarti a écrit :

Dear Wien2k users,

  I am now try to work in a system, in which while
giving the port number to the address bar of a browser & it is then asking
for user id & password to open graphical interface of wien2k. Unfortunately
I do not know the id & the password for the interface to open. Is there any
file or directory in which I can get the log in information for graphical
interface.

Looking forward to your response in this regard.

with regards,

--
Dr. Shamik Chakrabarti
Research Associate
Electroceramics Lab
Dept. of Metallurgical & Materials Engineering
IIT Kharagpur
Kharagpur 721302
INDIA


___
Wien mailing  
listw...@zeus.theochem.tuwien.ac.athttp://zeus.theochem.tuwien.ac.at/mailman/listinfo/wien
SEARCH the MAILING-LIST at:   
http://www.mail-archive.com/wien@zeus.theochem.tuwien.ac.at/index.html




___
Wien mailing list
Wien@zeus.theochem.tuwien.ac.at
http://zeus.theochem.tuwien.ac.at/mailman/listinfo/wien
SEARCH the MAILING-LIST at:  http://www.mail-archive.com/
wien@zeus.theochem.tuwien.ac.at/index.html





--
Dr. Shamik Chakrabarti
Post Doctoral Research Associate
Dept. of Condensed Matter Physics and  Material Science
S N Bose National Centre for Basic Sciences
Kolkata-700098
INDIA



___
Wien mailing list
Wien@zeus.theochem.tuwien.ac.at
http://zeus.theochem.tuwien.ac.at/mailman/listinfo/wien
SEARCH the MAILING-LIST at:  
http://www.mail-archive.com/wien@zeus.theochem.tuwien.ac.at/index.html


Re: [Wien] How to know the user id & password for graphical interface of wien2k

2017-03-31 Thread Xavier Rocquefelte

Dear Shamik

The userid and password are generated when you start for the first time 
the w2web interface by typing in the unix terminal:


w2web

If it has been already generated, you have then created a directory 
named .w2web which contains configuration files.


One easy solution is to remove this directory: rm -f .w2web

Then type: w2web and answer the questions which will lead to recreate 
the directory with the userid and password you will choose.


Best Regards

Xavier


Le 31/03/2017 à 18:29, shamik chakrabarti a écrit :

Dear Wien2k users,

  I am now try to work in a system, in which 
while giving the port number to the address bar of a browser & it is 
then asking for user id & password to open graphical interface of 
wien2k. Unfortunately I do not know the id & the password for the 
interface to open. Is there any file or directory in which I can get 
the log in information for graphical interface.


Looking forward to your response in this regard.

with regards,

--
Dr. Shamik Chakrabarti
Research Associate
Electroceramics Lab
Dept. of Metallurgical & Materials Engineering
IIT Kharagpur
Kharagpur 721302
INDIA


___
Wien mailing list
Wien@zeus.theochem.tuwien.ac.at
http://zeus.theochem.tuwien.ac.at/mailman/listinfo/wien
SEARCH the MAILING-LIST at:  
http://www.mail-archive.com/wien@zeus.theochem.tuwien.ac.at/index.html


___
Wien mailing list
Wien@zeus.theochem.tuwien.ac.at
http://zeus.theochem.tuwien.ac.at/mailman/listinfo/wien
SEARCH the MAILING-LIST at:  
http://www.mail-archive.com/wien@zeus.theochem.tuwien.ac.at/index.html


Re: [Wien] Need help to set k-points

2017-02-01 Thread Xavier Rocquefelte
One of the best approach is to do a SCF calculation for a small number 
of k-points and then increase gradually this number.


Then you plot the total energy as a function of the number of k-points 
and you will choose the smallest number giving rise to a good convergency.


Ideally you should do such calculations as soon as you have a new system 
to insure that your k-point mesh is OK.


I should add that the mesh will also depend on the property you are 
looking for.


Regards

Xavier


Le 01/02/2017 à 12:09, GM RAI a écrit :

Dear Wien2k User,

How can we select number of k-points for best convergence?

Regards

--
Dr. G. Murtaza



___
Wien mailing list
Wien@zeus.theochem.tuwien.ac.at
http://zeus.theochem.tuwien.ac.at/mailman/listinfo/wien
SEARCH the MAILING-LIST at:  
http://www.mail-archive.com/wien@zeus.theochem.tuwien.ac.at/index.html


___
Wien mailing list
Wien@zeus.theochem.tuwien.ac.at
http://zeus.theochem.tuwien.ac.at/mailman/listinfo/wien
SEARCH the MAILING-LIST at:  
http://www.mail-archive.com/wien@zeus.theochem.tuwien.ac.at/index.html


Re: [Wien] UNPHYSICAL RMT of atom

2017-01-31 Thread Xavier Rocquefelte
It seems that you did a mistake in the generation of the case.struct 
file. If you have the structure in VESTA you can export in CIF format 
and then use cif2struct.


Best Regards

Xavier


Le 01/02/2017 à 08:21, Rajneesh Chaurasiya a écrit :

Dear Wien2k user,

I want to compute the electronic properties of TiO2 anatase phase so 
in this order, i used the structural parameters

a=b=3.7A and c=9.7A
after doing the process of rmt reduction i found the rmt vales are
Ti=1.38
O=1.24
sum of NN distance is 2.63647 which is close to sum of rmt value.
i also searched in your mailing but same problem found by some other 
users but i could not find the proper solution.


I have seen the .struct file in VESTA which is looking fine as per my 
knowledge.


So please suggest me where i did wrong in calculation.
-
Thanks & Regards
Rajneesh Chaurasiya
Research Scholar
IIT Jodhpur, India
Mob. No. +91-9584499697
  +91-7610950803


___
Wien mailing list
Wien@zeus.theochem.tuwien.ac.at
http://zeus.theochem.tuwien.ac.at/mailman/listinfo/wien
SEARCH the MAILING-LIST at:  
http://www.mail-archive.com/wien@zeus.theochem.tuwien.ac.at/index.html


___
Wien mailing list
Wien@zeus.theochem.tuwien.ac.at
http://zeus.theochem.tuwien.ac.at/mailman/listinfo/wien
SEARCH the MAILING-LIST at:  
http://www.mail-archive.com/wien@zeus.theochem.tuwien.ac.at/index.html


Re: [Wien] Mixer surprise when using PBE0 hybrid on-site functional

2017-01-22 Thread Xavier Rocquefelte
Thank you Peter for your reply. I had no doubt that it was possible to 
converge the calculation using such strategy. My point was that such 
calculation was easy to converge starting from the scratch, directly 
using PBE0 and without constraining the total magnetic moment of the 
unit cell. At this moment I do not know if the problem is related to the 
mixer or to the PBE0 on-site. Indeed, I can imagine that not only mixer 
has changed but also PBE0 onsite compared to the WIEN2k version I was 
using in 2008. I plan to do the test Fabien has proposed.


Best wishes

Xavier


Le 23/01/2017 à 07:00, Peter Blaha a écrit :

Sorry, but I cannot reproduce this.

Starting from a converged GGA+U calculation, -eece converges smoothly, 
keeps the :mmt at 8 uB and a reasonable gap of 1.6 eV and :MV goes to 
10-4.


(quick test with few k-points and rkmax only 6.5)

Am 20.01.2017 um 22:16 schrieb Laurence Marks:

I can provide some partial responses, although there are also some
things that I don't understand. Some of this (maybe most) is not the
mixer but in other parts of Wien2k.

First, the old (2008) version is there if you use MSEC1, but I have not
tested it and it may fail. Better is to use MSEC3 which is almost the
old version. For some classes of problems this is more stable than MSR1,
and works better. If you are talking about the pre-multisecant version
(BROYD) that vanished some time ago.

Second, there is a nasty "feature" particularly for +U (eece) cases,
which is partially discussed in the mixer Readme. There is no guarantee
that a solution exists -- the KS theorem is for densities but U is an
orbital term. It is very possible to have cases where there is no
fixed-point solution. The older MSEC1 (maybe BROYD) could find a fake
solution where the density was consistent but the orbital potential was
not. The latest version is much better in avoiding them and going for
"real" solutions rather than being trapped. For orbital potentials it is
very important to look at :MV to check that one really has a
self-consistent orbital potential.

Third, there are cases where PBE (and all the GGA's in Wien2k that I
have tested) give unphysical results when applied to isolated d or f
electrons as done for -eece. I guess that the GGA functionals were not
designed for the densities of just high L orbitals. This leads to very
bad behavior of the mixing. I know of no way to solve this in the mixer,
it is a structural problem. It goes away if LDA is used as the form for
VXC in -eece.

Fourth, larger problem with low symmetry (P1 in particular) can
certainly behave badly. Part of this might be "somewhere" in Wien2k
coding, part of it is generic to a low symmetry problem. In many cases
these have small eigenvalues in the mixing Jacobian which are removed
when symmetry is imposed. All one can do is use MSEC3 or some of the
additional flags (see the mixer README) such as "SLOW".

Fifth...probably exists, but I can't think of it immediately.

On Fri, Jan 20, 2017 at 2:03 PM, Xavier Rocquefelte
<xavier.rocquefe...@univ-rennes1.fr
<mailto:xavier.rocquefe...@univ-rennes1.fr>> wrote:

Dear Colleagues

I did recently a calculation which has been published long time ago
using a old WIEN2k version (in 2008).

It corresponds to a spin-polarized calculation for the compound 
CuO. The
symmetry is removed and the idea is to estimate the total 
energies for
different magnetic orders to extract magnetic couplings from a 
mapping
analysis. Such calculations were converging fastly without any 
trouble

in 2008.

Here I have started from the scratch with a case.cif file to 
generate

the case.struct file and initializing the calculation in a standard
manner.

Then I wanted to have the energy related to a ferromagnetic 
situation

(not the more stable). I have 8 copper sites in the unit cell I am
using.

When this calculation is done using PBE+U everything goes fine. 
However
when PBE0 hybrid on-site functional is used we observed 
oscillations and

the magnetic moment disappear, which is definitely not correct. It
should be mentionned that the convergency is really bad. If we do a
similar calculation on the cristallographic unit cell (2 copper 
sites

only) the calculations converge both in PBE+U and PBE0.

The convergency problems only arises for low-symmetry and high 
number of
magnetic elements. I didn't have such problems before and I 
wonder if we

could still use old mixer scheme in such situations. Looking at the
userguide, it seems that the mixer does not allow to do as before 
and

PRATT mixer is too slow.

Did you encounter similar difficulties (which were not in older 
WIEN2k

versions)?

Best Regards

Xavier

Here is the case.struct:

blebleble
P   LATTICE,NONEQUIV.ATOMS: 16 1_P1
MODE OF CALC=RELA unit=bohr
  14.167163  6.46 11.993298 90.000

Re: [Wien] Mixer surprise when using PBE0 hybrid on-site functional

2017-01-20 Thread Xavier Rocquefelte

Dear Laurence

Thank you so much for your detailled replies.

I agree that something curious happens here. In particular, my surprise 
is why the convergency is fast and leads to a ferromagnetic solution in 
GGA+U and not in PBE0 on-site hybrid. These two schemes must be quite 
similar in the way they correct the GGA eigenvalues. I will continue to 
test the different options of mixer. Just one question, I didn't know 
the :MV keyword. Where should I find it?


Best Regards

Xavier

Le 20/01/2017 à 22:16, Laurence Marks a écrit :
I can provide some partial responses, although there are also some 
things that I don't understand. Some of this (maybe most) is not the 
mixer but in other parts of Wien2k.


First, the old (2008) version is there if you use MSEC1, but I have 
not tested it and it may fail. Better is to use MSEC3 which is almost 
the old version. For some classes of problems this is more stable than 
MSR1, and works better. If you are talking about the pre-multisecant 
version (BROYD) that vanished some time ago.


Second, there is a nasty "feature" particularly for +U (eece) cases, 
which is partially discussed in the mixer Readme. There is no 
guarantee that a solution exists -- the KS theorem is for densities 
but U is an orbital term. It is very possible to have cases where 
there is no fixed-point solution. The older MSEC1 (maybe BROYD) could 
find a fake solution where the density was consistent but the orbital 
potential was not. The latest version is much better in avoiding them 
and going for "real" solutions rather than being trapped. For orbital 
potentials it is very important to look at :MV to check that one 
really has a self-consistent orbital potential.


Third, there are cases where PBE (and all the GGA's in Wien2k that I 
have tested) give unphysical results when applied to isolated d or f 
electrons as done for -eece. I guess that the GGA functionals were not 
designed for the densities of just high L orbitals. This leads to very 
bad behavior of the mixing. I know of no way to solve this in the 
mixer, it is a structural problem. It goes away if LDA is used as the 
form for VXC in -eece.


Fourth, larger problem with low symmetry (P1 in particular) can 
certainly behave badly. Part of this might be "somewhere" in Wien2k 
coding, part of it is generic to a low symmetry problem. In many cases 
these have small eigenvalues in the mixing Jacobian which are removed 
when symmetry is imposed. All one can do is use MSEC3 or some of the 
additional flags (see the mixer README) such as "SLOW".


Fifth...probably exists, but I can't think of it immediately.

On Fri, Jan 20, 2017 at 2:03 PM, Xavier Rocquefelte 
<xavier.rocquefe...@univ-rennes1.fr 
<mailto:xavier.rocquefe...@univ-rennes1.fr>> wrote:


Dear Colleagues

I did recently a calculation which has been published long time ago
using a old WIEN2k version (in 2008).

It corresponds to a spin-polarized calculation for the compound
CuO. The
symmetry is removed and the idea is to estimate the total energies for
different magnetic orders to extract magnetic couplings from a mapping
analysis. Such calculations were converging fastly without any trouble
in 2008.

Here I have started from the scratch with a case.cif file to generate
the case.struct file and initializing the calculation in a
standard manner.

Then I wanted to have the energy related to a ferromagnetic situation
(not the more stable). I have 8 copper sites in the unit cell I am
using.

When this calculation is done using PBE+U everything goes fine.
However
when PBE0 hybrid on-site functional is used we observed
oscillations and
the magnetic moment disappear, which is definitely not correct. It
should be mentionned that the convergency is really bad. If we do a
similar calculation on the cristallographic unit cell (2 copper sites
only) the calculations converge both in PBE+U and PBE0.

The convergency problems only arises for low-symmetry and high
number of
magnetic elements. I didn't have such problems before and I wonder
if we
could still use old mixer scheme in such situations. Looking at the
userguide, it seems that the mixer does not allow to do as before and
PRATT mixer is too slow.

Did you encounter similar difficulties (which were not in older WIEN2k
versions)?

Best Regards

Xavier

Here is the case.struct:

blebleble
P   LATTICE,NONEQUIV.ATOMS: 16 1_P1
MODE OF CALC=RELA unit=bohr
  14.167163  6.46 11.993298 90.00 95.267000 90.00
ATOM  -1: X=0.8750 Y=0.7500 Z=0.8750
   MULT= 1  ISPLIT= 8
Cu NPT=  781  R0=0.5000 RMT=1.9700   Z: 29.0
LOCAL ROT MATRIX:1.000 0.000 0.000
  0.000 1.000 0.000
  0.000 0.000

[Wien] Mixer surprise when using PBE0 hybrid on-site functional

2017-01-20 Thread Xavier Rocquefelte

Dear Colleagues

I did recently a calculation which has been published long time ago 
using a old WIEN2k version (in 2008).


It corresponds to a spin-polarized calculation for the compound CuO. The 
symmetry is removed and the idea is to estimate the total energies for 
different magnetic orders to extract magnetic couplings from a mapping 
analysis. Such calculations were converging fastly without any trouble 
in 2008.


Here I have started from the scratch with a case.cif file to generate 
the case.struct file and initializing the calculation in a standard manner.


Then I wanted to have the energy related to a ferromagnetic situation 
(not the more stable). I have 8 copper sites in the unit cell I am using.


When this calculation is done using PBE+U everything goes fine. However 
when PBE0 hybrid on-site functional is used we observed oscillations and 
the magnetic moment disappear, which is definitely not correct. It 
should be mentionned that the convergency is really bad. If we do a 
similar calculation on the cristallographic unit cell (2 copper sites 
only) the calculations converge both in PBE+U and PBE0.


The convergency problems only arises for low-symmetry and high number of 
magnetic elements. I didn't have such problems before and I wonder if we 
could still use old mixer scheme in such situations. Looking at the 
userguide, it seems that the mixer does not allow to do as before and 
PRATT mixer is too slow.


Did you encounter similar difficulties (which were not in older WIEN2k 
versions)?


Best Regards

Xavier

Here is the case.struct:

blebleble
P   LATTICE,NONEQUIV.ATOMS: 16 1_P1
MODE OF CALC=RELA unit=bohr
 14.167163  6.46 11.993298 90.00 95.267000 90.00
ATOM  -1: X=0.8750 Y=0.7500 Z=0.8750
  MULT= 1  ISPLIT= 8
Cu NPT=  781  R0=0.5000 RMT=1.9700   Z: 29.0
LOCAL ROT MATRIX:1.000 0.000 0.000
 0.000 1.000 0.000
 0.000 0.000 1.000
ATOM  -2: X=0.1250 Y=0.2500 Z=0.6250
  MULT= 1  ISPLIT= 8
Cu NPT=  781  R0=0.5000 RMT=1.9700   Z: 29.0
LOCAL ROT MATRIX:1.000 0.000 0.000
 0.000 1.000 0.000
 0.000 0.000 1.000
ATOM  -3: X=0.1250 Y=0.2500 Z=0.1250
  MULT= 1  ISPLIT= 8
Cu NPT=  781  R0=0.5000 RMT=1.9700   Z: 29.0
LOCAL ROT MATRIX:1.000 0.000 0.000
 0.000 1.000 0.000
 0.000 0.000 1.000
ATOM  -4: X=0.8750 Y=0.7500 Z=0.3750
  MULT= 1  ISPLIT= 8
Cu NPT=  781  R0=0.5000 RMT=1.9700   Z: 29.0
LOCAL ROT MATRIX:1.000 0.000 0.000
 0.000 1.000 0.000
 0.000 0.000 1.000
ATOM  -5: X=0.6250 Y=0.2500 Z=0.6250
  MULT= 1  ISPLIT= 8
Cu NPT=  781  R0=0.5000 RMT=1.9700   Z: 29.0
LOCAL ROT MATRIX:1.000 0.000 0.000
 0.000 1.000 0.000
 0.000 0.000 1.000
ATOM  -6: X=0.3750 Y=0.7500 Z=0.8750
  MULT= 1  ISPLIT= 8
Cu NPT=  781  R0=0.5000 RMT=1.9700   Z: 29.0
LOCAL ROT MATRIX:1.000 0.000 0.000
 0.000 1.000 0.000
 0.000 0.000 1.000
ATOM  -7: X=0.3750 Y=0.7500 Z=0.3750
  MULT= 1  ISPLIT= 8
Cu NPT=  781  R0=0.5000 RMT=1.9700   Z: 29.0
LOCAL ROT MATRIX:1.000 0.000 0.000
 0.000 1.000 0.000
 0.000 0.000 1.000
ATOM  -8: X=0.6250 Y=0.2500 Z=0.1250
  MULT= 1  ISPLIT= 8
Cu NPT=  781  R0=0.5000 RMT=1.9700   Z: 29.0
LOCAL ROT MATRIX:1.000 0.000 0.000
 0.000 1.000 0.000
 0.000 0.000 1.000
ATOM  -9: X=0.8750 Y=0.4184 Z=0.6250
  MULT= 1  ISPLIT= 8
O  NPT=  781  R0=0.0001 RMT=1.6900   Z: 8.0
LOCAL ROT MATRIX:1.000 0.000 0.000
 0.000 1.000 0.000
 0.000 0.000 1.000
ATOM -10: X=0.1250 Y=0.9184 Z=0.8750
  MULT= 1  ISPLIT= 8
O  NPT=  781  R0=0.0001 RMT=1.6900   Z: 8.0
LOCAL ROT MATRIX:1.000 0.000 0.000
 0.000 1.000 0.000
 0.000 0.000 1.000
ATOM -11: X=0.1250 Y=0.5816 Z=0.3750
  MULT= 1  ISPLIT= 8
O  NPT=  781  R0=0.0001 RMT=1.6900   Z: 8.0
LOCAL ROT MATRIX:1.000 0.000 0.000
 0.000 1.000 0.000
 0.000 

Re: [Wien] Optic with and without SO

2017-01-13 Thread Xavier Rocquefelte
Could you give more details ... for instance the sequence of 
calculations you did to obtain epsilon2 in both cases.



Le 13/01/2017 à 20:24, Nacir GUECHI a écrit :
Thanks for your response. The calculation was done without SP in the 2 
cases.

/**
*Nacir GUECHI*
/
/Dr. Physique de la matière solide./
/Enseignant-Chercheur à l'université du Dr. Yahia FARES de Médéa 
<http://www.univ-medea.dz/fr/>,  Algeria (www.*univ-media*.dz)./
/Laboratoire d'études des surfaces et interfaces des matériaux solides 
(L.E.S.I.M.S), université Sétif1 (Algeria)/

/http://laboratoires.univ-setif.dz/L.E.S.I.M.S//


Le Vendredi 13 janvier 2017 20h04, Xavier Rocquefelte 
<xavier.rocquefe...@univ-rennes1.fr> a écrit :



I imagine that you have done a spin-polarized calculations. If so, you 
must sum the spin-up and spin-down contributions when you are doing 
the epsilon2 spectrum without SO.

Cheers
Xavier

Le 13/01/2017 à 19:06, Nacir GUECHI a écrit :

Dear Professor Peter Blaha and Wien2k users.
I calculated the optical properties of a compound without SO effect 
then I recalculated it with SO effect. I found that the intensity of 
peaks in the epsillon2 spectrum of the case with SO is practically 
double than that of the case without So. My query: is this normal ? 
which case we should take?

/**
*Nacir GUECHI*
/
/Dr. Physique de la matière solide./
/Enseignant-Chercheur à l'université du Dr. Yahia FARES de Médéa 
<http://www.univ-medea.dz/fr/>,  Algeria (www.*univ-media*.dz)./
/Laboratoire d'études des surfaces et interfaces des matériaux 
solides (L.E.S.I.M.S), université Sétif1 (Algeria)/

/http://laboratoires.univ-setif.dz/L.E.S.I.M.S//


___
Wien mailing list
Wien@zeus.theochem.tuwien.ac.at <mailto:Wien@zeus.theochem.tuwien.ac.at>
http://zeus.theochem.tuwien.ac.at/mailman/listinfo/wien
SEARCH the MAILING-LIST 
at:http://www.mail-archive.com/wien@zeus.theochem.tuwien.ac.at/index.html



___
Wien mailing list
Wien@zeus.theochem.tuwien.ac.at <mailto:Wien@zeus.theochem.tuwien.ac.at>
http://zeus.theochem.tuwien.ac.at/mailman/listinfo/wien
SEARCH the MAILING-LIST at: 
http://www.mail-archive.com/wien@zeus.theochem.tuwien.ac.at/index.html





___
Wien mailing list
Wien@zeus.theochem.tuwien.ac.at
http://zeus.theochem.tuwien.ac.at/mailman/listinfo/wien
SEARCH the MAILING-LIST at:  
http://www.mail-archive.com/wien@zeus.theochem.tuwien.ac.at/index.html


___
Wien mailing list
Wien@zeus.theochem.tuwien.ac.at
http://zeus.theochem.tuwien.ac.at/mailman/listinfo/wien
SEARCH the MAILING-LIST at:  
http://www.mail-archive.com/wien@zeus.theochem.tuwien.ac.at/index.html


Re: [Wien] Optic with and without SO

2017-01-13 Thread Xavier Rocquefelte
I imagine that you have done a spin-polarized calculations. If so, you 
must sum the spin-up and spin-down contributions when you are doing the 
epsilon2 spectrum without SO.


Cheers

Xavier


Le 13/01/2017 à 19:06, Nacir GUECHI a écrit :

Dear Professor Peter Blaha and Wien2k users.
I calculated the optical properties of a compound without SO effect 
then I recalculated it with SO effect. I found that the intensity of 
peaks in the epsillon2 spectrum of the case with SO is practically 
double than that of the case without So. My query: is this normal ? 
which case we should take?

/**
*Nacir GUECHI*
/
/Dr. Physique de la matière solide./
/Enseignant-Chercheur à l'université du Dr. Yahia FARES de Médéa 
,  Algeria (www.*univ-media*.dz)./
/Laboratoire d'études des surfaces et interfaces des matériaux solides 
(L.E.S.I.M.S), université Sétif1 (Algeria)/

/http://laboratoires.univ-setif.dz/L.E.S.I.M.S//


___
Wien mailing list
Wien@zeus.theochem.tuwien.ac.at
http://zeus.theochem.tuwien.ac.at/mailman/listinfo/wien
SEARCH the MAILING-LIST at:  
http://www.mail-archive.com/wien@zeus.theochem.tuwien.ac.at/index.html


___
Wien mailing list
Wien@zeus.theochem.tuwien.ac.at
http://zeus.theochem.tuwien.ac.at/mailman/listinfo/wien
SEARCH the MAILING-LIST at:  
http://www.mail-archive.com/wien@zeus.theochem.tuwien.ac.at/index.html


Re: [Wien] it possible to apply first DFT+U and then mBJ+U?

2016-11-27 Thread Xavier Rocquefelte

Dear Bhamu

In the present case, you are dealing with Ag+ ions and thus a d10 
electronic configuration for silver.


In such a case applying a Hubbard correction will mainly lead to correct 
the position of the Ag(4d) states which are all occupied and below the 
O(2p) band. It should not affect so much the band gap. Do you have the 
band gap values when doing PBE only?


I would recommend to do:

- PBE

- PBE+mBJ

- and if you want to see the impact of the proper description of the 4d 
band of silver which is too high in energy when doing PBE only, you can 
apply an Hubbard term


You must also consider the impact of relativistic treatment. For silver, 
the spin-orbit coupling will lead to split the p1/2 and p3/2 states 
which could contribute to the conduction band.


Best Regards

Xavier


Le 27/11/2016 à 16:19, Dr. K. C. Bhamu a écrit :


Dear experts
I did a calculation for hexagonal AgAlO2 and found that if I do first 
PBE+U and then apply mBJ+ U, it give smart band gap value which is 
very close to experimental band gap.


But I do not know whether we can apply this U twice or not.
The DFT+U calculation underestimates and only mBJ slightly 
overestimate the experimental band gap for this system.


I did the calculation for -ec 0.1 -cc 0.0002 -fc 2 (setrmt -r 5).

Any comment will be helpful.

Sincerely
Bhamu



___
Wien mailing list
Wien@zeus.theochem.tuwien.ac.at
http://zeus.theochem.tuwien.ac.at/mailman/listinfo/wien
SEARCH the MAILING-LIST at:  
http://www.mail-archive.com/wien@zeus.theochem.tuwien.ac.at/index.html


___
Wien mailing list
Wien@zeus.theochem.tuwien.ac.at
http://zeus.theochem.tuwien.ac.at/mailman/listinfo/wien
SEARCH the MAILING-LIST at:  
http://www.mail-archive.com/wien@zeus.theochem.tuwien.ac.at/index.html


Re: [Wien] overestimated band gap by PBE

2016-11-21 Thread Xavier Rocquefelte

Dear Bhamu

First of all, your band gap to be accurate need many kpoints when taken 
from the SCF file.


The best is to plot the band structure. Then you will have the 
fundamental band gap, not the optical band gap (usually larger).


In your case, you must also include the spin-orbit coupling to properly 
treat the lead states which define the band gap.


Including the spin-orbit coupling will reduce your band gap and thus you 
will reach the GGA underestimation ;)


Cheers

Xavier


Le 21/11/2016 à 13:25, Dr. K. C. Bhamu a écrit :

Dear Prof. Peter,

This is in the queue on my previous query regarding CH3NH3PbI3:

I used two strategy and got different results:


The experimental band gap is 1.67 eV for orthorhombic lead halide 
perovskite.


I ran two cases (with PBE);
1. with 1000 k, div: ( 11  7 11) and "min -j 'run_lapw -p -I -NI -i 
120 *-ec *0.1 *-ec * 0.0001 -fc 1
2. with 400 k, div: (  8  5  8)   and   min -j 'run_lapw -p -I -NI -i 
120 *-ec * 0.0001 *-ec * 0.0005 -fc 2'


-ec switch is used twice instead of ec and cc (it is by mistake)


Results:

1. GAP 1.687 eV,FER: 0.1030934295, ENE: -339059.11079128
2. GAP  1.777 eV,   FER: 0.1004906191, ENE: -339059.12432403

In case 2 with less k-points and normal scf criteria (-ec 0.0001 -ec 
0.0005 -fc 2), I got minimized ENE than case 1. But the band gap is 
good from case 1.


You see that band gap is in the error of /+- 0.01eV.


I did nothing special, just reduced rmt (5% and further rmt of "I" was 
reduced by 0.02).
Please correct me what is wrong here. Because usually PBE always 
underestimates the experimental band gap.


Kind regards
Bhamu


___
Wien mailing list
Wien@zeus.theochem.tuwien.ac.at
http://zeus.theochem.tuwien.ac.at/mailman/listinfo/wien
SEARCH the MAILING-LIST at:  
http://www.mail-archive.com/wien@zeus.theochem.tuwien.ac.at/index.html


___
Wien mailing list
Wien@zeus.theochem.tuwien.ac.at
http://zeus.theochem.tuwien.ac.at/mailman/listinfo/wien
SEARCH the MAILING-LIST at:  
http://www.mail-archive.com/wien@zeus.theochem.tuwien.ac.at/index.html


Re: [Wien] degeneracy problem in dipole matrix elements

2016-11-13 Thread Xavier Rocquefelte

Dear Mengxi Wu

If I clearly understand you, you are considering the GAMMA point where 
the bands are degenerate.


At the GAMMA point these bands have the same symmetry to my point of 
view, explaining why the dipole matrix is non-zero. Thus your result is 
expected to my point of view.


Best Regards

Xavier


Le 13/11/2016 à 17:29, Little Grass a écrit :

Dear WIEN2k users,

I'm trying to study the dipole matrix elements of MgO at Gamma point, 
but the dipole matrix elements between the degenerated bands seem to 
have been mixed together. Are there any ways to sort out the 
degeneracy, so that the dipole matrix elements are non-zero only for 
bands with the same symmetry?



The band structure can be generated from the spaghetti program. In 
addition, bands with different symmetries are colored differently with 
irrep.  For example, Fig1 shows the band structure from X to Gamma to 
X. According to symmetry, only the bands with the same color will be 
dipole allowed in px. So we expect that the dipole matrix elements (in 
file mommat2) for px is zero between bands with different colors. For 
example, the dipole matrix elements between band 6 and band 9 is 
non-zero, but the dipole matrix elements between band 7 and 9 or 8 and 
9 are zero. But if one looks at the px component of the dipole matrix 
element from the optic program, we see that at Gamma point, the dipole 
matrix elements between the degenerated bands are mixed together. For 
example, dipole matrix elements between band 6 and 9, between 7 and 9, 
and between 8 and 9 are all non-zero. In other words, the dipole 
matrix is much denser than it should be.
So is there a way to fix the dipole matrix so that the dipole matrix 
become very sparse and only bands with the same symmetry have non-zero 
dipole matrix elements?


Fig1:http://imgur.com/a/c7p0u

Thanks in advance for the help.
Best,
Mengxi Wu


___
Wien mailing list
Wien@zeus.theochem.tuwien.ac.at
http://zeus.theochem.tuwien.ac.at/mailman/listinfo/wien
SEARCH the MAILING-LIST at:  
http://www.mail-archive.com/wien@zeus.theochem.tuwien.ac.at/index.html


___
Wien mailing list
Wien@zeus.theochem.tuwien.ac.at
http://zeus.theochem.tuwien.ac.at/mailman/listinfo/wien
SEARCH the MAILING-LIST at:  
http://www.mail-archive.com/wien@zeus.theochem.tuwien.ac.at/index.html


Re: [Wien] charge density

2016-11-13 Thread Xavier Rocquefelte

Could you clarify your question. What do you mean by "each range"?

- Energy range

- Density value



Le 13/11/2016 à 16:54, boudiaf khadidja a écrit :

Dear Wien2k user
I use VIETA to plot the charge density ;But I need to know a way to 
plot or get the values of each range for the charge density; What can 
i do ?

Best regardes.


___
Wien mailing list
Wien@zeus.theochem.tuwien.ac.at
http://zeus.theochem.tuwien.ac.at/mailman/listinfo/wien
SEARCH the MAILING-LIST at:  
http://www.mail-archive.com/wien@zeus.theochem.tuwien.ac.at/index.html


___
Wien mailing list
Wien@zeus.theochem.tuwien.ac.at
http://zeus.theochem.tuwien.ac.at/mailman/listinfo/wien
SEARCH the MAILING-LIST at:  
http://www.mail-archive.com/wien@zeus.theochem.tuwien.ac.at/index.html


Re: [Wien] Query about SOC in WIEN2k

2016-11-05 Thread Xavier Rocquefelte

Dear R. Chouhan

Just to complete the very nice answer of Peter. I have used the force 
theorem as explained by Peter using GGA+U to estimate MCA and in the 
cases I have considered it works amazingly nicely.


As Peter said, I was using P1 symmetry and checking carefully the 
convergency before including the spin-orbit effect along specific 
directions.


Here is an example of what we obtained for CuO (Fig. 3). Similar results 
are obtained using on-site hybrid of GGA+U.


http://www.nature.com/articles/ncomms3511

Best Regards

Xavier


Le 05/11/2016 à 17:43, Peter Blaha a écrit :

There are a lot of problems in your calculations:

Usually the energy difference between 2 such calculations is extremely 
small and one must be very careful to get meaningful numbers.


When you do initso  for a spinpolarized system, the symmetry can be 
reduced. In such cases you MUST take the newly generated struct file.


> Do you want to use the new structure for SO calculations ? (y/N)   N

This is the wrong answer !

In fact, one should even do the calculations for the different 
magnetization directions with the same symmetry (eventually in P1). 
Thus checkout what symmetries you get for the different directions and 
choose the one which is compatible with all cases.
Check your case.inorb/dmc files in case that the number of atoms has 
been changed.


Important:
Increase Emax in case.in1 to a VERY large number (10 Ry), and test 
your results with respect to this EMAX.
Increase the k-mesh (for the final low-symmetry struct file) to a very 
good one AND TEST the convergence. of your results.


With such a struct file you run again (in the same directory) the 
non-so calculation to selfconsistency with a good k-mesh and good 
convergence:

runsp -orb -ec 0.01 -cc 0.01

Since the E differences are very small, set the DEs  in case.in1 to a 
smaller value like:

0   -4.09  .0001 STOP 1
   ^^^  DE

Once the non-so calculation is converged:

save_lapw non_so

Now the force theorem:
select  001 direction in case.inso
x lapw1 -up/dn
x lapwso -up -orb
x lapw2 -so -up/dn
cp case.scf2up/dn 001_kmesh_emax.scf2up/dn

select other direction(s) and repeat the above steps.
Compare the band energies listed in the scf2xx files.

for the energy:
select 001 direction
runsp  -so -orb -ec 0.01 -cc 0.01
save 001_kmesh_emax

restore non_so
do everything again for another direction.

Make sure to check EMX and kmesh convergence.

PS: I've never done MCA calculations using GGA+U !!

Am 05.11.2016 um 06:26 schrieb Rajiv Chouhan:

Dear Dr. P. Blaha,

I am trying to calculate the magnetic anisotropy energy (MAE) for the
magnetic system Nd-metal. For this I tried couple of methods (energy
theorem and force theorem) to calculate MAE.

Energy theorem:

step-1: Performed GGA+U ; Step-2: executed initso_lapw  for different
directions 001, 100 etc ; Step-3: Didn't put RLO (relativistic local
orbital in the .inso file)

Step-4: Performed runsp_lapw -so -orb -dm  -ec 0.01 ; Step-5 Took
the difference of energies E[001] - E[100]

The above method give correct result in very few cases. Am I doing wrong
somewhere?


Force Theorem:

Will you please explain the steps correctly because I am confused while
doing the steps. I followed the below steps and this is not working 
for me.


Step-1: Performed GGA+U (runsp_lapw -dm -orb -ec 0.01);

Step-2: initso_lapw (by taking below actions)

>Add RLO for NONE, ALL, CHOOSE elements? (N/a/c) : N

Do you have a spinpolarized case (and want to run symmetso) ? (y/N) Y

Do you want to use the new structure for SO calculations ? (y/N)   N

Step-3: x lapwso -up

Step-4: x lapw2 -up

Step-5: x lapw2 -dn

Step-6:  MAE = 001(sum of eigenvalues up & down) - 100(sum of
eigenvalues up & down)

This force method is also not working properly. Will you please correct
me where I am doing wrong.

I also followed the following links for my above steps:

https://www.mail-archive.com/wien@zeus.theochem.tuwien.ac.at/msg09408.html 



http://zeus.theochem.tuwien.ac.at/pipermail/wien/2010-September/013730.html 



Thank you,
R. Chouhan


___
Wien mailing list
Wien@zeus.theochem.tuwien.ac.at
http://zeus.theochem.tuwien.ac.at/mailman/listinfo/wien
SEARCH the MAILING-LIST at: 
http://www.mail-archive.com/wien@zeus.theochem.tuwien.ac.at/index.html






___
Wien mailing list
Wien@zeus.theochem.tuwien.ac.at
http://zeus.theochem.tuwien.ac.at/mailman/listinfo/wien
SEARCH the MAILING-LIST at:  
http://www.mail-archive.com/wien@zeus.theochem.tuwien.ac.at/index.html


Re: [Wien] What does it mean the commment " Unmatched " during SCF cycle

2016-11-02 Thread Xavier Rocquefelte

Dear Colleague

The option "-o" does not exist.

It seems that you wanted to put "-so" ... Is it the case ?

Cheers

Xavier


Le 02/11/2016 à 14:24, Abderrahmane Reggad a écrit :

Thanks Dr Assmann for your quick reply

I repetad the calculation and I same problem and I want to note that I 
modified the optimization script as follows:


runsp_c_lapw -orb -o -ec 0.0001

Perhaps it's related to the use of "runsp_c_lapw " because it's the 
first time I used it


I want to know if it's just a remark or an error.

Best regards
--
Mr: A.Reggad
Laboratoire de Génie Physique
Université Ibn Khaldoun - Tiaret
Algerie




___
Wien mailing list
Wien@zeus.theochem.tuwien.ac.at
http://zeus.theochem.tuwien.ac.at/mailman/listinfo/wien
SEARCH the MAILING-LIST at:  
http://www.mail-archive.com/wien@zeus.theochem.tuwien.ac.at/index.html


___
Wien mailing list
Wien@zeus.theochem.tuwien.ac.at
http://zeus.theochem.tuwien.ac.at/mailman/listinfo/wien
SEARCH the MAILING-LIST at:  
http://www.mail-archive.com/wien@zeus.theochem.tuwien.ac.at/index.html


Re: [Wien] Which notation for the spin direction in hexagonal structure

2016-10-09 Thread Xavier Rocquefelte

Dear Colleague

The notation is with three indices.

It will be along the directions defined in your case.struct file.

In other words, if you put 0 0 1, it will be along the c-axis of your 
case.struct file.


Best Regards

Xavier



Le 09/10/2016 à 12:52, Abderrahmane Reggad a écrit :

Dear Wien2k users

I want to introduce the spin-orbit coupling in my calculation for a 
hexagonal structure.


Would you inform me which notation should we use to introduce the spin 
direction : that of 3 indicesor that of 4 indices.


NB: The spin direction for my case is the z direction

Best regards

--
Mr: A.Reggad
Laboratoire de Génie Physique
Université Ibn Khaldoun - Tiaret
Algerie




___
Wien mailing list
Wien@zeus.theochem.tuwien.ac.at
http://zeus.theochem.tuwien.ac.at/mailman/listinfo/wien
SEARCH the MAILING-LIST at:  
http://www.mail-archive.com/wien@zeus.theochem.tuwien.ac.at/index.html


___
Wien mailing list
Wien@zeus.theochem.tuwien.ac.at
http://zeus.theochem.tuwien.ac.at/mailman/listinfo/wien
SEARCH the MAILING-LIST at:  
http://www.mail-archive.com/wien@zeus.theochem.tuwien.ac.at/index.html


Re: [Wien] Error in initialising GGA+U

2016-08-04 Thread Xavier Rocquefelte

Do you have an inversion center in your space group?
If yes, you should use a case.indmc file and not case.indm.
Cheers
Xavier


Le 04/08/2016 à 06:36, Shakeel Khandy a écrit :

Dear Xavier

I tried as you suggested, but only case.inorb got created and next it 
showed again the same message. Then i manually created the case.indm 
file, the scf runs only a single cycle . Thereafter showing a Stop message



Thanks

On Tue, Aug 2, 2016 at 1:45 PM, Xavier Rocquefelte 
<xavier.rocquefe...@univ-rennes1.fr 
<mailto:xavier.rocquefe...@univ-rennes1.fr>> wrote:


Dear Shakeel

You must create two files when initializing a GGA+U (or LDA+U)
calculation, i.e. case.inorb and case.indm (or case.indmc if no
inversion center in the space group).

Normally when you start such a calculation from the w2web
interface, the interface open a template file to allow you to
create properly the case.indm file.

How did you proceed? Look at the userguide and the manual of Pawel
Novak in the website. You will find more information.

Cheers

Xavier


Le 02/08/2016 à 09:27, Shakeel Khandy a écrit :


Dear Users,


I have tried to run the GGA+U on FeVTiAl, After running a normal
GGA cycle i initialised for GG+ U, But it showed an error message as
Can't read file FeVTiAl_u.indmc.

Only *FeVTiAl.inorb* file is created and then .indm is missing

Please help me to sort out this

With Kind Regards
Shakeel


___
Wien mailing list
Wien@zeus.theochem.tuwien.ac.at
<mailto:Wien@zeus.theochem.tuwien.ac.at>
http://zeus.theochem.tuwien.ac.at/mailman/listinfo/wien
SEARCH the MAILING-LIST 
at:http://www.mail-archive.com/wien@zeus.theochem.tuwien.ac.at/index.html



___
Wien mailing list
Wien@zeus.theochem.tuwien.ac.at
<mailto:Wien@zeus.theochem.tuwien.ac.at>
http://zeus.theochem.tuwien.ac.at/mailman/listinfo/wien
SEARCH the MAILING-LIST at:
http://www.mail-archive.com/wien@zeus.theochem.tuwien.ac.at/index.html





___
Wien mailing list
Wien@zeus.theochem.tuwien.ac.at
http://zeus.theochem.tuwien.ac.at/mailman/listinfo/wien
SEARCH the MAILING-LIST at:  
http://www.mail-archive.com/wien@zeus.theochem.tuwien.ac.at/index.html


___
Wien mailing list
Wien@zeus.theochem.tuwien.ac.at
http://zeus.theochem.tuwien.ac.at/mailman/listinfo/wien
SEARCH the MAILING-LIST at:  
http://www.mail-archive.com/wien@zeus.theochem.tuwien.ac.at/index.html


Re: [Wien] Error in initialising GGA+U

2016-08-02 Thread Xavier Rocquefelte

Dear Shakeel

You must create two files when initializing a GGA+U (or LDA+U) 
calculation, i.e. case.inorb and case.indm (or case.indmc if no 
inversion center in the space group).


Normally when you start such a calculation from the w2web interface, the 
interface open a template file to allow you to create properly the 
case.indm file.


How did you proceed? Look at the userguide and the manual of Pawel Novak 
in the website. You will find more information.


Cheers

Xavier


Le 02/08/2016 à 09:27, Shakeel Khandy a écrit :


Dear Users,


I have tried to run the GGA+U on FeVTiAl, After running a normal GGA 
cycle i initialised for GG+ U, But it showed an error message as

Can't read file FeVTiAl_u.indmc.

Only *FeVTiAl.inorb* file is created and then .indm is missing

Please help me to sort out this

With Kind Regards
Shakeel


___
Wien mailing list
Wien@zeus.theochem.tuwien.ac.at
http://zeus.theochem.tuwien.ac.at/mailman/listinfo/wien
SEARCH the MAILING-LIST at:  
http://www.mail-archive.com/wien@zeus.theochem.tuwien.ac.at/index.html


___
Wien mailing list
Wien@zeus.theochem.tuwien.ac.at
http://zeus.theochem.tuwien.ac.at/mailman/listinfo/wien
SEARCH the MAILING-LIST at:  
http://www.mail-archive.com/wien@zeus.theochem.tuwien.ac.at/index.html


Re: [Wien] which file I can delete

2016-04-24 Thread Xavier Rocquefelte

Dear Dr. Bhamu
you forget to put "-d" which explain why you did not restore from the 
files contained in the directory.

You must use the command

restore_lapw -d directory

More details, here:
http://100.36.165.205/usersguide/5_Shell_scripts.html#SECTION05222000

Regards
Xavier


Le 24/04/2016 22:25, Dr. K. C. Bhamu a écrit :

Dear Prof Tran

restore_lapw -f case did not restore the case.scf file in parent DIR. 
In parent DIR I have only case.scf0, case.scf1 and so on but not 
case.scf which was save in a DIR when I used save_lapw.


any issue or I should copy it manually?

regards




Dr. K. C. Bhamu
(UGC-Dr. D. S. Kothari Postdoc Fellow)
Department of Physics
Goa University, Goa-403 206
India
Mob. No.  +91-9975238952

On Sat, Apr 23, 2016 at 1:28 PM, > wrote:


Hi,

When a calculation is completed, it is recommended to use
save_lapw to save the important files (inputs, scf, clmsum)
under a new name. Since the vector file is not included in these
renamed files it should not be a problem to transfer them.

F. Tran

On Saturday 2016-04-23 09:10, Dr. K. C. Bhamu wrote:

Date: Sat, 23 Apr 2016 09:10:46
From: Dr. K. C. Bhamu >
Reply-To: A Mailing list for WIEN2k users
>
To: A Mailing list for WIEN2k users
>
Subject: [Wien] which file I can delete

Dear Wien2k users

I relaxed a structure and have couples of clmsum and vector
files which are very large and its very difficult to take them
from remote to local.
So please suggest is there any idea so that I can delete some
of these files (unnecessary large files) and by doing this I
can get files very easily from host.

Sincerely
Bhamu




___
Wien mailing list
Wien@zeus.theochem.tuwien.ac.at

http://zeus.theochem.tuwien.ac.at/mailman/listinfo/wien
SEARCH the MAILING-LIST at:
http://www.mail-archive.com/wien@zeus.theochem.tuwien.ac.at/index.html




___
Wien mailing list
Wien@zeus.theochem.tuwien.ac.at
http://zeus.theochem.tuwien.ac.at/mailman/listinfo/wien
SEARCH the MAILING-LIST at:  
http://www.mail-archive.com/wien@zeus.theochem.tuwien.ac.at/index.html


___
Wien mailing list
Wien@zeus.theochem.tuwien.ac.at
http://zeus.theochem.tuwien.ac.at/mailman/listinfo/wien
SEARCH the MAILING-LIST at:  
http://www.mail-archive.com/wien@zeus.theochem.tuwien.ac.at/index.html


Re: [Wien] negative epsilon 1

2016-01-10 Thread Xavier Rocquefelte
Dear Brik
What you found is usual and is simply the signature of a high absorption in 
this high energy region. 
Indeed, a simple way to understand the negative value of eps1 is to look at the 
following relation: 

eps1 = n^2 - k^2

with n the real part and k the imaginary part of the complex refractive index. 
k is the extinction coefficient and is related to absorption while n is related 
to diffusion. In a metal the refractive index AT LOW energy (in the IR region) 
is negative due to a very high absorption in this region. As a consequence k is 
large and thus eps1 is negative. 

For a semiconductor, you have no absorption in the IR and part of the visible 
range (depending on the gap value) but you can have strong absorption at higher 
energies leading to a similar feature. 
To conclude the fact that eps1 is negative is not a proof of a metallic 
character by itself. It will depend where you find this negative eps1 value, it 
should be at very low energy. 

Here is a reference in which I discussed this point based on chemist arguments. 
At this time I was working on the concept of optical channels. You will see on 
figure 1 the impact of changing the compound density on eps2 and eps1. Look to 
Figs. 1a and 1c. The absorption is exactly the same because the chromophore are 
the same. But this material is lamellar (2D) and I just increase the interlayer 
distance leading to have more vaccum in the system, thus lower density. The 
consequence is that for low density eps1 is always positive (k is never higher 
than n) while in for higher density k is negative after 5 eV, because k is 
larger than n due to a larger absorption (twice more in this case). 

http://pubs.acs.org/doi/abs/10.1021/jp055445u

Best Regards
Xavier

Brik Hamida  a écrit :

> Hi
> Im working on semiconductor material. I have calculated  the real part of
> dielectric function (epsilon1) as function of energy. For high energy , I
> found epsilon1 negative !
> Please can someone explain me what means this negative values ? I read that
> any material havig an negative epsilon1 , it becomes a metal , it is right
> ??
> Thanks in advance
> Brik
>___
Wien mailing list
Wien@zeus.theochem.tuwien.ac.at
http://zeus.theochem.tuwien.ac.at/mailman/listinfo/wien
SEARCH the MAILING-LIST at:  
http://www.mail-archive.com/wien@zeus.theochem.tuwien.ac.at/index.html


Re: [Wien] Graphene bandstructure

2015-12-23 Thread Xavier Rocquefelte
Dear Fhokrul
Your structure looks strange to me. The angle is 60° and not 120° and it seems 
that you do not take benefit of the symetry. 
Here is a cif file of graphene based on the graphite structure in which I have 
simply increase the c parameter. 
The structure you are using may explain why you still have a gap (symetry 
problem). 
Best Regards
Xavier

#==

# CRYSTAL DATA

#--

data_VESTA_phase_1

_pd_phase_name 'C'
_cell_length_a 2.45600
_cell_length_b 2.45600
_cell_length_c 15.0
_cell_angle_alpha  90
_cell_angle_beta   90
_cell_angle_gamma  120
_symmetry_space_group_name_H-M 'P 63 m c'
_symmetry_Int_Tables_number    186

loop_
_symmetry_equiv_pos_as_xyz
'x, y, z'
'-y, x-y, z'
'-x+y, -x, z'
'-x, -y, z+1/2'
'y, -x+y, z+1/2'
'x-y, x, z+1/2'
'-y, -x, z'
'-x+y, y, z'
'x, x-y, z'
'y, x, z+1/2'
'x-y, -y, z+1/2'
'-x, -x+y, z+1/2'

loop_
_atom_site_label
_atom_site_occupancy
_atom_site_fract_x
_atom_site_fract_y
_atom_site_fract_z
_atom_site_adp_type
_atom_site_B_iso_or_equiv
_atom_site_type_symbol
C1 1.0 0.00  0.00  0.00 Biso  1.00 C
C2 1.0 0.33  0.67  0.00 Biso  1.00 C

"Islam, Md F"  a écrit :

> Sorry, in my earlier email, I meant to say with 30 x 30 x 1 mesh, gap 
> is smaller but
> it doesn't close completely.
>
> Thanks,
> Fhokrul
>
>
> 
> From: wien-boun...@zeus.theochem.tuwien.ac.at 
> [wien-boun...@zeus.theochem.tuwien.ac.at] On Behalf Of Islam, Md F 
> [isl...@uta.edu]
> Sent: Wednesday, December 23, 2015 6:22 AM
> To: A Mailing list for WIEN2k users
> Subject: Re: [Wien] Graphene bandstructure
>
> Hi Prof Blaha,
>
>     Here is the structure I am using. I tried to be as precise as 
> possible with position coordinates.
> I think size of the vacuum is sufficiently large so that 
> supercell-supercell interaction is negligible.
> So far what I have got is that with LDA xc if I use 25 x 25 x 1 
> k-mesh there is a gap at the Fermi
> level (which passes through K point) but if I use 30 x 30 x 1 k-mesh, 
> gap closes but DOS is not
> zero at the Fermi level (about 0.001/eV). Please let me know if I am 
> doing anything wrong with
> this calculation.
>
>
> Thanks,
> Fhokrul
>
>
> Graphene
> P LATTICE,NONEQUIV.ATOMS: 2
> MODE OF CALC=RELA unit=bohr
> 4.647800 4.647800 30.00 90.00 90.00 60.00
> ATOM -1: X=0. Y=0. Z=0.5000
>              MULT= 1 ISPLIT= 8
> C             NPT= 781 R0=0.0001 RMT= 1.3200 Z: 6.0
> LOCAL ROT MATRIX:    1.000 0.000 0.000
>                                0.000 1.000 0.000
>                                0.000 0.000 1.000
> ATOM -2: X=0.6667 Y=0.6667 Z=0.5000
>              MULT= 1 ISPLIT= 8
> C             NPT= 781 R0=0.0001 RMT= 1.3200 Z: 6.0
> LOCAL ROT MATRIX:     1.000 0.000 0.000
>                                 0.000 1.000 0.000
>                                 0.000 0.000 1.000
> 12 NUMBER OF SYMMETRY OPERATIONS
> -1-1 0 0.
> 0 1 0 0.
> 0 0-1 0.
> 1
> -1-1 0 0.
> 0 1 0 0.
> 0 0 1 0.
> 2
> -1-1 0 0.
> 1 0 0 0.
> 0 0-1 0.
> 3
> -1-1 0 0.
> 1 0 0 0.
> 0 0 1 0.
> 4
> 0 1 0 0.
> -1-1 0 0.
> 0 0-1 0.
> 5
> 0 1 0 0.
> -1-1 0 0.
> 0 0 1 0.
> 6
> 0 1 0 0.
> 1 0 0 0.
> 0 0-1 0.
> 7
> 0 1 0 0.
> 1 0 0 0.
> 0 0 1 0.
> 8
> 1 0 0 0.
> -1-1 0 0.
> 0 0-1 0.
> 9
> 1 0 0 0.
> -1-1 0 0.
> 0 0 1 0.
> 10
> 1 0 0 0.
> 0 1 0 0.
> 0 0-1 0.
> 11
> 1 0 0 0.
> 0 1 0 0.
> 0 0 1 0.
> 12
>
>
>
>
> 
> From: wien-boun...@zeus.theochem.tuwien.ac.at 
> [wien-boun...@zeus.theochem.tuwien.ac.at] On Behalf Of Peter Blaha 
> [pbl...@theochem.tuwien.ac.at]
> Sent: Tuesday, December 22, 2015 3:29 PM
> To: A Mailing list for WIEN2k users
> Subject: Re: [Wien] Graphene bandstructure
>
> The two eigenvalues at K must be identical by symmetry.
>
> If you have a splitting of some meV it means that your structure is
> slightly wrong.
>
> Typical errors: positions of 1/3 and 2/3 must be given in full precision.
>   0.   is NOT 1/3
>
> Am 22.12.2015 um 20:31 schrieb Islam, Md F:
>> Hi Prof Blaha,
>>
>>      I have got the gap from plotting bandstructure as well as DOS 
>> calculations.
>> I can see the linear dispersion at the K point just like the way it 
>> is suppose to be
>> when I plot all the bands in eV range. But if 

Re: [Wien] elast for magnetic material + curie temp

2015-08-20 Thread Xavier Rocquefelte
About the Néel temperature, I know two main strategies: 
- If you know an analytical formula which relates TN with J values (J: magnetic 
exchange parameters of your system) then you can estimate TN from DFT 
calculations. It will require the calculation of all the J's values of your 
system. Such a calculation of TN is possible for simple systems for which the 
analytical equation is known (one-dimensional Heisenberg magnetic system). 
- In a more general case, you need to include the temperature fluctuation 
effects on your magnetic system.  To do this, you must go beyond DFT. For 
instance, doing Monte-Carlo calculations using DFT paramaters will allow you to 
estimate TN. 
Cheers
Xavier

Lyudmila Dobysheva lyuk...@mail.ru a écrit :

 20.08.2015 07:28, Murugan Sundareswari ?:
 Is it possible to calculate curie temperature with our code.
 I am not in the know of Curie temperature

 we are calculating the elastic constants at ambient condition for the
 above mentioned ferromagnetic materialsand still i have the doubt of
 using the structure file of magnetic or non-magnetic to do so?

 Making a calculational model that correctly simulate concrete samples 
 and their experimental data is the most important thing, it cannot be 
 done without knowing all tiny details of the task, and it takes, 
 imho, 80 % of time to work out it. After this, one can quickly 
 calculate properties, this is a routine work.
 So, nobody can answer you. We can only give you some small hints, for 
 example:
 On Tue, Aug 18, 2015 at 1:32 PM, Fecher, Gerhard fec...@uni-mainz.de wrote:
 Depends, do you like to calculate for above or below the Curie temperature ?

 Best wishes
   Lyudmila Dobysheva
 --
 Phys.-Techn. Institute of Ural Br. of Russian Ac. of Sci.
 426001 Izhevsk, ul.Kirova 132I am not in the know of Curie temperature
 RUSSIA
 --
 Tel.:7(3412) 432045(office), 722529(Fax)
 E-mail: l...@ftiudm.ru, lyuk...@mail.ru (office)
         lyuk...@gmail.com (home)
 Skype:  lyuka17 (home), lyuka18 (office)
 http://ftiudm.ru/content/view/25/103/lang,english/
 --
 ___
 Wien mailing list
 Wien@zeus.theochem.tuwien.ac.at
 http://zeus.theochem.tuwien.ac.at/mailman/listinfo/wien
 SEARCH the MAILING-LIST at:  
 http://www.mail-archive.com/wien@zeus.theochem.tuwien.ac.at/index.html
___
Wien mailing list
Wien@zeus.theochem.tuwien.ac.at
http://zeus.theochem.tuwien.ac.at/mailman/listinfo/wien
SEARCH the MAILING-LIST at:  
http://www.mail-archive.com/wien@zeus.theochem.tuwien.ac.at/index.html


Re: [Wien] need your help

2015-08-04 Thread Xavier Rocquefelte
You must also read this document produced by Stefaan Cottenier. 

http://www.wien2k.at/reg_user/textbooks/DFT_and_LAPW-2_cottenier.pdf

In particular p43 and the following pages. 
Regards
Xavier

sikander Azam sikander.physi...@gmail.com a écrit :

 Dear all
 When we plot the band structure so what the brillion zone symmetry shows
 i.e. lets suppose Gamma point (0 0 0) what this zero shows, these are u v w
 or h k l or x y z.
 regards
 sikander
___
Wien mailing list
Wien@zeus.theochem.tuwien.ac.at
http://zeus.theochem.tuwien.ac.at/mailman/listinfo/wien
SEARCH the MAILING-LIST at:  
http://www.mail-archive.com/wien@zeus.theochem.tuwien.ac.at/index.html


Re: [Wien] need your help

2015-08-04 Thread Xavier Rocquefelte
Dear Azam
Many excellent books could help you to understand how a band structure works. 
I recommend you to look at the following web sites: 

http://www.chem.uci.edu/~lawm/Hoffmann.pdf

http://www.chemeddl.org/alfresco/service/api/node/content/workspace/SpacesStore/addbf469-059f-44ff-beb5-3417c58ee56d/QBAND11.pdf?guest=true

Just a brief answer, but you must read about band structure before doing WIEN2k 
calculations to my point of view. Indeed this periodic code is based on k 
points. So ... the Gamma point is the center of the Brillouin zone which is in 
the reciprocal space, i.e. the space defined by the wave-vectors (k points). It 
allows to define Bloch functions which are a product of a molecular orbital 
(defined in the unit cell) and a periodic function (dependent on the k point). 

The Gamma point is defined by three numbers (0 0 0), meaning kx = 0, ky = 0 and 
kz = 0. Indeed, the reciprocal space has three dimension as the real space. In 
the case of a cubic symmetry (or orthorhombic) a* = 2.pi / a, b* = 2.pi /b and 
c* = 2.pi /c. I will not do all the teaching of the band structure theory but 
you can find all these details in books and websites. 

Good luck 
Xavier

sikander Azam sikander.physi...@gmail.com a écrit :

 Dear all
 When we plot the band structure so what the brillion zone symmetry shows
 i.e. lets suppose Gamma point (0 0 0) what this zero shows, these are u v w
 or h k l or x y z.
 regards
 sikander
___
Wien mailing list
Wien@zeus.theochem.tuwien.ac.at
http://zeus.theochem.tuwien.ac.at/mailman/listinfo/wien
SEARCH the MAILING-LIST at:  
http://www.mail-archive.com/wien@zeus.theochem.tuwien.ac.at/index.html


Re: [Wien] question regarding magetic moment

2015-07-19 Thread Xavier Rocquefelte
Dear Rishi
Indeed, WIEN2K provides numbers with 4 digits however it does not mean that the 
value is accurate up to 4 digits. 
The accuracy will depend on:
- the convergence criteria you are using. 
- the RMT you are using. Indeed the magnetic moment for each atom is estimated 
inside the muffin-tin radius you have defined. If you use small RMT you will 
have a significant discrepancy. 

In addition, you must think about the experimental accuracy. For magnetic 
moment, I usually consider 2 digits not more. 

Regards
Xavier

Rishi Singh rishising...@gmail.com a écrit :

 Dear wien users

 I am unable to know that what is accuracy of magnetic moment calculated by
 wien2k package. Is it true that its accuracy is not up to .0001

 Please help me regarding this.

 I will be highly obliged. Thanks in advance.



 kind regard

 Dr R P Singh
___
Wien mailing list
Wien@zeus.theochem.tuwien.ac.at
http://zeus.theochem.tuwien.ac.at/mailman/listinfo/wien
SEARCH the MAILING-LIST at:  
http://www.mail-archive.com/wien@zeus.theochem.tuwien.ac.at/index.html


Re: [Wien] Adding excitonic effects

2015-05-16 Thread Xavier Rocquefelte
Here is a document that can be usefull in which Peter discussed briefly 
valence hole for emission spectra: 
http://www.wien2k.at/reg_user/textbooks/WIEN2k_lecture-notes_2011/Blaha_xas_eels.pdf
Regards
Xavier

Dileep Krishnan dil...@jncasr.ac.in a écrit :

 Dear Xavier,

 Thank you very much for your quick and detailed reply. I am 
 calculating the valance optical properties, where the hole is 
 positioned in the valance states; JDOS, Energy loss function etc.

 Thanks

 - Original Message -
 From: Xavier Rocquefelte xavier.rocquefe...@univ-rennes1.fr
 To: A Mailing list for WIEN2k users wien@zeus.theochem.tuwien.ac.at
 Sent: Saturday, May 16, 2015 11:58:08 AM
 Subject: Re: [Wien] Adding excitonic effects


 To properly describe an exciton, which is a two-particle entitie, you 
 need a two-particle theory at least.
 DFT is a one particle theory, thus it is not suited to properly 
 describe excitonic effects.
 For solids you must use the Bethe Salpeter Equation (BSE) which is 
 implemented in other codes and very expensive in terms of 
 computational time and memory. Another strategy is to mimick the 
 effect of the exciton on the optical properties using DFT and a 
 corehole approach. This last strategy is pragmatic and not exact, but 
 it allows to deals with this 2-particle problem using a one-particle 
 approach in a mean-field manner. Usually, a half corehole is used. It 
 corresponds to the Slater transition state and it gives nice results 
 in general. What kind of transtions are your doing? Does it implies 
 core states? If yes you can easily remove half an electron in the 
 file case.inc. If the transition is from valence states, the problem 
 is more tricky to solve.
 Best Regards
 Xavier


 Dileep Krishnan dil...@jncasr.ac.in a écrit :

 Hi Users and Developers,

 I was calculating optical properties by OPTIC program. How can I add
 excitonic effects into the calculation?

 --
 Dileep Krishnan,
 Int. Ph. D Student,
 International Centre for Materials Science (ICMS),
 Jawaharlal Nehru Centre for Advanced Scientific Research (JNCASR),
 Jakkur, Bangalore-560064,
 INDIA.
 ___
 Wien mailing list
 Wien@zeus.theochem.tuwien.ac.at
 http://zeus.theochem.tuwien.ac.at/mailman/listinfo/wien
 SEARCH the MAILING-LIST at:
 http://www.mail-archive.com/wien@zeus.theochem.tuwien.ac.at/index.html


 ___
 Wien mailing list
 Wien@zeus.theochem.tuwien.ac.at
 http://zeus.theochem.tuwien.ac.at/mailman/listinfo/wien
 SEARCH the MAILING-LIST at:  
 http://www.mail-archive.com/wien@zeus.theochem.tuwien.ac.at/index.html

 --
 Dileep Krishnan,
 Int. Ph. D Student,
 International Centre for Materials Science (ICMS),
 Jawaharlal Nehru Centre for Advanced Scientific Research (JNCASR),
 Jakkur, Bangalore-560064,
 INDIA.
 ___
 Wien mailing list
 Wien@zeus.theochem.tuwien.ac.at
 http://zeus.theochem.tuwien.ac.at/mailman/listinfo/wien
 SEARCH the MAILING-LIST at:  
 http://www.mail-archive.com/wien@zeus.theochem.tuwien.ac.at/index.html
___
Wien mailing list
Wien@zeus.theochem.tuwien.ac.at
http://zeus.theochem.tuwien.ac.at/mailman/listinfo/wien
SEARCH the MAILING-LIST at:  
http://www.mail-archive.com/wien@zeus.theochem.tuwien.ac.at/index.html


Re: [Wien] Adding excitonic effects

2015-05-16 Thread Xavier Rocquefelte
To properly describe an exciton, which is a two-particle entitie, you need a 
two-particle theory at least. 
DFT is a one particle theory, thus it is not suited to properly describe 
excitonic effects. 
For solids you must use the Bethe Salpeter Equation (BSE) which is implemented 
in other codes and very expensive in terms of computational time and memory. 
Another strategy is to mimick the effect of the exciton on the optical 
properties using DFT and a corehole approach. This last strategy is pragmatic 
and not exact, but it allows to deals with this 2-particle problem using a 
one-particle approach in a mean-field manner. Usually, a half corehole is used. 
It corresponds to the Slater transition state and it gives nice results in 
general. What kind of transtions are your doing? Does it implies core states? 
If yes you can easily remove half an electron in the file case.inc. If the 
transition is from valence states, the problem is more tricky to solve. 
Best Regards
Xavier

Dileep Krishnan dil...@jncasr.ac.in a écrit :

 Hi Users and Developers,

 I was calculating optical properties by OPTIC program. How can I add 
 excitonic effects into the calculation?

 --
 Dileep Krishnan,
 Int. Ph. D Student,
 International Centre for Materials Science (ICMS),
 Jawaharlal Nehru Centre for Advanced Scientific Research (JNCASR),
 Jakkur, Bangalore-560064,
 INDIA.
 ___
 Wien mailing list
 Wien@zeus.theochem.tuwien.ac.at
 http://zeus.theochem.tuwien.ac.at/mailman/listinfo/wien
 SEARCH the MAILING-LIST at:  
 http://www.mail-archive.com/wien@zeus.theochem.tuwien.ac.at/index.html
___
Wien mailing list
Wien@zeus.theochem.tuwien.ac.at
http://zeus.theochem.tuwien.ac.at/mailman/listinfo/wien
SEARCH the MAILING-LIST at:  
http://www.mail-archive.com/wien@zeus.theochem.tuwien.ac.at/index.html


Re: [Wien] Wien2k OPTIC crashing for XMCD calculation

2015-05-14 Thread Xavier Rocquefelte
 

Did you try without the parallel version of WIEN2k? 
If the calculation is fast it will be an easy way to figure out if the problem 
is related to optic program or its parallel version. Indeed, the last step of 
the parallel version of optic program is to gather all the data and I remember 
that I had a similar problem long time ago (but only with the parallel version 
... look at opticpara and the way the symmat file is generated using cat 
command). 
Cheers
Xavier 

Santu Baidya santubaidya2...@gmail.com a écrit :

 Dear Prof. Blaha and Wien2k users,

    I am using wien2k version WIEN2k_14.1 (Release 8/9/2014) for calculating
 xmcd spectra of Co L23 edge for 8 atoms system. So I generated case.struct
 file. The did normal GGA+U spin polarized calculation using runsp_lapw
 -orb -p.

 Then I ran few commands as mentioned in userguide and wien2k forum:
 1) x kgen -p (for denser grid)
 2) x lapw1 -orb -up/dn -p (for eigenvalue calculations)
 3) x lapw2 -fermi -up/dn -p (for fermi energy calculation)
 4) editing case.inop file with additional line XMCD 1 L23
 5) x optic -orb -up -p

 Before optic it ran very well. But after running optic it crashes with
 error:

 running OPTIC in parallel mode
 [1] 13265
 [1]  + Done                          ( cd $PWD; $t $exe ${def}_${loop}.def;
 rm -f .lock_$lockfile[$p] )   ...
 [1] 13273
 [1]  + Done                          ( cd $PWD; $t $exe ${def}_${loop}.def;
 rm -f .lock_$lockfile[$p] )   ...
 [1] 13281
 [1]  + Done                          ( cd $PWD; $t $exe ${def}_${loop}.def;
 rm -f .lock_$lockfile[$p] )   ...
 [1] 13289
 [1]  + Done                          ( cd $PWD; $t $exe ${def}_${loop}.def;
 rm -f .lock_$lockfile[$p] )   ...
 [1] 13295
 [1]  + Done                          ( cd $PWD; $t $exe ${def}_${loop}.def;
 rm -f .lock_$lockfile[$p] )   ...
 **  OPTIC crashed!
 0.016u 0.032s 0:05.26 0.7% 0+0k 0+840io 0pf+0w
 error: command   /home/santu/wien2k/14/opticcpara -up -c upoptic.def
 failed

 After looking into other such error in wen2k forum I do not find its
 solution which led me to report it in the forum and asking for solution if
 known. My system has only 8 atoms which is not big. So why should it crash
 
 Even I tried in GGA+U+SO calculation I got same problem which led me to do
 normal GGA+U calculation for trial. But it gives me same optic crash.


 I would like to request you to help me to solve this problem in running
 optic program. Is wien2k optic program is unstable for XMCD calculation ?!


 Thanking you in advance,

 Santu Baidya
 University of Duisburg
 Germany
___
Wien mailing list
Wien@zeus.theochem.tuwien.ac.at
http://zeus.theochem.tuwien.ac.at/mailman/listinfo/wien
SEARCH the MAILING-LIST at:  
http://www.mail-archive.com/wien@zeus.theochem.tuwien.ac.at/index.html


Re: [Wien] reletivistic effects

2015-03-12 Thread Xavier Rocquefelte

By default, WIEN2k do scalar relativistic calculations, i.e.
- relativistic calculations for the core states inside the muffin-tin 
spheres

- non-relatativistic calculations for the valence states

To include relativistic effects for the valence states you must do 
spin-orbit calculations on top of your PBE-GGA calculation.

More details are available here:

http://www.wien2k.at/reg_user/textbooks/WIEN2k_lecture-notes_2013/Relativity-NCM.pdf 



and here:

http://www.wien2k.at/reg_user/textbooks/novak_lecture_on_spinorbit.pdf

Cheers

Xavier



Le 12/03/2015 07:08, Ридный Ярослав Максимович a écrit :

Hello, I am a user of software package WIEN2k.
In their calculations, I use PBE-GGA.
I've been doing calculations of iron.
Are the calculations in my relativistics effects.
If yes that as.
___
Wien mailing list
Wien@zeus.theochem.tuwien.ac.at
http://zeus.theochem.tuwien.ac.at/mailman/listinfo/wien
SEARCH the MAILING-LIST at:  
http://www.mail-archive.com/wien@zeus.theochem.tuwien.ac.at/index.html


___
Wien mailing list
Wien@zeus.theochem.tuwien.ac.at
http://zeus.theochem.tuwien.ac.at/mailman/listinfo/wien
SEARCH the MAILING-LIST at:  
http://www.mail-archive.com/wien@zeus.theochem.tuwien.ac.at/index.html


Re: [Wien] Choice of magnetization axis in the case with spin-orbit coupling

2014-07-30 Thread Rocquefelte

Dear Jianxin

Yes the direction is for the total magnetic moment (thus the sum of spin 
and orbital contribution).


Best Regards

Xavier


Le 29/07/2014 21:40, Zhu, Jianxin a écrit :

Dear Xavier,

Thanks a lot for the suggestion.
When we say such a direction is perhaps experimentally known and we 
impose a given direction in real-space for the magnetic moment 
 moments when we specify the (h k l) direction, do the magnetic 
moment mean the total magnetic moment, which is the sum of spin and 
orbital moments?
In my system, we have a significant contribution of orbital moment, 
which is even larger than the spin moment.


Best,

Jianxin

From: Rocquefelte xavier.rocquefe...@cnrs-imn.fr 
mailto:xavier.rocquefe...@cnrs-imn.fr

Organization: IMN Nantes
Reply-To: A Mailing list for WIEN2k users 
wien@zeus.theochem.tuwien.ac.at mailto:wien@zeus.theochem.tuwien.ac.at

Date: Tuesday, July 29, 2014 9:38 AM
To: A Mailing list for WIEN2k users wien@zeus.theochem.tuwien.ac.at 
mailto:wien@zeus.theochem.tuwien.ac.at
Subject: Re: [Wien] Choice of magnetization axis in the case with 
spin-orbit coupling


You must try different directions in order to find the magnetization
easy axis.
Such a direction is perhaps experimentally know in your case. Thus
you
can check by considering few crystallographic directions.

Should I always use positive values for (h k l)?
Or should I choose (0 0 1) when I prepare the spin moment to be
up, that
is, during the stage of instgen_lapw -ask


The answer is no. The spin sign (up/down) has no relationship with
the
direction of the magnetic moment.
You are just imposing a given direction in real-space for the
magnetic
moments when you specify the (h k l) direction.

Cheers

Xavier



Le 29/07/2014 16:04, Zhu, Jianxin a écrit :

Dear Peter and Wien2k users,

For the spin-orbit case, when we run initso_lapw, we are
prompted to
choose the moment direction

Please select the direction of the moment ( h k l )

I notice that the choices of e.g.,  (0  0  1) and (0  0  -1)
give slight
different values.
In particular, when I choose (0  0  1), I obtain from LDA+U+SO

:MMINT: MAGNETIC MOMENT IN INTERSTITIAL = -0.32640
:MMI001: MAGNETIC MOMENT IN SPHERE   1= -4.71648

:ORB001:  ORBITAL MOMENT:  0.0  0.0  4.59939
PROJECTION ON M
4.59939

while with (0 0 -1), I obtain from LDA+U+SO with the rest of
conditions
the same

:MMINT: MAGNETIC MOMENT IN INTERSTITIAL = -0.34861
:MMI001: MAGNETIC MOMENT IN SPHERE   1= -4.72342


:ORB001:  ORBITAL MOMENT:  0.0  0.0  4.52838
PROJECTION ON M
-4.52838


Should I always use positive values for (h k l)?
Or should I choose (0 0 1) when I prepare the spin moment to
be up, that
is, during the stage of instgen_lapw -ask
select spinpolarization up, dn or non-magnetic ( u, d, n )
u

while (0 0 -1) when I prepare the spin moment to be down, that
is, during
the stage of instgen_lapw -ask
select spinpolarization up, dn or non-magnetic ( u, d, n )
d


I appreciate if you can share your thought.



Thanks,

Jianxin

___
Wien mailing list
Wien@zeus.theochem.tuwien.ac.at
mailto:Wien@zeus.theochem.tuwien.ac.at
http://zeus.theochem.tuwien.ac.at/mailman/listinfo/wien
SEARCH the MAILING-LIST at:
http://www.mail-archive.com/wien@zeus.theochem.tuwien.ac.at/index.html


___
Wien mailing list
Wien@zeus.theochem.tuwien.ac.at
mailto:Wien@zeus.theochem.tuwien.ac.at
http://zeus.theochem.tuwien.ac.at/mailman/listinfo/wien
SEARCH the MAILING-LIST at:
http://www.mail-archive.com/wien@zeus.theochem.tuwien.ac.at/index.html



___
Wien mailing list
Wien@zeus.theochem.tuwien.ac.at
http://zeus.theochem.tuwien.ac.at/mailman/listinfo/wien
SEARCH the MAILING-LIST at:  
http://www.mail-archive.com/wien@zeus.theochem.tuwien.ac.at/index.html


___
Wien mailing list
Wien@zeus.theochem.tuwien.ac.at
http://zeus.theochem.tuwien.ac.at/mailman/listinfo/wien
SEARCH the MAILING-LIST at:  
http://www.mail-archive.com/wien@zeus.theochem.tuwien.ac.at/index.html


Re: [Wien] Choice of magnetization axis in the case with spin-orbit coupling

2014-07-29 Thread Rocquefelte
You must try different directions in order to find the magnetization 
easy axis.
Such a direction is perhaps experimentally know in your case. Thus you 
can check by considering few crystallographic directions.


Should I always use positive values for (h k l)?
Or should I choose (0 0 1) when I prepare the spin moment to be up, that
is, during the stage of instgen_lapw -ask


The answer is no. The spin sign (up/down) has no relationship with the 
direction of the magnetic moment.
You are just imposing a given direction in real-space for the magnetic 
moments when you specify the (h k l) direction.


Cheers

Xavier



Le 29/07/2014 16:04, Zhu, Jianxin a écrit :

Dear Peter and Wien2k users,

For the spin-orbit case, when we run initso_lapw, we are prompted to
choose the moment direction

Please select the direction of the moment ( h k l )

I notice that the choices of e.g.,  (0  0  1) and (0  0  -1) give slight
different values.
In particular, when I choose (0  0  1), I obtain from LDA+U+SO

:MMINT: MAGNETIC MOMENT IN INTERSTITIAL =   -0.32640
:MMI001: MAGNETIC MOMENT IN SPHERE   1=   -4.71648

:ORB001:  ORBITAL MOMENT:  0.0  0.0  4.59939 PROJECTION ON M
4.59939

while with (0 0 -1), I obtain from LDA+U+SO with the rest of conditions
the same

:MMINT: MAGNETIC MOMENT IN INTERSTITIAL =   -0.34861
:MMI001: MAGNETIC MOMENT IN SPHERE   1=   -4.72342


:ORB001:  ORBITAL MOMENT:  0.0  0.0  4.52838 PROJECTION ON M
-4.52838


Should I always use positive values for (h k l)?
Or should I choose (0 0 1) when I prepare the spin moment to be up, that
is, during the stage of instgen_lapw -ask
select spinpolarization up, dn or non-magnetic ( u, d, n )
u

while (0 0 -1) when I prepare the spin moment to be down, that is, during
the stage of instgen_lapw -ask
select spinpolarization up, dn or non-magnetic ( u, d, n )
d


I appreciate if you can share your thought.



Thanks,

Jianxin

___
Wien mailing list
Wien@zeus.theochem.tuwien.ac.at
http://zeus.theochem.tuwien.ac.at/mailman/listinfo/wien
SEARCH the MAILING-LIST at:  
http://www.mail-archive.com/wien@zeus.theochem.tuwien.ac.at/index.html


___
Wien mailing list
Wien@zeus.theochem.tuwien.ac.at
http://zeus.theochem.tuwien.ac.at/mailman/listinfo/wien
SEARCH the MAILING-LIST at:  
http://www.mail-archive.com/wien@zeus.theochem.tuwien.ac.at/index.html


Re: [Wien] (no subject)

2014-06-24 Thread Rocquefelte

It is mentioned in the userguide when the file case.struct is described:
- atomic positions are specified in fractional coordinates.

Cheers

Xavier


Le 24/06/2014 14:21, sikandar azam a écrit :

Dear All
Please clear me that the atomic position which we are using in wien2k 
are in cartesian coordinates or in fractional.

with regard
azam


___
Wien mailing list
Wien@zeus.theochem.tuwien.ac.at
http://zeus.theochem.tuwien.ac.at/mailman/listinfo/wien
SEARCH the MAILING-LIST at:  
http://www.mail-archive.com/wien@zeus.theochem.tuwien.ac.at/index.html




smime.p7s
Description: Signature cryptographique S/MIME
___
Wien mailing list
Wien@zeus.theochem.tuwien.ac.at
http://zeus.theochem.tuwien.ac.at/mailman/listinfo/wien
SEARCH the MAILING-LIST at:  
http://www.mail-archive.com/wien@zeus.theochem.tuwien.ac.at/index.html


Re: [Wien] case.inkram

2014-05-26 Thread Rocquefelte

You must first use x addjoint-updn and only after use x kram.
Look at the user guide for more details about addjoint-updn.

Cheers

Xavier


Le 26/05/2014 17:00, ivan paul a écrit :

Dear users

I have a quistion about inkram fille ; should i put one value in the 4 
th line or two in a SP case.

 --- top of file: case.inkram ---
  0.0gamma for Lorentz broadening (in units selected in joint)
  0.0energy shift (scissors operator) (in units selected in joint)
   1  add intraband contributions? yes/no: 1/0
--- 12.60   plasma frequencies (for each ``column'' in case.injoint)
  0.20   Gammas for Drude terms (for each ``column'' in case.injoint)
 --- bottom of file ---
Thx.


___
Wien mailing list
Wien@zeus.theochem.tuwien.ac.at
http://zeus.theochem.tuwien.ac.at/mailman/listinfo/wien
SEARCH the MAILING-LIST at:  
http://www.mail-archive.com/wien@zeus.theochem.tuwien.ac.at/index.html




smime.p7s
Description: Signature cryptographique S/MIME
___
Wien mailing list
Wien@zeus.theochem.tuwien.ac.at
http://zeus.theochem.tuwien.ac.at/mailman/listinfo/wien
SEARCH the MAILING-LIST at:  
http://www.mail-archive.com/wien@zeus.theochem.tuwien.ac.at/index.html


  1   2   3   >